olimpiada matematica española 1993-2010

132
OLIMPIADA MATEMÁTICA ESPAÑOLA Fase Nacional 1993 (Madrid) Primera sesión 1.- En una reunión hay 201 personas de 5 nacionalidades diferentes. Se sabe que, en cada grupo de 6, al menos dos tienen la misma edad. Demostrar que hay al menos 5 personas del mismo país, de la misma edad y del mismo sexo. 2.- Escrito el triángulo aritmético: 0 1 2 3 4 ........ 1991 1992 1993 1 3 5 7 ...... ........ ........ 3983 3985 4 8 12 ...... ...... ........ ........ ........ 7968 ........................................................................................... donde cada número es la suma de los dos que tiene encima (cada fila tiene un número menos y en la última sólo hay un número). Razonar que el último número es múltiplo de 1993. 3.- Justificar razonadamente que, en cualquier triángulo, el diámetro de la circunferencia inscrita no es mayor que el radio de la circunferencia circunscrita. Segunda sesión 4.- Demostrar que para todo número primo p distinto de 2 y de 5, existen infinitos múltiplos de p de la forma 1111......1 (escrito sólo con unos). 5.- Se dan 16 puntos formando una cuadrícula como en la figura: De ellos se han destacado A y D. Se pide fijar de todos los modos posibles otros dos puntos B y C con la condición de que las seis distancias determinadas por los cuatro puntos sean distintas. En ese conjunto de cuaternas, estudiar: a) Cuántas figuras de 4 puntos existen con las condiciones del enunciado. b) Cuántas de ellas son geométricamente distintas, es decir, no deducibles unas de otras por transformaciones de igualdad. c) Si cada punto se designa por un par de enteros (X i , Y i ), razonar que la suma: |X i - X j | + | Y i - Y j | extendida a los seis pares AB, AC, AD, BC, BD, CD es constante. 6.- Una máquina de juego de un casino tiene una pantalla en la que se ofrece un esquema como el de la figura. Para comenzar el juego aparece una bola en el punto S. A cada impulso que recibe del jugador, esa bola se mueve hasta una de las letras inmediatas con la misma probabilidad para cada una de ellas. La partida termina al ocurrir el primero de los dos hechos siguientes:

Upload: vicente-vazquez

Post on 28-Jun-2015

666 views

Category:

Documents


17 download

DESCRIPTION

Problemas de las olimpiadas matemáticas desde 1993 hasta 2010.

TRANSCRIPT

Page 1: Olimpiada Matematica Española 1993-2010

OLIMPIADA MATEMÁTICA ESPAÑOLA

Fase Nacional 1993 (Madrid)

Primera sesión

1.- En una reunión hay 201 personas de 5 nacionalidades diferentes. Se sabe que, en cada grupo de 6, al menos

dos tienen la misma edad. Demostrar que hay al menos 5 personas del mismo país, de la misma edad y del mismo

sexo.

2.- Escrito el triángulo aritmético:

0 1 2 3 4 ........ 1991 1992 1993

1 3 5 7 ...... ........ ........ 3983 3985

4 8 12 ...... ...... ........ ........ ........ 7968

...........................................................................................

donde cada número es la suma de los dos que tiene encima (cada fila tiene un número menos y en la última sólo

hay un número). Razonar que el último número es múltiplo de 1993.

3.- Justificar razonadamente que, en cualquier triángulo, el diámetro de la circunferencia inscrita no es mayor

que el radio de la circunferencia circunscrita.

Segunda sesión

4.- Demostrar que para todo número primo p distinto de 2 y de 5, existen infinitos múltiplos de p de la forma

1111......1 (escrito sólo con unos).

5.- Se dan 16 puntos formando una cuadrícula como en la figura:

De ellos se han destacado A y D. Se pide fijar de todos los modos posibles otros dos

puntos B y C con la condición de que las seis distancias determinadas por los cuatro

puntos sean distintas. En ese conjunto de cuaternas, estudiar:

a) Cuántas figuras de 4 puntos existen con las condiciones del enunciado.

b) Cuántas de ellas son geométricamente distintas, es decir, no deducibles unas de otras

por transformaciones de igualdad.

c) Si cada punto se designa por un par de enteros (Xi, Yi), razonar que la suma:

|Xi - Xj| + | Yi - Yj|

extendida a los seis pares AB, AC, AD, BC, BD, CD es constante.

6.- Una máquina de juego de un casino tiene una pantalla en la que se ofrece un

esquema como el de la figura. Para comenzar el juego aparece una bola en el punto S. A

cada impulso que recibe del jugador, esa bola se mueve hasta una de las letras

inmediatas con la misma probabilidad para cada una de ellas. La partida termina al

ocurrir el primero de los dos hechos siguientes:

Page 2: Olimpiada Matematica Española 1993-2010

a) La bola vuelve a S y entonces el jugador pierde.

b) La bola llega a G y entonces el jugador gana.

Se pide la probabilidad de que el jugador gane y la duración media de las partidas.

Page 3: Olimpiada Matematica Española 1993-2010

Problemas de la XXIX Olimpiada

1.- En una reunión hay 201 personas de 5 nacionalidades diferentes. Se sabe que, en cada grupo de

6, al menos dos tienen la misma edad. Demostrar que hay al menos 5 personas del mismo país, de la

misma edad y del mismo sexo.

Solución (Mª Gaspar Alonso-Vega)

Si en cada grupo de 6 personas, 2 son de la misma edad, sólo puede haber 5 edades diferentes, ya

que, si hubiese 6 edades diferentes, eligiendo una persona de cada edad tendríamos 6 personas de

edades distintas contra la hipótesis.

Como 200 = 2 · 100 + 1⇒ al menos hay 101 personas del mismo sexo.

101 = 5 · 20 + 1 ⇒ al menos hay 21 personas de la misma edad y sexo.

21 = 4 · 5 + 1 ⇒ al menos hay 5 personas de la misma nacionalidad, edad y sexo.

2.- Escrito el triángulo aritmético:

0 1 2 3 4 ............. 1991 1992 1993

1 3 5 7 ............................ 3983 3985

4 8 12 ........................................ 7968

.........................................................................

donde cada número es la suma de los dos que tiene encima (cada fila tiene un número menos y en la

última sólo hay un número). Razonar que el último número es múltiplo de 1993.

Solución.

Si representamos los elementos de la primera fila por a0, a1, a2, ........

los elementos de la segunda serán: a0 + a1, a1 + a2, a2 + a3, ..............

los de la tercera serán : a0 + 2a1 + a1, a1 + 2a2 + a3, ..............

para la cuarta : a0 + 3a1 + 3a1 + a1, a1 + 3a2 + 3a3 + a4,............

Supongamos que los dos primeros elementos bp,0 y bp,1 de la fila p-ésima son:

bp

ap

ap

pap p, .......0 0 1 1

1

0

1

1

1

1=

+

+ +

− ; b

pa

pa

p

pap p, .......1 1 2

1

0

1

1

1

1=

+

+ +

entonces, el primer elemento de la fila siguiente será :

bp

ap

ap

pap p+

=

+

+ +

1 0 0 10 1, ....... (*)

en nuestro caso la primera fila tiene 1994 elementos, la segunda 1993, ... y la última corresponde a p

+ 1 = 1994 y su único elemento será

b1994

1993

00

1993

11

1993

19931993=

+

+ +

• • ....... •

Page 4: Olimpiada Matematica Española 1993-2010

Al ser 1993 primo, 1993

k

es múltiplo de 1993 para todo k menor que 1993 y por tanto b1993 es

múltiplo de 1993.

3.- Justificar razonadamente que, en cualquier triángulo, el diámetro de la circunferencia inscrita no

es mayor que el radio de la circunferencia circunscrita.

Solución (F. Bellot)

La desigualdad propuesta, R - 2r ≥ 0 es una consecuencia del teorema de Euler. “Si I, O son el

incentro y el circuncentro de un triángulo, r y R los radios de las circunferencias inscrita y

circunscritas, se verifica: IO2 = R

2 - 2Rr”.

Entonces IO2 = R (R - 2r) ≥ 0 ⇒ R - 2r ≥ 0.

4.- Demostrar que para todo número primo p distinto de 2 y de 5, existen infinitos múltiplos de p de

la forma 1111......1 (escrito sólo con unos).

Solución (Alvaro Begué Aguado)

Veamos primero que p tiene infinitos múltiplos de la forma 999...9. Consideremos la sucesión: 9,

99, 999, ......,999...9 (el último tiene n nueves). Entonces se tiene:

9 = 10 - 1; 99 = 102 - 1; 999 = 10

3 - 1;.......999..9 = 10

n - 1

en la sucesión hay infinitos términos de la forma 10p-1

- 1 con p ≠ 2, p ≠ 5 y p primo.

Puesto que, por el teorema de Fermat: 10p-1

- 1 ≡ 1 (mód p) si p ≠ 2, p ≠ 5 la afirmación queda

demostrada.

Finalmente 999...9 = 9 · 111...1 entonces si p es primo con 9 (p ≠ 3), p divide al producto, es primo

con 9 luego divide a 111...1.

Queda el caso p = 3 que es evidente ya que los infinitos números: 111; 111111, .......... son

múltiplos de tres.

5.- Se dan 16 puntos formando una cuadrícula como en la figura:

De ellos se han destacado A y D. Se pide fijar de todos los modos posibles otros dos puntos B y C

con la condición de que las seis distancias determinadas por los cuatro puntos sean distintas. En ese

conjunto de cuaternas, estudiar:

a) Cuántas figuras de 4 puntos existen con las condiciones del enunciado.

b) Cuántas de ellas son geométricamente distintas, es decir, no deducibles unas de otras por

transformaciones de igualdad.

c) Si cada punto se designa por un par de enteros (Xi, Yi), razonar que la suma:

|Xi - Xj| + | Yi - Yj| extendida a los seis pares AB, AC, AD, BC, BD, CD es constante.

A

D

Page 5: Olimpiada Matematica Española 1993-2010

Solución

El problema admite dos ejes de simetría coincidentes con las diagonales del cuadrado.

Clasificaremos las soluciones posibles por la posición del punto B respecto del vértice A. Usaremos

coordenadas enteras con origen en A.

Las tres posiciones “fundamentales” (no deducibles unas de otras por las simetrías anteriores) son

aquellas en las que B está en los puntos de coordenadas (0,1); (0,2) y (1,1) para cada una de ellas

dibujamos us esquema con las posibles posiciones del punto C.

Las posiciones “prohibidas” se dibujan en negro, la posición de B en gris y las de Ci en blanco.

Un criterio general para prohibir ubicaciones es localizar aquellos puntos que estan en la

“mediatriz” de dos puntos ya situados. Como A y D son dados y fijos, la diagonal principal siempre

contiene puntos “prohibidos”

El esquema de la izquierda contiene 4 posiciones “originales” y cada una de ellas genera otras

cuatro por aplicación de las dos simetrías, en total 16.

El esquema del centro contiene 3 posiciones “originales” y cada una de ellas genera otras cuatro por

aplicación de las dos simetrías, en total 12.

El esquema de la derecha contiene 1 posición “original” que genera otras cuatro por aplicación de

las dos simetrías, en total 4.

Por tanto existen 32 posiciones posibles y 8 “originales”, esto contesta a los apartados a) y b).

Para el apartado c) hay que suponer que los enteros asignados a cada punto son sus coordenadas en

un origen cualquiera, nosotros supondremos que el origen está en A con lo que las coordenadas de

A son (0,0) y las de D(3,0).

los seis sumandos corresponden a las parajas AB, AC, AD, BC, BD y CD.

El correspondiente a AD es constante y vale 3+3 = 6.

Los correspondientes a AB y BD valen en conjunto siempre 6 ya que A está en fila inferior y

columna izquierda y D en la fila superior y columna derecha.

Por el mismo motivo los sumandos correspondientes a AC y CD valen entre los dos siempre 6.

Sólo queda el sumando |Xi - Xj| + | Yi - Yj| correspondiente a BC que por simple comprobación en

todos los casos “originales” vale siempre 3.

La suma completa es entonces constante y vale 6 + 6 + 6 + 3 = 21.

A

D

A

D

A

D

B

B

B

C1

C2

C3

C4

C1

C2

C3

C1

C1

Page 6: Olimpiada Matematica Española 1993-2010

6.- Una máquina de juego de un casino tiene una pantalla en la que se ofrece un esquema como el de

la figura. Para comenzar el juego aparece una bola en el punto S. A cada impulso que recibe del

jugador, esa bola se mueve hasta una de las letras inmediatas con la misma probabilidad para cada

una de ellas. La partida termina al ocurrir el primero de los dos hechos

siguientes:

a) La bola vuelve a S y entonces el jugador pierde.

b) La bola llega a G y entonces el jugador gana.

Se pide la probabilidad de que el jugador gane y la duración media de las

partidas.

Solución (F. Bellot)

Podemos representar el desarrollo del juego mediante un diagrama en árbol:

La probabilidad de que el juego tenga longitud 2 es 1

3

La probabilidad de que el juego tenga longitud 4 es :1

3

1

2

1

32 2

2

32• • • • =

La probabilidad de que el juego tenga longitud 6 es :1

3

1

2

1

32 2 2

1

3

2

3

2

3• • • • • • = , etc, en general

la probabilidad de que el juego tenga longitud 2n es: 2

3

1n

n

Entonces, la duración media M de un juego es la suma de cada longitud por la probabilidad

respectiva :

M n n

n

nn n

n

= =

=

=

∑ ∑2

32

2

3

1

1 1

serie aritmético-geométrica que se suma por el mismo método que la geométrica:

M MM

M− = = +

= ⇒ = =2

3 3

2

3

2

3

12

3

2 2 3 6

2

S G

A B

C

D

S C D

A

G C D

B

S C D

A

C D G

B

C D

A

S

S

1

1/3

1/2

1/3

1/3

Page 7: Olimpiada Matematica Española 1993-2010

La probabilidad P de ganar será la suma de las probabilidades de ganar en 4 pasos más la de que

gane en 6 pasos ...etc.:

P = + + + =1

3

2

3

2

3

1

32 3

2

4 ....

Page 8: Olimpiada Matematica Española 1993-2010
Page 9: Olimpiada Matematica Española 1993-2010

OLIMPIADA MATEMÁTICA ESPAÑOLA

Fase Nacional 1994 (Madrid)Primera sesión

1.- Demostrar que si entre los infinitos términos de una progresión aritmética de números enteros positivos hay

un cuadrado perfecto, entonces infinitos términos de la progresión son cuadrados perfectos.

2.- Sea OXYZ un triedro trirectángulo de vértice O y aristas X, Y, Z. Sobre la arista Z se toma un punto fijo C,

tal que OC = c. Sobre X e Y se toman respectivamente dos puntos variables P y Q de modo que la suma OP + OQ

sea una constante dada k. Para cada par de puntos P y Q, los cuatro puntos O, C, P, Q están en una esfera, cuyo

centro W se proyecta sobre el plano OXY. Razonar cuál es el lugar geométrico de esa proyección. Razonar

también cuál es el lugar geométrico de W.

3.- Una oficina de Turismo va a realizar una encuesta sobre número de días soleados y número de días

lluviosos que se dan en el año. Para ello recurre a seis regiones que le transmiten los datos de la siguiente tabla:

Región Soleados o lluviosos Inclasificables

A 336 29

B 321 44

C 335 30

D 343 22

E 329 36

F 330 35

La persona encargada de la encuesta no es imparcial y tiene esos datos más detallados. Se da cuenta de que,

prescindiendo de una de las regiones, la observación da un número de días lluviosos que es la tercera parte del de

días soleados. Razonar cuál es la región de la que prescindirá.

Segunda sesión

4.- El ángulo A del triángulo isósceles ABC mide 2/5 de recto, siendo iguales sus ángulos B y C. La bisectriz

de su ángulo C corta al lado opuesto en el punto D. Calcular las medidas de los ángulos del triángulo BCD.

Expresar la medida a del lado BC en función de la medida b del lado AC, sin que en la expresión aparezcan

razones trigonométricas.

5.- Con 21 fichas de damas, unas blancas y otras negras, se forma un rectángulo de 3x7. Demostrar que

siempre hay cuatro fichas del mismo color situadas en los vértices de un rectángulo.

6.- Un polígono convexo de n lados se descompone en m triángulos, con los interiores disjuntos, de modo que

cada lado de esos m triángulos lo es también de otro triángulo contíguo o del polígono dado. Probar que m + n es

par. Conocidos n y m hallar el número de lados distintos que quedan en el interior del polígono y el número de

vértices distintos que quedan en ese interior.

Page 10: Olimpiada Matematica Española 1993-2010

Problemas de la XXX Olimpiada Madrid 1994

1.- Demostrar que si entre los infinitos términos de una progresión aritmética de

números enteros positivos hay un cuadrado perfecto, entonces infinitos términos de la

progresión son cuadrados perfectos.

Solución

Bastará probar que a partir de un cuadrado perfecto podemos construir otro. Sea la progresión:

a2, a

2 + d, a

2 + 2d, ......,a

2 + kd......

Como (a + d)2 = a

2 + 2ad + d

2 = a

2 + (2a + d)d, basta tomar k = 2a + d para obtener otro

cuadrado en la progresión.

2.- Sea OXYZ un triedro trirectángulo de vértice O y aristas X, Y, Z. Sobre la arista Z se

toma un punto fijo C, tal que OC = c. Sobre X e Y se toman respectivamente dos puntos

variables P y Q de modo que la suma OP + OQ sea una constante dada k. Para cada par

de puntos P y Q, los cuatro puntos O, C, P, Q están en una esfera, cuyo centro W se

proyecta sobre el plano OXY. Razonar cuál es el lugar geométrico de esa proyección.

Razonar también cuál es el lugar geométrico de W.

Solución

En la figura se muestran con trazo discontínuo

las circunferencias que resultan de intersecar la

esfera con los planos coordenados. Las

proyecciones del centro W de la esfera sobre

estos planos coinciden con los centros de estas

circunferencias (denotados F, G y H en la figura)

y al ser el triedro trirectángulo, F, G y H estan en

los puntos medio de los segmentos PQ, QC y CP

que son diámetros de sus circunferencias.

Parametrizando con la distancia OP= λ tenemos trivialmente en la referencia OXYZ la

siguientes coordenadas:

P(λ,0,0); Q(0,k-λ,0); C(0,0,c);

Fk k c c k cλ λ λ λ λ λ

2 20 0

2 2 20

2 2 2 2, , ; , , ; , , ; , ,

G H W

El lugar de F es la recta x yk

+ =2

del plano XOY. El lugar de W es una recta paralela a la

anterior situada en el plano zc

=2

, más concretamente es la intersección de los planos:

x yk

zc

+ =

=

2

2

X

Y

Z

C

P

Q

WH

G

F

O

Page 11: Olimpiada Matematica Española 1993-2010

3.- Una oficina de Turismo va a realizar una encuesta sobre número de días soleados y

número de días lluviosos que se dan en el año. Para ello recurre a seis regiones que le

transmiten los datos de la siguiente tabla:

Región Soleados o lluviosos Inclasificables

A 336 29

B 321 44

C 335 30

D 343 22

E 329 36

F 330 35

La persona encargada de la encuesta no es imparcial y tiene esos datos más detallados. Se da

cuenta de que, prescindiendo de una de las regiones, la observación da un número de días

lluviosos que es la tercera parte del de días soleados. Razonar cuál es la región de la que

prescindirá.

Solución

Al suprimir una región, la suma de días soleados o lluviosos de las restantes ha de ser múltiplo

de 4. Esta suma vale para las seis regiones 1994 que dividido entre 4 da resto 2. El único dato

de esta columna que da resto 2 al dividirlo entre 4 es 330 correspondiente a la región F.

Suprimiendo esta región quedan entre las cinco restantes 416 días lluviosos y 3·416 = 1248

días soleados.

4.- El ángulo A del triángulo isósceles ABC mide 2/5 de recto, siendo iguales sus ángulos

B y C. La bisectriz de su ángulo C corta al lado opuesto en el punto D. Calcular las

medidas de los ángulos del triángulo BCD. Expresar la medida a del lado BC en función

de la medida b del lado AC, sin que en la expresión aparezcan razones trigonométricas.

Solución

Con los datos del enunciado tenemos:

en el triángulo ABC ∠ BAC = 36º; ∠ ABC = ∠ ACB = 72º

en el triángulo CBD ∠ BCD = 36º; ∠ CDB = ∠ BDC = 72º

en el triángulo ADC ∠ DAC = ∠ ACD = 72º; ∠ ADC = 108º

por tanto ∆BCD y ∆ADC son isósceles y ademas ∆BCD es

semejante al ∆ABC.

Para los lados se tiene: DC = AD = a; BD = b - a.

Expresando la proporcionalidad derivada de la semejanza anterior:

b a

a

a

ba b ab a ab b

a

b

a

b

−= ⇔ = − ⇔ + − = ⇔

+ − =2 2 2 2

2

0 1 0

y resolviendo queda ( )a

ba

b=

−⇔ =

−5 1

2

5 1

2 es decir a es la sección áurea de b.

A

B

D

b-aa

a

C

b

Page 12: Olimpiada Matematica Española 1993-2010

5.- Con 21 fichas de damas, unas blancas y otras negras, se forma un rectángulo de 3x7.

Demostrar que siempre hay cuatro fichas del mismo color situadas en los vértices de un

rectángulo.

Solución

Dispondremos el tablero en posición vertical, es decir, con 7 filas y 3

columnas. Asignaremos el color blanco a la cifra 0 y el negro a la

cifra 1. De este modo cada fila representa un número escrito en base

2.

En primer lugar es fácil ver que si en una fila se colocan todas las

fichas del mismo color, por ejemplo el negro, necesariamente habrá

un rectángulo ya que no podemos colocar en ninguna fila dos fichas

negras y sólo podemos llenar un máximo de 5 filas en total sin

formar rectángulo.

Por otra parte si dos números son iguales sus filas forman rectángulo, luego todas las filas han

de representar números distintos. Por la consideración anterior hemos de excluir los números

000 y 111. Con tres cifras en base dos existen 23 = 8 números distintos, quitando los

anteriores quedan 6 para 7 filas por lo que necesariamente hemos de repetir y formar

rectángulo. El problema tendría solución en un tablero de 3x6 tal como se muestra en la

figura.

6.- Un polígono convexo de n lados se descompone en m triángulos, con los interiores

disjuntos, de modo que cada lado de esos m triángulos lo es también de otro triángulo

contíguo o del polígono dado. Probar que m + n es par. Conocidos n y m hallar el

número de lados distintos que quedan en el interior del polígono y el número de vértices

distintos que quedan en ese interior.

Solución

Como hay m triángulos, hay 3m lados; de ellos 3m - n son interiores, y como lado interior

pertenece a dos triángulos, hay 3

2

m n− lados interiores distintos. En particular 3m - n es par,

luego m y n tienen la misma paridad y m + n es par.

Supongamos que el número de vértices v sólo depende de m y n. Razonemos por inducción

sobre v.

Si no hay ningún vértice interior (v = 0), uniendo un vértice del polígono con los otros, se

divide en n - 2 = n + 2v - 2 triángulos.

Supongamos que hay v vértices interiores y n + 2v - 2 triángulos. Al añadir un vértice hay dos

posibilidades:

a) El vértice está en el interior de un triángulo, entonces, para que se cumplan las condiciones

del enunciado, debe unirse a cada uno de los tres vértices del triángulo que se divide en tres y

el número de triángulos ahora es: n + 2v - 2 + 2 = n + 2(v + 1) - 2.

b) El vértice está en un lado, entonces hay que unirlo con el vértice opuesto de cada uno de los

dos triángulos que comparten ese lado, cada triángulo se descompone en dos y el número de

triángulos es ahora: n + 2v - 2 + 2 = n + 2(v + 1) - 2.

En conclusión:

m = n + 2v - 2 ⇒ =− +

vm n 2

2

1

2

3

4

5

6

Page 13: Olimpiada Matematica Española 1993-2010
Page 14: Olimpiada Matematica Española 1993-2010

OLIMPIADA MATEMÁTICA ESPAÑOLA

Fase nacional 1995 (Castellón)

Primera sesión

1.- Se consideran conjuntos A de cien números naturales distintos, que tengan la propiedad de que si a, b y c

son elementos cualesquiera de A (iguales o distintos), existe un triángulo no obtusángulo cuyos lados miden a, b y c

unidades.

Se denomina S(A) a la suma de los perímetros considerados en la definición de A. Calcula el valor mínimo de S(A)

2.- Recortamos varios círculos de papel (no necesariamente iguales) y los extendemos sobre una mesa de

modo que haya algunos solapados (con parte interior común), pero de tal forma que no haya ningún círculo dentro

de otro.

Prueba que es imposible ensamblar las piezas que resultan de recortar las partes no solapadas y componer con ellas

círculos distintos.

3.- Por el baricentro G de un triángulo ABC se traza una recta que corta al lado AB en P y al lado AC en Q.

Demuestra que:

Segunda sesión

4.- Siendo p un número primo, halla las soluciones enteras de la ecuación:

p.(x + y) = x.y

5.- Demuestra que en el caso de que las ecuaciones:

x3 + mx - n = 0

nx3 - 2 m

2 x2 - 5mnx - 2m

3 - n

2 = 0 (n no nulo)

tengan una raíz común, la primera tendrá dos raíces iguales y determina entonces las raíces de las dos ecuaciones

en función de n.

6.- En la figura, AB es un segmento fijo y C un punto variable dentro

de él. Se construyen triángulos equiláteros de lados AC y CB, ACB' y

CBA' en el mismo semiplano definido por AB, y otro de lado AB, ABC'

en el semiplano opuesto. Demuestra:

a) Las rectas AA', BB' y CC' son concurrentes.

b) Si llamamos P al punto común a las tres rectas del apartado a), hallar el

lugar geométrico de P cuando C varía en el segmento AB.

Page 15: Olimpiada Matematica Española 1993-2010

lugar geométrico de P cuando C varía en el segmento AB.

c) Los centros A'', B'' y C'' de los tres triángulos forman un triángulo

equilátero.

d) Los puntos A'', B'', C'' y P están sobre una circunferencia.

Page 16: Olimpiada Matematica Española 1993-2010

Olimpiada Española 1995

1.- Se consideran conjuntos A de cien números naturales distintos, que tengan la propiedad de que si

a, b y c son elementos cualesquiera de A (iguales o distintos), existe un triángulo no obtusángulo

cuyos lados miden a, b y c unidades.

Se denomina S(A) a la suma de los perímetros considerados en la definición de A. Calcula el valor

mínimo de S(A).

Solución:

Si n es el menor de los elementos de A y m el mayor, al tener A cien elementos distintos, será

m ≥ n + 99. Para que el triángulo isósceles de lados n, n, m sea no obtusángulo debe ocurrir que

m2 ≤ 2 n

2 ; si m es lo menor posible, m = n + 99 deberá ser (n + 99)

2 ≤ 2n

2 , o sea:

( )n - 198n - 99 0 n 99 n 99 1 + 22 2 2≥ ⇔ ≥ + + ⇔ ≥ ⇔ ≥99 99 2402n .

Si n < 240, es seguro que el conjunto no cumple la condición del enunciado pues

m2 ≥ (n+99)

2 ≥ 2n

2

y el triángulo de lados n, n, m no puede ser no obtuságulo. En particular la condición se cumple para

el conjunto:

A = {240, 241, 242, ...., 339}

Cualquier otro conjunto que cumpla la condición, tendrá sus elementos respectivamente iguales o

mayores que los de éste. Este es, por tanto el que da lugar al mínimo S(A).

El número de triángulos que debe considerarse es el de variaciones ternarias con repetición de los

elementos de A, que es 1003 = 1000000, con lo que el número de lados en total será de 3000000; de

ellos habrá 30000 de longitud 240, otros tantos de longitud 241, etc. Luego

( )

S A( ) • ( .... ) ••

= + + + + =+

=30000 240 241 242 339 30000100 240 339

2868500000 unidades.

Este es el valor mínimo buscado.

2.- Recortamos varios círculos de papel (no necesariamente iguales) y los extendemos sobre una

mesa de modo que haya algunos solapados (con parte interior común), pero de tal forma que no

haya ningún círculo dentro de otro.

Prueba que es imposible ensamblar las piezas que resultan de recortar las partes no solapadas y

componer con ellas círculos distintos.

Solución:

La frontera de las piezas recortadas (que no sean círculos

completos) está formada por arcos cóncavos y convexos (vistos

desde fuera) que se cortan en puntos que llamaremos vértices. En

un vértice pueden concurrir dos arcos cóncavos o uno cóncavo y

otro convexo, pero nunca dos convexos ya que éstos únicamente

provienen de la frontera de los círculos iniciales. Además, los

ángulos que forman los arcos en cada vértice no son de 0º ni de

180º ya que excluimos las tangencias interiores.

Page 17: Olimpiada Matematica Española 1993-2010

Supongamos que tenemos un círculo obtenido ensamblando piezas recortadas. Existe al menos un

punto P de la frontera de dicho círculo en el que concurren tres o más arcos de la frontera de las

pieza ensambladas (P es vértice de dos o más piezas). La tangente al

círculo en P deja a todos los arcos en un mismo semiplano. Elegido

un sentido de rotación en P a partir de la tangente, y avanzando en

este sentido, el primer arco que encontramos es convexo y el último

cóncavo. Por lo tanto es necesario que existan dos arcos consecutivos

uno convexo y el otro cóncavo los cuales forman parte de la frontera

de una de las piezas ensambladas. Como el arco que forman dichas

piezas no puede ser ni 0º ni 180º, el punto P es un vértice de la pieza.

Esto es contradictorio pues en ningún vértice pueden concurrir dos

arcos convexos vistos desde fuera.

Nota. Hay que entender en el enunciado.que quedan excluidas las tangencias interiores. De no ser

así pueden encontrarse contraejemplos como el “despiece” que se muestra en la figura:

3.- Por el baricentro G de un triángulo ABC se traza una recta que corta al lado AB en P y al lado

AC en Q. Demuestra que: PB

PA

QC

QA• ≤

1

4

Solución:

Dupliquemos el triángulo trazando AD paralela a BC y CD paralela a BA como muestra la figura y

tomemos la longitud del lado AB como unidad. Llamando M a la intersección de CD con la recta

PQ y x = PB; 1-x = AP, tenemos:

Por semejanza de ∆AQP y ∆QMC: QC

QA

MC

AP

MC

x= =

−1

Por semejanza de ∆GPB y ∆GMD: PB

MD

GB

GD= =

1

2.

Luego: MD = 2x y MC = 1 - 2x.

Sustituyendo en el primer miembro de la relación del

enunciado queda:

( )PB

PA

QC

QA

x x

xx x x•

( )

( )≤ ⇔

−≤ ⇔ − + ≥ ⇔ − ≥

1

4

1 2

1

1

49 6 1 0 3 1 0

2

2 2

Relación válida para cualquier x.

La igualdad se alcanza para PB x MC= = ⇔ =1

3

1

3 ⇔ PQ paralela al lado BC.

P

2

P

G

A

C

D

QM

B

Page 18: Olimpiada Matematica Española 1993-2010

4.- Halla las soluciones enteras de la ecuación:

p·(x + y) = x·y

siendo p un número primo.

Solución:

Ya que p es primo, p ≠ 0 y p ≠ 1. De la ecuación resulta que p divide a x o p divide a y. Como la

ecuación es simétrica respecto de x e y, si (α, β) es solución, también lo será (β, α).

Si p divide a x, x = p·a, (a∈Z) la ecuación se puede poner como:

( )p pa y pay pa y ay ypa

a+ = ⇒ + = ⇒ =

−)

1 ya que a es entero

además a y a - 1 son primos entre sí, luego a - 1 divide a p. Al ser p primo sólo hay cuatro

posibilidades: a - 1 = ± 1 y a - 1 = ± p.

Examinemos todos los casos.

i) a - 1 = -1, entonces a = 0, x = 0, y = 0

ii) a - 1 = 1, entonces a = 2, x = 2p ⇒ =−

=yp

p2

2 12 .

iii) a - 1 = p, entonces a = p + 1, x = p(p + 1) ( )

⇒ =+

+ −= +y

p p

pp

1

1 11 .

iiii) a - 1 = -p, entonces a = 1 - p, x = p(1 - p) ( )

⇒ =−

− −= −y

p p

pp

1

1 11 .

En resumen las soluciones son: (0, 0); (2p, 2p); (p(p+1), p+1); (p(1-p), p-1); y por la simetría

añadimos (p+1, p(p+1)); (p-1, p(1-p)).

5.- Demuestra que en el caso de que las ecuaciones:

x3 + mx - n = 0 , nx

3 - 2 m

2 x

2 - 5mnx - 2m

3 - n

2 = 0 (n ≠ 0)

tengan una raíz común, la primera tendrá dos raíces iguales y determina entonces las raíces de las

dos ecuaciones en función de n.

Solución:

Sea α la raíz común de ambas ecuaciones. Entonces

α3 + mα = n (1)

y sustituyendo en la segunda ecuación se obtiene, tras hacer operaciones:

6m α4 + 8m

2 α2 +2m

3 = 0

Supongamos m ≠ 0, entonces simplificando la relación anterior queda:

3 α4 + 4m α2

+m2 = 0 (2)

Resolviendo (2) respecto de m obtenemos m =−

α

α

2

2

(i)

-3 (ii) Analicemos cada caso.

(i) Si m = -α2, sustituyendo en la primera ecuación y despejando n queda: n = α3

- α3 = 0 en contra

de lo supuesto. Por tanto (i) queda descartado.

(ii) Si m = -3α3, sustituyendo en la primera ecuación y despejando n queda:

n = α3 -3 α3

= -2 α3 y la primera ecuación queda:

( )( ) ( ) ( )x x x x x x x3 2 3 2 2 23 2 2 2− + = − + − = − +α α α α α α α

que, efectivamente, tiene la raíz α doble.

Page 19: Olimpiada Matematica Española 1993-2010

de n = -2 α3 obtenemos α = −

n

23 .

Entonces la segunda ecuación es de la forma

( )− + + + =2 9 15 25 03 3 2 2 3α α α αx x x ,

y, dividiendo por (x - α) resulta ( )( )− − + =2 5 03 2α α αx x ,

cuyas raíces son α y −5 α siendo doble la última.

Si m = 0, las dos ecuaciones son iguales y sus tres raíces son las mismas pero la primera no tiene

dos raíces iguales por lo que en el enunciado debería haberse añadido m ≠ 0.

6.- En la figura, AB es un segmento fijo y C un punto

variable dentro de él. Se construyen triángulos equiláteros

de lados AC y CB, ACB’ y CBA’ en el mismo semiplano

definido por AB, y otro de lado AB, ABC’ en el semiplano

opuesto. Demuestra:

a) Las rectas AA’, BB’ y CC’ son concurrentes.

b) Si llamamos P al punto común a las tres rectas del

apartado a), hallar el lugar geométrico de P cuando C varía

en el segmento AB.

c) Los centros A’’, B’’ y C’’ de los tres triángulos forman

un triángulo equilátero.

d) Los puntos A’’, B’’, C’’ y P están sobre una

circunferencia.

Solución:

a) Se traza la circunferencia circunscrita al triángulo

ABC’ y se llama P a la intersección de CC’ con ella.

Evidentemente (arco capaz) ∠APB=120º y PC’ es su

bisectriz con lo que ∠APC=∠CPB=60º y P ha de

estar en las circunferencias circunscritas a los

triángulos ACB’ y BCA’. Por tanto las tres

circunferencias se cortan en P.

Como ∠CPB’=120º y ∠CPB=60º sumando queda:

∠BPB’=180º y P está alineado con BB’.

De modo análogo se ve que P está alineado con AA’.

b) Como P está definido por la intersección de la

recta CC’ con la circunferencia circunscrita al

triángulo ABC’ el lugar pedido es el arco APB de esa

circunferencia.

c) Los lados del triángulo son perpendiculares a las

cuerdas PA , PB y PC que forman ángulos de 60º o

120º. por ello, entre sí forman ángulos iguales de 60º.

B'

A

A'P

C

C'

B

A''

B''

C''

C'

A BC

A'

B'

P

C''

A''

B''

Page 20: Olimpiada Matematica Española 1993-2010

d) Basta comprobar que los centros C’’, B’’, A’’ y P verifican el teorema de Tolomeo:

PC A B PA B C PB A C PC PA PB AB AC CB' ' ' ' ' ' ' ' ' ' ' ' ' ' ' ' ' ' ' ' ' ' ' ' = + ⇔ = + ⇔ = +

siendo la última igualdad evidente por construcción.

Page 21: Olimpiada Matematica Española 1993-2010

OLIMPIADA MATEMÁTICA ESPAÑOLA

Fase nacional 1996 (Tarragona)

Primera sesión

1.- Los números naturales a y b son tales que:

es entero. Demostrar que el máximo común divisor de a y b no es mayor que

2.- Sea G el baricentro del triángulo ABC. Si se verifica

demostrar que el triángulo es isósceles.

3.- Sean a, b ,c números reales. Se consideran las funciones

.

Sabiendo que

demostrar que si -1 < = x < = 1, entonces:

Segunda sesión

4.- Discutir la existencia de soluciones de la ecuación

según los valores del parámetro real p, y resolverla siempre que sea posible.

5.- En Port Aventura hay 16 agentes secretos. Cada uno de ellos vigila a algunos de sus colegas. Se sabe que si

el agente A vigila al agente B, entonces B no vigila a A. Además, 10 agentes cualesquiera pueden ser numerados de

forma que el primero vigila al segundo, éste vigila al tercero,....., el último (décimo) vigila al primero.

Demostrar que también se pueden numerar de este modo 11 agentes cualesquiera.

6.- La figura de la izquierda se compone de seis pentágonos regulares de lado 1m.

Se dobla por las líneas de puntos hasta que coincidan las aristas no punteadas que

confluyen en cada vértice.

¿Qué volumen de agua cabe en el recipiente formado?.

Page 22: Olimpiada Matematica Española 1993-2010
Page 23: Olimpiada Matematica Española 1993-2010

Olimpiada Matemática. Fase Nacional. Tarragona 1996

1.- Los números naturales a y b son tales que: a

b

b

a

++

+1 1 es entero. Demostrar que el máximo

común divisor de a y b no es mayor que a b+

Solución:

Se tiene:

a

b

b

a

a b a b

ab

++

+=

+ + +1 1 2 2

.

Sea d = m.c.d (a,b). Como ab es divisible por d2, entonces a b ab2 2+ + es divisible por d

2 y

también lo son a2 + b

2 y a + b, y al ser a y b naturales, se tiene :

a b d a b d+ ≥ ⇔ + ≥2

2.- Sea G el baricentro del triángulo ABC. Si se verifica:

AB + GC = AC + GB

demostrar que el triángulo es isósceles.

Solución:

Primera solución.

Teniendo en cuenta el teorema de la mediana, la relación del enunciado se escribe:

c ba c b a b c

− =+

− −+

2

3 2 4 2 4

2 2 2 2 2 2

,

multiplicando y dividiendo por la expresión conjugada queda:

( )( )c b

c b

m mc b m m

c b

c b

c b− =

+⇔ − + −

+

=

2

3

3

4

20

2 2

Probaremos que el segundo factor es positivo, de donde se deduce la conclusión.

Llamando B’ y C’ a los puntos medios de AC y Ab respectivamente, en los triángulos CC’A y

BB’A tenemos por la desigualdad triangular:

mb

c mc

bb c+ > + >2 2

; .

Sumando ambas desigualdades se obtiene el resultado.

Segunda solución.

Page 24: Olimpiada Matematica Española 1993-2010

Llamando A’, B’, C’ a los puntos medios de los lados

BC, AC y AB respectivamente y dividiendo por dos la

condición del enunciado podemos escribirla como:

C' A C' G B A B G

2 2 2 2+ = +

' ',

es decir los puntos C’ y B’ están en una elipse de focos

A y G.

Llamando M al punto medio de C’B’ , M esta en la

mediana AA’ y no es el centro de la elipse (punto medio del segmento AG), por tanto C’B’ ha de

ser perpendicular a AA’, y entonces AA’ además de mediana es altura y el triángulo es isósceles.

3.- Sean a, b, c números reales. Se consideran las funciones:

f x ax bx c g x cx bx a( ) , ( )= + + = + +2 2 .

Sabiendo que

f f f( ) , ( ) , ( ) ,− ≤ ≤ ≤1 1 0 1 1 1

demostrar que si -1≤ x ≤ 1, entonces:

f x y g x( ) ( ) .≤ ≤5

42

Solución:

Podemos conseguir coeficientes A, B, c tales que se tenga idénticamente:

f x Ax x Bx x C(x( ) ( ) ( ) )= + + − + −1 1 12 .

Particularizando para x = 1, -1 ,0 y resolviendo el sistema queda:

f xf

x xf

x x f x x R( )( )

( )( )

( ) ( )( )= + +−

− + − ∀ ∈1

21

1

21 0 1 2

De aquí se deduce:

f x x x x x x( ) ( ) ( )≤ + + − + −1

21

1

21 1 2 ;

como − ≤ ≤ + ≥ − ≥ − ≥1 1 1 0 1 0 1 02x x x y x, , , resulta

( ) ( )f xx

xx

x x x x x( ) ≤ + + − + − = − + + = − −

21

21 1 1

5

4

1

2

5

4

2 2

2

,

por otra parte, para x ≠ 0, g x x fx

( ) =2

1. Entonces

( ) ( ) ( )g xf

xf

x f x( )( ) ( )

( )= + +−

− + −1

21

1

21 0 12

válido para − ≤ ≤1 1x . Así pues

g xx x

x x( ) ≤+

+−

+ − = − ≤1

2

1

21 2 22 2

A

B

C

G

C'B'

A'

M

Page 25: Olimpiada Matematica Española 1993-2010

4.- Discutir la existencia de soluciones de la ecuación

x p x x2 22 1− + − =

según los valores del parámetro real p, y resolverla siempre que sea posible.

Solución:

Si p < 0, entonces x p x2 − > ; como 2 1 02

x − > , no existe solución. Por tanto p ≥ 0.

Aislando un radical y elevando al cuadrado dos veces se llega a la ecuación:

( ) ( )( )

8 2 44

8 2

2 2

− − =−

−p x p de donde x

p

p, .

Como x ∈ R, p < 2, así que

( )x

p

p=

4

8 2.

Sustituyendo en la ecuación dada se obtiene

( )( ) ( ) ( )

4 3

8 22

8 2

4

8 2

22−

−+

−=

p

p

p

p

p

p

como p > 0, |p| = p; y finalmente:

4 3 2 4 4 3 4 3 4 3 0 04

3− + = − ⇒ − = − ⇒ − ≥ ⇔ ≤ ≤p p p p p p p

5.- En Port Aventura hay 16 agentes secretos. Cada uno de ellos vigila a algunos de sus colegas. Se

sabe que si el agente A vigila al agente B, entonces B no vigila a A. Además, 10 agentes

cualesquiera pueden ser numerados de forma que el primero vigila al segundo, éste vigila al

tercero,....., el último (décimo) vigila al primero.

Demostrar que también se pueden numerar de este modo 11 agentes cualesquiera.

Solución:

Diremos que los agentes A y B son neutrales si A no vigila a B ni B vigila a A.

Sean A1, A2,........An los agentes. Sean:

ai el número de agentes que vigilan a Ai.

bi el número de agentes que son vigilados por Ai.

ci el número de agentes que son neutrales con Ai.

Es claro que

a b c a c b c ii i i i i i i+ + = + ≤ + ≤ ∀ =15 8 8 1 2 16, , , , ....

Page 26: Olimpiada Matematica Española 1993-2010

Notemos que si una cualquiera de las dos últimas desigualdades no se verificase, entonces no se

podrían numerar 10 espías en la forma indicada.

Combinando las relaciones anteriores obtenemos ci ≤ 1. Por tanto para cualquier espía el número de

sus colegas neutrales es 0 ó 1.

Razonemos por reducción al absurdo.

Supongamos que hubiera un grupo de 11 espías que NO se pudiera numerar en la forma descrita.

Sea B uno cualquiera de los espías de este grupo.

Numeramos los otros 10 espías como C1, C2, ....C10 de modo que C1 vigila C2....., C10 vigila a C1.

Supongamos que ninguno de los Ci sea neutral respecto de B. Entonces si C1 vigila a B, B no puede

vigilar a C2, pues en tal caso C1, B, C2, ....C10 formaría un grupo en las condiciones del problema,

luego C2 vigila B , etc . De este modo llegamos a la contradicción de que todos los espías del grupo

vigilan a B. Por tanto cada uno de los 11 espías debe tener uno y solo uno del grupo neutral con él,

lo cual es imposible.

6.- La figura de la izquierda se compone de seis pentágonos regulares de

lado 1m. Se dobla por las líneas de puntos hasta que coincidan las aristas

no punteadas que confluyen en cada vértice.

¿Qué volumen de agua cabe en el recipiente formado?.

Solución:

La figura formada por el agua es un tronco de pirámide pentagonal cuya base menor es el pentágono

dado y cuya base mayor es otro pentágono regular que tiene por lado la diagonal del anterior

paralela a la arista de la base como se muestra en la figura inferior derecha.

Más abajo, se ha dibujado en forma invertida para una mejor

comprensión del dibujo. (Figura central).

Establezcamos primero algunas relaciones conocidas para un

pentágono regular de lado 1. (Figura de la izquierda).

Llamemos d a la diagonal. Por semejanza de los triángulos ABE y

PCD tenemos:

( )1

1 11 0

1 5

212

d

dd d d

−= ⇔ − − = ⇒ =

+= ϕ

R

r

h

H

R-r

1H-h

A

B C

B

A

C

36º

1

O

P

A

B

C

D

E

72º

Page 27: Olimpiada Matematica Española 1993-2010

ϕ es el llamado número áureo y representa la relación entre la diagonal y el lado de un pentágono

regular. En nuestro caso es la relación de semejanza entre las bases del tronco de pirámide.

Además : cos º362 2

1 5

4= = =

+d ϕ y para el radio r : ( )sen º

sen º36

1

2

1

2 36

1

4

22

= ⇔ = =

−rr

ϕ

.

Llamando V al volumen de la pirámide grande , v al de la pequeña, sabemos que V = ϕ3v ; y para el

volumen del tronco de cono Vt queda:

V V v v v v aht = − = − = − = −ϕ ϕ ϕ3 3 311

31( ) ( ); siendo a el área del pentágono de lado 1. Sólo nos

queda calcular a, h, sustituir y operar:

El área a la calculamos sumado 5 triángulos isósceles de lados iguales r, r formando 72º

a r r r r= = = =5

272

5

22 36 36

5

236

5

4

2 2sen º sen º cos º cos º ϕ . (hemos usado 2rsen36º = 1 de (2)).

Para calcular h, por la semejanza de los triángulos de la figura central, tenemos:

( )H

R

h

r

H h

R rh

r H h

R r

r R r

r

r= =

−⇒ =

−=

− −

−=

− −

−=

−−

1

1

1 1

1

11

4

1

2 2 2

2

2( )

( )

( )

( )

ϕ

ϕ

ϕ

ϕ

ϕ

ϕ

Como ϕ verifica la ecuación (1): ϕ2 = ϕ +1; tenemos para la expresión de h:

( ) ( ) ( )h =

−−

−=

− − + −

− −

=− − +

− −

=

− −

11

4

1

4 2 1

1 4

3 2 2 2

1 4

1

1 4

2

2 2 2

2 2 2

( )ϕ

ϕ

ϕ

ϕ ϕ ϕ

ϕ ϕ

ϕ ϕ

ϕ ϕ ϕ ϕ

Sustituyendo las expresiones de a y h y poniendo ϕ3

-1= (ϕ-1)(ϕ2

+ ϕ + 1); queda:

Vt =

− −

=+ +

−=

+

−=

+

1

3

5

4 4

1

1 4

5

12

1

4

5

6

1

3

5

6

2 1

32

3

2

2

2

ϕ

ϕ

ϕ

ϕ ϕ

ϕ ϕ ϕ

ϕ

ϕ ϕ

ϕ

ϕ

ϕ

( )

( )

( ) ( )

y sustituyendo el valor de ϕ de (1), queda finalmente:

Vt =+

−=

+≅

5

3

2 5

5 5

15 7 5

122,554m3

Page 28: Olimpiada Matematica Española 1993-2010

OLIMPIADA MATEMÁTICA ESPAÑOLA

XXXIII Olimpiada Matemática EspañolaFase nacional 1997 (Valencia)

Primera sesión

1.- Calcular la suma de los cuadrados de los cien primeros términos de una progresión aritmética, sabiendo que

la suma de ellos vale -1, y que la suma de los términos de lugar par vale +1.

2.- Un cuadrado de lado 5 se divide en 25 cuadrados unidad por rectas paralelas a los lados. Sea A el conjunto

de los 16 puntos interiores, que son vértices de los cuadrados unidad, pero que no están en los lados del cuadrado

inicial.

¿Cuál es el mayor número de puntos de A que es posible elegir de manera que TRES cualesquiera de ellos NO sean

vértices de un triángulo rectángulo isósceles?.

3.- Se consideran las parábolas y = x2 + px + q que cortan a los ejes de coordenadas en tres puntos distintos

por los que se traza una circunferencia. Demostrar que todas las circunferencias trazadas al variar p y q en R pasanpor un punto fijo que se determinará.

Segunda sesión

4.- Sea p un número primo. Determinar todos los enteros tales que es natural.

5.- Demostrar que en un cuadrilátero convexo de área unidad, la suma de las longitudes de todos los lados y

diagonales no es menor que .

6.- Un coche tiene que dar una vuelta a un circuito circular. En el circuito hay n depósitos con cierta cantidad

de gasolina. Entre todos los depósitos contienen la cantidad exacta que el coche necesita para dar una vuelta. El

coche comienza con el depósito vacío. Demostrar que con independencia del número, posición y cantidad de

combustible de cada depósito, siempre se puede elegir un punto de comienzo que le permita completar la vuelta.

Notas: a) El consumo es uniforme y proporcional a la distancia recorrida. b) El tamaño del depósito es suficiente

para albergar toda la gasolina necesaria para dar una vuelta.

Page 29: Olimpiada Matematica Española 1993-2010

XXXIII Olimpiada Matemática Española

Fase Nacional

Valencia, Marzo 1997

Primera Sesión

1.- Calcular la suma de los cuadrados de los cien primeros términos de una progresión aritmética,

sabiendo que la suma de ellos vale -1, y que la suma de los términos de lugar par vale +1.

2.- Un cuadrado de lado 5 se divide en 25 cuadrados unidad por rectas paralelas a los lados. Sea A

el conjunto de los 16 puntos interiores, que son vértices de los cuadrados unidad, pero que no están

en los lados del cuadrado inicial.

¿Cuál es el mayor número de puntos de A que es posible elegir de manera que TRES cualesquiera

de ellos NO sean vértices de un triángulo rectángulo isósceles?.

3.- Se consideran las parábolas y = x2 + px + q que cortan a los ejes de coordenadas en tres puntos

distintos por los que se traza una circunferencia. Demostrar que todas las circunferencias trazadas

al variar p y q en R pasan por un punto fijo que se determinará.

Segunda Sesión

4.- Sea p un número primo. Determinar todos los enteros k∈Z tales que k kp2 − es natural.

5.- Demostrar que en un cuadrilátero convexo de área unidad, la suma de las longitudes de todos

los lados y diagonales no es menor que ( )2 2 2+ .

6.- Un coche tiene que dar una vuelta a un circuito circular. En el circuito hay n depósitos con

cierta cantidad de gasolina. Entre todos los depósitos contienen la cantidad exacta que el coche

necesita para dar una vuelta. El coche comienza con el depósito vacío. Demostrar que con

independencia del número, posición y cantidad de combustible de cada depósito, siempre se puede

elegir un punto de comienzo que le permita completar la vuelta.

Notas:

a) El consumo es uniforme y proporcional a la distancia recorrida.

b) El tamaño del depósito es suficiente para albergar toda la gasolina necesaria para dar una

vuelta.

Page 30: Olimpiada Matematica Española 1993-2010

Soluciones

1.- Calcular la suma de los cuadrados de los cien primeros términos de una progresión aritmética,

sabiendo que la suma de ellos vale -1, y que la suma de los términos de lugar par vale +1.

Sea la progresión a, a + d, a + 2d, ......, a + 99d, entonces tenemos que hallar:

S = a2 + (a + d)

2 + (a + 2d)

2 +.....+ (a + 99d)

2 =100a

2 + 2ad (1 + 2 + ...+ 99) +d

2 (1

2 + 2

2 +....+ 99

2).

Para calcular a y d resolvemos el sistema: ( )

( )

a a d

a d a d

+ + = −

+ + + =

99 50 1

99 25 1que operado y resuelto sale:

a = -2,98; d = 0,06.

El resto es fácil de calcular. Los paréntesis son progresiones de primer y segundo orden.

1 + 2 + ...+ 99 = 4950; 12 + 2

2 +....+ 99

2 = 328350.

El resultado final es S = 299,98

2.- Un cuadrado de lado 5 se divide en 25 cuadrados unidad por rectas paralelas a los lados. Sea A

el conjunto de los 16 puntos interiores, que son vértices de los cuadrados unidad, pero que no están

en los lados del cuadrado inicial.

¿Cuál es el mayor número de puntos de A que es posible elegir de manera que TRES cualesquiera

de ellos NO sean vértices de un triángulo rectángulo isósceles?.

Numeremos los puntos como indica la figura

13 14 15 16

9 10 11 12

5 6 7 8

1 2 3 4

Por simple tanteo se obtiene un conjunto de seis puntos verificando la condición del enunciado, por

ejemplo {1, 2, 3, 8, 12, 16}.

Supongamos que hubiera un conjunto M de 7 puntos verificando la condición del enunciado.

Notemos que si cuatro puntos forman un cuadrado, a lo sumo figurarán dos de ellos en M. Los

puntos de los conjuntos

{1, 4, 16, 13}, {2, 8, 15, 9}, {3, 12, 14, 5}

forman cuadrados y su unión forma el “contorno exterior” de A, luego a los sumo 6 de los puntos

elegidos deben estar en M y por tanto al menos un punto de M debe ser del conjunto “interior” de

A: {6, 7, 10, 11}. Por la simetría de la figura supongamos que es el 7.

Como {7, 16, 9} y {1, 7, 14} forman triángulos rectángulo isósceles, a lo sumo 2 de los puntos del

conjunto {1, 9, 14, 16} deberán figurar en M. Además {5, 7, 13, 15} forman un cuadrado por tanto

a lo sumo podremos elegir dos números entre {5, 13, 15} , de ello se deduce en M deben figurar al

menos tres puntos de {2, 3, 4, 6, 8, 10, 11, 12} . Si descomponemos este conjunto en dos

subconjuntos “cuadrados” y disjuntos

:

{3, 6, 11, 8} y {2, 4, 10, 12}

forzosamente de uno de ellos habremos de tomar dos puntos y uno de otro.

Page 31: Olimpiada Matematica Española 1993-2010

Si tomamos dos puntos del primero las únicas posibilidades son {3, 11} y {6, 8} ambas

incompatibles con cualquier elección del punto restante en el segundo conjunto.

Si los dos puntos se eligen del segundo las ínicas maneras son {2, 12} y {4, 10}, de nuevo

incompatibles con cualquier elección del punto que falta en el primer conjunto.

En resumen el número máximo de elementos es 6.

3.- Se consideran las parábolas y = x2 + px + q que cortan a los ejes de coordenadas en tres puntos

distintos por los que se traza una circunferencia. Demostrar que todas las circunferencias trazadas

al variar p y q en R pasan por un punto fijo que se determinará.

1ª Solución (analítica)

Sean α y β las raíces . Los tres puntos que definen la circunferencia son A(α, 0); B(β, 0); C(0, q).

Verificando α + β = - p. y αβ = q. (1)

La mediatriz de AB es la recta paralela al eje OY de ecuación xp

= −2

.

Hallando la mediatriz de AC, cortando con la anterior y teniendo en cuenta (1) se obtiene para el

centro las coordenadas −+

p q

2

1

2, y para el radio

( )r

p q=

+ −2 2

1

4. La ecuación de la

circunferencia es: ( )

xp

yq p q

+

+ −

+

=

+ −

2

1

2

1

4

2 2 2 2

, que una vez operada queda:

( )x y px q y q2 2 1 0+ + − + + =

que se verifica para el punto (0, 1) con independencia de p y q como se comprueba por simple

sustitución.

Claramente el punto fijo se puede obtener a partir de tres circunferencias concretas.

2ª Solución (geométrica). Puesto que la parábola corta al eje de abscisas en dos puntos, se podrá

escribir en la forma:

y = (x - a) (x - b)

y los puntos de intersección son

A(a, 0); B(b, 0); C(0,ab)

La inversión de polo el origen que transforma A en B, transforma C en U(0, 1) , así que los cuatro

puntos A,B,C,U son concíclicos y todas las circunferencias pasan por el punto fijo U.

4.- Sea p un número primo. Determinar todos los enteros k∈Z tales que k kp2 − es natural.

Solución: Pongamos ( )k kp n k pk n kp p n

2 2 2

2 2

04

21− = ⇔ − − = ⇒ =

± +.

El radicando ha de ser cuadrado perfecto, llamésmole a. Se tiene:

p2 + 4n

2 = a

2 ⇔ p

2 = (a+2n)(a-2n).

Como p es primo y a + 2n ≥ a - 2n, sólo hay dos posibilidades:

1) a + 2n = p2 y a - 2n =1

2) a + 2n = p y a - 2n = p

Page 32: Olimpiada Matematica Española 1993-2010

En el caso 1) ap

=+2 1

2 ; n

p=

−2 1

4, lo que exige p ≠ 2 (n natural).

En el caso 2) resulta a = p ; n = 0.

Sustituyendo los valores de a en (1) y operando queda:

Si p = 2 , entonces k = 2 o k = 0

Su p ≠ 2 entonces quedan los cuatro valores:

kp

kp

k p k1

2

2

2

3 4

1

2

1

20=

+

= −

= =, , ,

5.- Demostrar que en un cuadrilátero convexo de área unidad, la suma de las longitudes de todos

los lados y diagonales no es menor que ( )2 2 2+ .

Solución. 1ª

Sea el cuadrilátero de lados a, b, c, d y diagonales p y q.

Trazando la paralelas por cada vértice a la diagonal que no pasa por él se

forma un paralelogramo de área 2 y lado p y q.

Por el teorema isoperimétrico, de todos los paralelogramos de área 2, el

cuadrado tiene perímetro mínimo que vale 4 2 , luego

( ) ( )2 4 2 2 2 1p q p q+ ≥ ⇔ + ≥

En cuanto al los lados por el mismo teorema para una cuadrado de área 1

el perímetro es 4 luego:

a + b + c + d ≥ 4 (2)

Sumando(1) y (2) se obtiene el resultado.

Solución 2ª (Sin usar la propiedad isoperimétrica).

Consiste en establecer directamente las desigualdades (1) y (2).

Si α es el ángulo que forman las diagonales, tenemos:

12 2

2= ≤ ⇔ ≥pq pq

pqsenα

pero (p + q)2 = (p - q)

2 + 4pq ≥ 4pq ≥ 8. de donde p q+ ≥ =8 2 2 (1).

Para los lados, si descomponemos el cuadrilátero en dos triángulos mediante la diagonal q,

tenemos:

12 2

≤ +ab cd

Descomponiendo ahora en dos triángulos mediante la diagonal p resulta:

12 2

≤ +bc da

y de ambas desigualdades se obtiene: ab + bc + cd + da ≥ 4.

Pero:(a + b + c + d)2 = ((a + c) - (b + d))

2 + 4 (a + c)(b + d) ≥ 4 (a + c)(b + d) ≥ 16, de donde

a + b + c + d ≥ 4 (2)

da

b

c

q

p

Page 33: Olimpiada Matematica Española 1993-2010

Basta sumar (1) y (2) para obtener lo pedido.

6.- Un coche tiene que dar una vuelta a un circuito circular. En el circuito hay n depósitos con

cierta cantidad de gasolina. Entre todos los depósitos contienen la cantidad exacta que el coche

necesita para dar una vuelta. El coche comienza con el depósito vacío. Demostrar que con

independencia del número, posición y cantidad de combustible de cada depósito, siempre se puede

elegir un punto de comienzo que le permita completar la vuelta.

Notas:

a) El consumo es uniforme y proporcional a la distancia recorrida.

b) El tamaño del depósito es suficiente para albergar toda la gasolina necesaria para dar una

vuelta.

Solución 1 (Sergi Elizalde . Concursante)

Sean c1, c2, .....,cn las cantidades de combustible en cada uno de los n depósitos y sean d1, d2,....,dn

las distancias a recorrer desde cada depósito hasta el siguiente.

Hagamos el gráfico del consumo comenzando en un punto de aprovisionamiento cualquiera.

Notemos que los tramos inclinado tienen todos la misma pendiente. Los tramos bajo el eje

representan las situaciones imposibles. La pendiente de los tramos inclinados vale : −∑∑

c

d

i

i

. La

hipótesis de que el total de combustible es la cantidad exacta para dar la vuelta se traduce en que la

gráfica comienza y termina en el eje OX.

La función resultante (trazo continuo) tiene un mínimo, en la figura el punto 3. Basta comenzar en

ese punto para asegurar que el recorrido es posible.

En efecto, gráficamente equivale a trasladar el eje OX en sentido vertical hasta el punto más bajo

con lo que aseguramos que ninguna zona queda bajo el eje.

La nueva gráfica puede trazarse a partir del punto 3 siguiendo el mismo trazado hacia la derecha y

trasladando la parte anterior (tramos 1-2 y 2-3) al punto final de la gráfica anterior (de puntos en la

figura).

Solución 2 (Mª A. López Chamorro. Miembro del Jurado).

Se numeran los depósitos de 1 a n comenzando por uno cualquiera en sentido antihorario.

Llamamos:

a1, a2, ...,an a la cantidad de gasolina de cada depósito.

b1, b2, ...,bn a la cantidad de gasolina necesaria para ir del depósito ai al siguiente.

d1 = a1 -b1 , d2 = a2 - b2 ,....., dn = an - bn

Diremos que un depósito es positivo o negativo según lo sea di .

Si di = 0 , la ubicación del depósito i no influye en la ordenación del recorrido. Por ello podemos

suponer sin pérdida de generalidad que di ≠ 0 para todo i.

Por otra parte, si hay varios depósitos consecutivos positivos o negativos, el tramo limitado por

ellos se puede considerar como un único tramo positivo o negativo. Así, el problema se reduce a

1

2

3

12

45

67

Page 34: Olimpiada Matematica Española 1993-2010

tener un número par de depósitos alternativamente positivos o negativos. Agrupando los tramos por

parejas, éstas resultarán positivas o negativas y volvemos a repetir el proceso.

Así reducimos el caso a un número de depósitos n1 < n/2.

Como n < 2k , a lo sumo en k - 1 etapas llegaremos a tener 2 depósitos, uno con más gasolina que

otro, en cuyo caso empezando por el que tenga más combustible se puede completar el circuito.

El caso de un sólo depósito es trivial. Se empieza y termina en ese único depósito.

Page 35: Olimpiada Matematica Española 1993-2010

OLIMPIADA MATEMÁTICA ESPAÑOLA

XXXIV Olimpiada Matemática EspañolaFase nacional 1998 (Tarazona)

Primera sesión

1.- Un cuadrado ABCD de centro O y lado 1, gira un ángulo a en torno a O. Hallar el

área común a ambos cuadrados.

2.- Hallar todos los números naturales de 4 cifras, escritos en base 10, que sean

iguales al cubo de la suma de sus cifras.

3.- Se considera el triángulo ABC y su circunferencia circunscrita. Si D y E son puntos sobre el lado BC tales

que AD y AE son, respectivamente, paralelas a las tangentes en C y en B a la circunferencia circunscrita,

demostrar que:

Segunda sesión

4.- Hallar las tangentes de los ángulos de un triángulo sabiendo que son números enteros positivos.

5.- Hallar todas las funciones estrictamente crecientes y tales que:

f(n + f(n)) = 2 f(n)

para n = 1, 2, 3, ...

6.- Determina los valores de n para los que es posible construir un cuadrado de n x n ensamblando piezas del

tipo:

Page 36: Olimpiada Matematica Española 1993-2010

Olimpiada Española 1998

1.- Un cuadrado ABCD de centro O y lado 1, gira un ángulo α en torno a O. Hallar el área común a

ambos cuadrados.

Solución 1.

Por la simetría bastará considerar 0 < α < 90º, ya que la función es

periódica con periodo de un cuarto de vuelta.

El área pedida S(α) sale restando del área del cuadrado cuatro triángulos

como el PA’M.

Llamando x al cateto PA’ e y al cateto A’M, el área de cuatro triángulos

vale 2xy. Como el lado B’A’ vale 1, tenemos:

( )x y x y+ = − +1 12 2

relación que elevada al cuadrado y simplificada queda:

( )2 1 2 22 2xy x y= − +

pero x x y y x y= + = +2 2 2 2cos , senα α , y sustituyendo en (1) resulta:

( )x y x y2 2 2 21 11

1+ + + = ⇔ + =

+ +cos sen

sen cosα α

α α

sustituyendo en (2) y operando obtenemos:

2 12

1

1

1xy = −

+ +=

+ −

+ +sen cos

sen cos

sen cosα α

α α

α α.

Finalmente para el área pedida obtenemos:

( )S αα α

α α α α= −

+ −

+ +=

+ +1

1

1

2

1

sen cos

sen cos sen cos con 0 ≤ α ≤ 90º

Solución 2.

El área pedida consta de 8 triángulos como el sombreado en la figura

OPM.

Tomando como base b = MP, la altura es constante (de trazos en la

figura) y vale ½.

En el triángulo PA’M se tiene:

MA’ = b cos α, PA’ = b sen α ; pero BM = MA’ y PA = PA’, además:

BM + MP + PA = 1 ⇔ b cos α + b + b sen α = 1,

de donde

b =+ +

1

1sen cosα α

y el área pedida es:

O

D

AB

C

D'

A'

C'

B'

M P

O

D

AB

C

D'

A'

C'

B'

M P

Page 37: Olimpiada Matematica Española 1993-2010

S( )sen cos sen cos

αα α α α

=+ +

=+ +

81

2

1

2

1

1

2

1 con 0 ≤ α ≤ 90º

2.- Hallar todos los números naturales de 4 cifras, escritos en base 10, que sean iguales al cubo de la

suma de sus cifras.

Solución:

Sea n un número verificando el enunciado, y s la suma de sus cifras.

Como 1000 ≤ n ≤ 9999 y n = s3 , resulta

11 ≤ s ≤ 21 (1)

Si n = xyzt, tenemos:

1000x + 100y + 10z + t = s3 (2)

x + y + z + t = s

restando queda:

999x + 99y + 9z = s3 - s (3)

cuyo segundo miembro ha de ser múltiplo de 9 (por serlo el primero) y, habida cuenta de que

s3 - s = (s - 1) s (s + 1)

y por (1), sólo hay tres valores de s3 - s que son múltiplos de 9:

16·17·18; 17·18·19 y 18·19·20

sustituimos en (3) y analizamos cada caso.

999x + 99y + 9z = 16·17·18 ⇔ 111x + 11y + z = 544

resulta inmediatamente x = 4; y = 9; z = 1, valores que llevados a (2) con s = 17 se obtiene t = 3 y

finalmente n = 4913

999x + 99y + 9z = 17·18·19 ⇔ 111x + 11y + z = 646

de donde x = 5; y = 8; z = 3, valores que llevados a (2) con s = 18 se obtiene t = 2 y finalmente

n = 5832

999x + 99y + 9z = 18·19·20 ⇔ 111x + 11y + z = 760

resulta x = 6; y = 8; z = 6, valores que llevados a (2) con s = 19 resulta una contradicción.

Resumiendo, las únicas soluciones son

Page 38: Olimpiada Matematica Española 1993-2010

4913 y 5832

Page 39: Olimpiada Matematica Española 1993-2010

3.- Se considera el triángulo ABC y su circunferencia circunscrita. Si D y E son puntos sobre el lado

BC tales que AD y AE son, respectivamente, paralelas a las tangentes en C y en B a la

circunferencia circunscrita, demostrar que:

BE

CD

AB

AC=

2

2

Solución:

Los triángulos ∆ABC y ∆ADC son semejantes pues tienen los tres

ángulos iguales ya que:

∠ADC = ∠BCM = ∠BAC (la primera igualdad por ser AC y CM

paralelas y la segunda por ser ∠BCM ángulo semiinscrito) y el ángulo

∠ACD es común.

Estableciendo la proporcionalidad entre sus lados, resulta:

( )CD

AC

AC

BCCD BC AC= ⇔ ⋅ =

21

De modo análogo los triángulos ∆ABC y ∆ABE son semejantes pues:

∠AEB = ∠EBM = ∠BAC y el ángulo ∠ABE es común.

Estableciendo la proporcionalidad entre sus lados, resulta:

( )BE

AB

AB

BCBE BC AB= ⇔ ⋅ =

22

Dividiendo las igualdades (1) y (2) se obtiene el resultado.

B

A

CED

M

Page 40: Olimpiada Matematica Española 1993-2010

4.- Hallar las tangentes de los ángulos de un triángulo sabiendo que son números enteros positivos.

Solución.

Sean α, β, γ los tres ángulos y supongamos α ≤ β ≤ γ . Si fuera γπ

≥2

, tendría que ser απ

<4

y

entonces tg α no es entero.

Si tg α > 1, entonces α ≥ arc tg 2 > arc tg 33

, imposible porque α + β + γ = π.

Por tanto tg α = 1 y β + γ = 3

4

π, con lo que:

( )tgtg tg

tg tgβ γ

β γ

β γ+ = − =

+

−1

1

relación que operada se convierte en:

(tg β -1)(tg γ -1) =2

de donde, por ser enteros positivos, se sigue tg β = 2 y tg γ = 3.

Existe una visualización “sin palabras” de la solución: arc tg 1 + arc tg 2 + arc tg 3 = π.

α

βγ

Page 41: Olimpiada Matematica Española 1993-2010

5.- Hallar todas las funciones f N N: → estrictamente crecientes y tales que:

f(n + f(n)) = 2 f(n)

para n = 1, 2, 3, ...

Solución:

Supongamos f(1) = b. Entonces, f(1 + b) = 2b, como f es estrictamente creciente, se tiene:

b = f(1) < f(1 +1 ) < ….< f(1+b) = 2b = b + b.

y resulta que f(1), f(2),….f(1+ b) son b + 1 naturales, distintos, el primero vale b y el último 2b, por

tanto han de ser consecutivos.

resulta entonces:

f(1) = b, f(2) = 1 + b, f(3) = 2 + b,……, f(1 + b) = b + b.

En general, para n > 1, si f(n) = c , f(n + c) = 2c = c + c y resulta que:

c = f(n) < f(n + 1) <.…< f(n + c) = c + c y los números f(n), f(n + 1), ….., f(n + c) son consecutivos.

Así pues,

f(n) = n - 1 + f(1)

Page 42: Olimpiada Matematica Española 1993-2010

6.- Determina los valores de n para los que es posible construir un cuadrado de n × n ensamblando

piezas del tipo:

Solución:

Evidentemente n2 debe ser múltiplo de 4 y, por tanto n necesariamente es par.

Si n = 4k podemos dividir cualquier cuadrado n × n en k2 sub-cuadrados del tipo 4 × 4 cada uno de

los cuales lo podemos rellenar en la forma señalada en la figura de la izquierda.

Queda sólo considerar el caso n = 4k + 2. Veamos que en ese caso la

repuesta es negativa.

Supongamos que fuera posible.

Si pintamos cada cuadradito alternativamente de blanco y negro como en un

tablero de ajedrez, hay dos posibilidades para cada pieza:

Sea a el número de piezas del tipo de las de la izquierda y b el número de piezas del tipo de las de la

derecha.

Tenemos:

( )( )a b

kk k k+ =

+= + = + +

4 2

42 1 4 4 1

2

2 2

luego a + b ha de ser impar.

Por otra parte, como hay tantas casillas blancas como negras, se tiene:

3a + b = 3b + a ⇔ a = b, de donde a + b = 2a ha de ser par en contradicción con lo anterior.

B

N

N N

B

B BN

Page 43: Olimpiada Matematica Española 1993-2010

OLIMPIADA MATEMÁTICA ESPAÑOLA

XXXV Olimpiada Matemática EspañolaFase nacional 1999 (Granada)

Primera sesión

1.- Las rectas t y t’, tangentes a la parábola de ecuación y = x2 en los puntos A y B, se cortan en el punto C.

La mediana del triángulo ABC correspondiente al vértice C tiene longitud m.

Determinar el área del triángulo ABC en función de m.

2.- Probar que existe una sucesión de enteros positivos a1, a2,…, an, … tal que

a12 + a2

2 +…….+ an

2

es un cuadrado perfecto para todo entero positivo n.

3.- Sobre un tablero en forma de triángulo equilátero como se indica en la figura; se juega un solitario.

Sobre cada casilla se coloca una ficha. Cada ficha es blanca por un lado, y negra por el otro. Inicialmente, sólo una

ficha, que está situada en un vértice, tiene la cara negra hacia arriba; el resto de las fichas tiene la cara blanca hacia

arriba. En cada movimiento se retira sólo una ficha negra del tablero y se da la vuelta a cada una de las fichas que

ocupan una casilla vecina. Casillas vecinas son las que están unidas por un segmento.

Después de varios movimientos ¿será posible quitar todas las fichas del tablero?

Segunda sesión

4.-. Una caja contiene 900 tarjetas, numeradas del 100 al 999. Se sacan al azar (sin reposición) tarjetas de la

caja y se anota la suma de los dígitos de cada tarjeta extraída. ¿Cuál es la menor cantidad de tarjetas que se deben

sacar, para garantizar que al menos tres de esas sumas sean iguales?

5.- El baricentro del triángulo ABC es G. Denotamos por las distancias desde G a los lados a, b y

c respectivamente.

Sea el radio de la circunferencia inscrita. Probar que

:i)

ii)

Page 44: Olimpiada Matematica Española 1993-2010

6.- Se divide el plano en un número finito de regiones N mediante tres familias de rectas paralelas. No hay tres

rectas que pasen por un mismo punto.

¿Cuál es el mínimo número de rectas necesarias para que N>1999?

Page 45: Olimpiada Matematica Española 1993-2010

XXXV OLIMPÍADA MATEMÁTICA ESPAÑOLA

FASE NACIONAL

Primera Sesión Granada, 12 de Marzo de 1999

Problema 1.

Las rectas t y t’, tangentes a la parábola de ecuación y = x2 en

los puntos A y B, se cortan en el punto C.

La mediana del triángulo ∆ ABC correspondiente al vértice

C tiene longitud m.

Determinar el área del triángulo ∆ ABC en función de m.

Solución:

Sean A(a, a2) ; B(b, b

2) . Las ecuaciones de t y t' son:

t: y = 2ax - a2 , t': y = 2bx - b

2

y su intersección C es:

+ab,

baC

2.

La mediana CM está en la recta: 2

bax

+= , paralela al eje

OY. Las coordenadas de M son:

++

22

22ba

,ba

.

Tenemos: ( )

2

2ba

CMm−

== y si h es la altura del triángulo ∆ BMC resulta: 22

mabh =

−=

Poniendo [ ]XYZ para denotar el área del triángulo de vértices X,Y,Z queda finalmente:

[ ] [ ]222

122

3mm

mBMCABC ===

Problema 2.

Probar que existe una sucesión de enteros positivos a1, a2,…, an, … tal que

a12 + a2

2 +…….+ an2

es un cuadrado perfecto para todo entero positivo n.

Solución:

Lo haremos por inducción sobre n, para n = 2 basta tomar a1 = 3; a2 = 4 con 32 + 4

2 = 5

2.

Supongamos que a12 + a2

2 +…….+ an2 = k

2 . Veamos que podemos encontrar un entero positivo an+1

tal que 22

1

2pak n =+

+ .

En efecto, ( )( )11

2

1

22

−++−+=−= nnn apapapk .

Pongamos 11 ++−=+= nn apb;apa .

X

Y

A

B

C

M

t

t'

Page 46: Olimpiada Matematica Española 1993-2010

Tenemos: 2

;2

1

baa

bap n

−=

+=

+;

22

2 babak

−⋅

+= .

La última expresión exige que a y b son de la misma paridad. Distinguiremos dos casos

1.- a y b son pares, entonces k2 = 4m . Tomado a = 2m; b = 2 queda:

14

114

12

1

2

−=−=+=+= +

kma;

kmp n

2.- a y b son impares, entonces k2 = 2m + 1. Tomando a = 2m +1, b = 1 queda:

2

11

2

11

2

1

2−

==+−

=+=+

kma;

kmp n

En ambos casos hemos encontrado an+1 entero verificando el enunciado.

Segunda Sesión Granada, 13 de Marzo de 1999

Problema 3.

Sobre un tablero en forma de triángulo equilátero como se indica

en la figura; se juega un solitario.

Sobre cada casilla se coloca una ficha. Cada ficha es blanca por un

lado, y negra por el otro. Inicialmente, sólo una ficha, que está

situada en un vértice, tiene la cara negra hacia arriba; el resto de

las fichas tiene la cara blanca hacia arriba. En cada movimiento se

retira sólo una ficha negra del tablero y se da la vuelta a cada una

de las fichas que ocupan una casilla vecina. Casillas vecinas son

las que están unidas por un segmento.

Después de varios movimientos ¿será posible quitar todas las fichas del tablero?

Solución:

En el tablero, hay casillas de tres tipos : vértice, lado, o interiores. Cada una de ellas tiene,

respectivamente, dos, cuatro o seis casillas vecinas.

Si pudiéramos retirar todas las fichas del tablero, habría un momento en que quedaría sobre él una

única ficha negra. Esa ficha era inicialmente blanca, luego ha tenido que cambiar de color un

número impar de veces. Pero esto es imposible, porque una ficha se vuelve cada vez que se retira

una ficha vecina, y ninguna ficha tiene un número impar de casillas vecinas.

Problema 4.

Una caja contiene 900 tarjetas, numeradas del 100 al 999. Se sacan al azar (sin reposición) tarjetas

de la caja y se anota la suma de los dígitos de cada tarjeta extraída. ¿Cuál es la menor cantidad de

tarjetas que se deben sacar, para garantizar que al menos tres de esas sumas sean iguales?

Solución:

Hay 27 posibles resultados para la suma de dígitos (de 1 a 27). Las sumas 1 y 27 sólo se puede

obtener de un modo (100 y 999) En el caso más desfavorable al sacar 52 (27 + 25) tarjetas todas

repetirán suma dos veces y en la siguiente (extracción 53) una de ellas aparecerá por tercera vez.

Por tanto el número pedido es 27 + 25 + 1 = 53.

Page 47: Olimpiada Matematica Española 1993-2010

Problema 5.

El baricentro del triángulo ∆ ABC es G. Denotamos por cb g ,g ,ag las distancias desde G a los

lados a, b y c respectivamente.

Sea r el radio de la circunferencia inscrita. Probar que:

i) 3

2g ,

3

2g ,

3

2cb

rrrga ≥≥≥

ii) 3≥++

r

ggg cba

Solución 1 (del autor del problema):

i) Es sabido que uniendo G con cada vértice, se

forman tres triángulos BGC de base a y altura

ga , AGC de base b y altura gb y AGB de base c

y altura gc de la misma área.

Por tanto, llamando S al área de ABC:

a·ga = b·gb = c·gc = 2

3

S (1)

Por otra parte sabemos que r·(a + b + c) = 2S

(basta unir el incentro con los tres vértices y

quedan tres triángulos de bases a, b, c y altura común r).

Sustituyendo 2S en (1), y despejando queda:

gr a b c

ag

r a b c

bg

r a b c

ca b c=

+ +=

+ +=

+ +

3 3 3; ; (2)

y por la desigualdad triangular (b + c ≥ a) , resulta :a b c

a

b c

a

+ += +

+≥1 2 , de donde g

ra ≥

2

3 y

de modo análogo para gb y gc .

b) De (2) , haciendo los inversos y sumando resulta:

( ) ( ) ( )

1 1 1 3 3 3 3

g g g

a

r a b c

b

r a b c

c

r a b c ra b c

+ + =+ +

++ +

++ +

=

finalmente, aplicando la desigualdad entre las medias aritmética y armónica:

g g g

g g g r

rg g g

r

a b c

a b c

a b c+ +≥

+ +

= = ⇔+ +

≥3

3

1 1 1

3

33

Nota.- Sumando las tres desigualdades de a) sólo obtenemos g g g

r

a b c+ +≥ 2

Solución 2 (de Ramón José que mereció mención especial)

i) Consideremos los puntos MA, HA, GA como indica la figura.

Pondremos hA a la altura correspondiente a A, p el semiperímetro y S el área de ABC.

gb

gc

ga

A

BC

G

a

bc

Page 48: Olimpiada Matematica Española 1993-2010

Los triángulos AMAHA y GMAGA son semejantes

siendo la razón de semejanza 3 (propiedad del

baricentro sobre cada mediana).

Entonces

hA = 3 gA (1)

Por la desigualdad triangular:

122 ≤⇔≥⇔≥⇔≥+p

aapapacb

multiplicando por hA y teniendo en cuenta (1)

queda:

p

Sg

p

ahg A

A

A3

2

3≥⇔≥

finalmente, como S = pr resulta rg A3

2≥ .

Análogamente obtendríamos las correspondientes desigualdades para gB y gC .

ii) Usaremos la desigualdad 21

≥+x

x que se deduce de la obvia ( ) 012

≥−x . (Consideraremos

siempre x positivo).

Tenemos entonces:

6≥

++

++

+

c

a

a

c

b

c

c

b

a

b

b

a

Sumando 3, ordenando y operando resulta:

9111111111

9111 ≥

+++

+++

++⇔≥++++++++

cbac

cbab

cbaa

b

c

a

c

c

b

a

b

c

a

b

a

sacando factor común, dividiendo por 3 y poniendo 2p = a + b + c, queda:

33

2

3

2

3

2≥++

c

p

b

p

a

p (2)

Por otra parte, como CcBbAa hg;hg;hg === 333 , resulta cgbgagS cba 3332 ===

Despejando 3a, 3b y 3c y sustituyendo en (2), queda:

( ) 3≥++S

pggg cba

Finalmente usando de nuevo S = pr, resulta 3≥++

r

ggg cba

Problema 6.

Se divide el plano en un número finito de regiones N mediante tres familias de rectas paralelas. No

hay tres rectas que pasen por un mismo punto.

¿Cuál es el mínimo número de rectas necesarias para que N>1999?

Solución:

gA

hA

A

B CMA

G

GA

HA

Page 49: Olimpiada Matematica Española 1993-2010

Supongamos que hay x rectas en la primera familia, y en la segunda y z en la tercera. Las x rectas de

la primera familia determinan x + 1 regiones. La primera recta de la segunda familia determina en el

plano (x +1)·2 regiones, la segunda (x +1)·3..... la y -ésima determina (x + 1)(y + 1) regiones.

La primera recta de la tercera familia es cortada por las x + y rectas existentes en x + y + 1 partes y

cada una de estas partes divide a en dos a cada región existente de modo que el número de regiones

se incrementa en x + y + 1 regiones. Cada recta de la tercera familia aumenta las regiones existentes

en la misma cantidad; luego el número total de regiones N vale:

( )( ) ( ) 11111 ++=++++++=+++++= mnyzxzxyzyxyxzyxN

con zyxn ++= y yzxzxym ++= .

Tenemos:

( ) ( ) ( )( ) 222222222

2

1zyxyxxzzyzyxm ++≤−+−+−−++= , entonces

332

22222 n

mmmzyxn ≤⇔≥−++= y 13

12

++≤++=n

nmnN .

Para n = 76, 200213

22

>++n

n . Así, si zyxn ++== 76 con x = 26, y = 25, z = 25, resulta:

m = 1925 y N = 2002.

Page 50: Olimpiada Matematica Española 1993-2010

OLIMPIADA MATEMÁTICA ESPAÑOLA

XXXVI Olimpiada Matemática EspañolaFase nacional 2000 (Palma de Mallorca)

Primera sesión

1.- Sean los polinomios:

P(x) = x4 + ax3 + bx2 + cx + 1;

Q(x) = x4 + cx

3 + bx

2 + ax + 1.

Halla las condiciones que deben cumplir los parámetros reales a, b y c (a distinto de c) para que P(x) y Q(x) tengan

dos raíces comunes y resuelve en ese caso las ecuaciones P(x) = 0; Q(x) = 0.

2.- La figura muestra un plano con calles que delimitan 12 manzanas cuadradas.

Una persona P va desde A hasta B y otra Q desde B hasta A. Ambas parten a la vez siguiendo caminos de longitud

mínima con la misma velocidad constante. En cada punto con dos posibles direcciones a tomar, ambas tienen la

misma probabilidad. Halla la probabilidad de que se crucen.

3.- Dos circunferencias secantes C1 y C2 de radios r1 y r2 se cortan en los puntos A y B.

Por B se traza una recta variable que corta de nuevo a C1 y C2 en dos puntos que llamaremos Pr y Qr

respectivamente.

Demuestra la siguiente propiedad: Existe un punto M, que depende sólo de C1 y C2, tal que la mediatriz del

segmento PrQr pasa por M.

Segunda sesión

4.- Encuentra el mayor número entero N que cumpla las siguientes condiciones :

a) tiene sus tres cifras iguales.

b) es suma de números naturales consecutivos comenzando en 1, es decir, existe un natural n tal que:

Page 51: Olimpiada Matematica Española 1993-2010

Nota: es la parte entera de x.

5.- Tomemos cuatro puntos situados en el interior o el borde de un cuadrado de lado 1. Demuestra que al

menos dos de ellos están a distancia menor o igual que 1.

6.- Demuestra que no existe ninguna función que cumpla: f(f(n)) = n + 1.

Page 52: Olimpiada Matematica Española 1993-2010

Olimpiada Española 2000

Problema 1. Sean los polinomios:

P(x) = x4 + ax

3 + bx

2 + cx + 1;

Q(x) = x4 + cx

3 + bx

2 + ax + 1.

Halla las condiciones que deben cumplir los parámetros reales a, b y c (a ≠ c) para que P(x) y Q(x)

tengan dos raíces comunes y resuelve en ese caso las ecuaciones P(x) = 0; Q(x) = 0.

Solución de Virginia García Madurga de Zaragoza.

Las raíces comunes a ambos polinomios serán raíces de la diferencia:

P(x) - Q(x) = (a -c) x3 + (c - a) x

Resolvemos la ecuación P(x) - Q(x) = 0, sacando primero x factor común:

( ) ( )[ ] 0acxcax 2 =−+−

Las tres raíces son: 0, 1 y -1, entre ellas tienen que estar las raíces comunes

Como 0 no es raíz ni de P(x) ni de Q(x), las dos raíces comunes tiene que ser 1 y -1.

Sustituyendo estos valores en P(x) y Q(x) obtenemos el sistema:

=−+−

=+++

0cba2

0cba2

que nos da las condiciones:

b = -2

a = -c

Los polinomios quedan en la forma:

P(x) = x4 + ax

3 - 2x

2 - ax + 1

Q(x) = x4 - ax

3 - 2x

2 + ax + 1

Para resolver las ecuaciones P(x) = 0, Q(x) = 0, separamos por Ruffini las raíces conocida 1 y -1 y

quedan las ecuaciones en la forma:

P(x) = (x + 1)(x - 1) (x2 + ax - 1) = 0

Q(x) =(x + 1)(x - 1) (x2 - ax - 1) = 0

Resolviendo las ecuaciones de segundo grado queda finalmente:

Soluciones de P(x) = 0:

x = 1; x = -1; 2

4aax;

2

4aax

22 +−−=

++−=

Soluciones de Q(x) = 0:

x = 1; x = -1; 2

4aax;

2

4aax

22 +−=

++=

Page 53: Olimpiada Matematica Española 1993-2010

Problema 2. La figura muestra un plano con calles que delimitan 12

manzanas cuadradas. Una persona P va desde A hasta B y otra

Q desde B hasta A.

Ambas parten a la vez siguiendo caminos de longitud mínima

con la misma velocidad constante.

En cada punto con dos posibles direcciones a tomar, ambas

tienen la misma probabilidad.

Halla la probabilidad de que se crucen.

Solución de Fernando Cruz Robledillo (Madrid 2).

Definamos un sistema de coordenadas con origen en A y unidad el lado de un cuadrado.

Como P y Q recorren caminos de longitud mínima, P sólo puede ir a la derecha o arriba y Q a la

izquierda o abajo.

Todos los caminos tienen longitud 7, P y Q sólo se podrán encontrar entre el 3º y el 4º movimiento,

se han marcado en rojo todas las posibles posiciones de P tras el

tercer movimiento y en verde las de Q.

Caso 1. P llega a (0, 3).

La probabilidad de que P llegue a (0, 3) es: 8

1

2

1

2

1

2

1=

Sólo se puede cruzar con Q si éste está en (1, 3) lo que sucede

también con probabilidad 8

1

2

1

2

1

2

1=

P está obligado a pasar a (1, 3) pero Q pasa a (0, 3) con

probabilidad 2

1.

La probabilidad de que se crucen entre (0, 3) y (1, 3) es: 72

1

2

1

8

1

8

1=

Caso 2. P llega a (1, 2).

La probabilidad de que P llegue a (1, 2) es 8

3

2

13

3

=

(hay tres modos de llegar (1, 2)).

Sólo se puede cruzar con Q si éste está en (1, 3) o en (2, 2). Distingamos ambos casos:

a) Q llega a (1, 3) con probabilidad 8

1, entonces se cruzarán entre (1, 2) y (1, 3) si P se mueve hacia

(1, 3) y Q hacia (1, 2) ambos movimientos con probabilidad 2

1.

La probabilidad de cruzarse es 82

3

2

1

2

1

8

1

8

3=

b) Q llega a (2, 2) con probabilidad 8

3, entonces se cruzarán entre (1, 2) y (2, 2) si P se mueve hacia

(2, 2) y Q hacia (1, 2) ambos movimientos con probabilidad 2

1.

La probabilidad de cruzarse es 82

9

2

1

2

1

8

3

8

3=

Caso 3. P llega a (2, 1).

A

B

A

B

Page 54: Olimpiada Matematica Española 1993-2010

Procediendo de modo análogo, la probabilidad de cruzarse entre los puntos (2, 1) y (2, 2) es: 82

9 y

la de cruzarse entre (2, 1) y ( 3, 1) es 82

9.

Caso 4. P llega a (3, 0). La probabilidad de cruzarse entre (3, 0) y (3, 1), es 82

3 y la de cruzarse

entre (3, 0) y (4, 0) es 72

1.

La probabilidad pedida es la suma de todos los caso, resulta:

256

37

2

1

2

3

2

9

2

9

2

9

2

3

2

17888887

=++++++

Problema 3.

Dos circunferencias secantes C1 y C2 de radios r1 y r2 se cortan en los puntos A y B.

Por B se traza una recta variable que corta de nuevo a C1 y C2 en dos puntos que llamaremos Pr y Qr

respectivamente.

Demuestra la siguiente propiedad: Existe un punto M, que depende sólo de C1 y C2, tal que la

mediatriz del segmento PrQr pasa por M.

Solución de Luis Emilio García Martínez (Valencia U. Politécnica):

Sea O el punto medio del segmento M1M2.

demostraré que todas las mediatrices de los

segmentos PrQr pasan por el simétrico de B

respecto de O.

Sean ε = ∠PrBM1; γ = ∠M1BM2, Entonces:

∠M2BQr = 180º - (γ + ε)

y como el triángulo M2BQr es isósceles,

∠BM2Qr = 180º - 2 ∠M2BQr = -180º + 2 (γ + ε)

y por tanto,

∠MM2Qr = 180º - γ + ∠BM2Qr = 180º - γ - 180º + 2 (γ + ε) = γ + 2 ε

De modo análogo, por ser el triángulo PrM1B isósceles, se tiene:

∠ PrM1B = 180º - 2ε

y

∠PrM1M = 360º - (∠PrM1B + 180º - γ) = 360º - 180º +2ε -180º + γ = 2ε + γ

Resulta que para cualquier posición de la recta variable los triángulos MM1Pr y MM2Qr son iguales

y por tanto MPr = MQr y M está en la mediatriz de PrQr.

Como M no depende de la recta variable queda probada la propiedad del enunciado.

Problema 4.

Encuentra el mayor número entero N que cumpla las siguientes condiciones :

a)

3

NE tiene sus tres cifras iguales.

C1

C2

M1

M2

A

B

M

O

Qr

Pr

ε

εγ

Page 55: Olimpiada Matematica Española 1993-2010

b)

3

NE es suma de números naturales consecutivos comenzando en 1, es decir, existe un natural n

tal que

3

NE = 1 + 2 + ....+ (n-1) + n .

Nota: E(x) es la parte entera de x.

Solución de Roberto Alonso Pérez del País Vasco.

Condición a): 9k1;Nk;k·1113

NEz ≤≤∈∀=

=

Condición b): ( )

2

z811n0z2nn

2

1nnzn...321

3

NEz 2 ++−

=⇒=−+⇒+

=⇒++++=

=

(la otra raíz es negativa).

Juntando las dos condiciones, queda:

2

k·111·811n

++−=

Como n es natural, el radicando ha ser cuadrado perfecto lo que ocurre sólo para k = 6 que

sustituido en la expresión anterior resulta n = 36.

Recuperando la condición a):

1998N20016663

N6676666·111

3

NEz >>⇒>>⇒==

=

Por tanto el mayor N que cumple a) y B es N = 2000

Problema 5.

Tomemos cuatro puntos situados en el interior o el borde de un cuadrado de lado 1. Demuestra que

al menos dos de ellos están a distancia menor o igual que 1.

Solución de Manuel Pérez Molina del Alicante.

Vamos a demostrarlo por reducción al absurdo. Supongamos que distribuimos 4 puntos en el

cuadrado de manera que cada una de las seis distancias se mayor que 1. Entonces hay dos

posibilidades:

a) Los cuatro puntos forman un cuadrilátero convexo.

b) Los cuatro puntos forman un cuadrilátero no convexo.

Veamos ambos casos:

a) sean α, β, γ, δ los ángulos del cuadrilátero convexo. Sabemos

que α + β + γ + δ = 360º. Además cualquier pareja de puntos del

interior (o frontera) del cuadrado están a una distancia d ≤ 2 ya

que el diámetro de dicho cuadrado es 2 .

De la condición α + β + γ + δ = 360º, se deduce que

necesariamente uno de los ángulos ha de ser mayor o igual que

90º, digamos por ejemplo α ≥ 90º. Tenemos (ver figura):

1PP ji > , i ≠ j

luego

P1

P2

P3

P4

α

β

γ

δ

Page 56: Olimpiada Matematica Española 1993-2010

α⋅−+= cosPPPP2PPPPPP 3221

2

32

2

21

2

31

como el cuadrilátero es convexo, 90º ≤ α ≤ 180º y por tanto cos α ≤ 0 y en consecuencia:

2PP2PPPPPP 31

2

32

2

21

2

31 >⇒>+≥

lo que es imposible.

b) Si se forma un cuadrilátero no convexo podemos elegir tres de los cuatro puntos formando un

triángulo de modo que el cuarto punto sea interior. Supongamos que el punto interior es P4.

Cada lado de dicho triángulo es menor o igual que 2 (diámetro del cuadrado) y por tanto estará contenido en un triángulo equilátero de lado

2 , y circunradio 13

2

3

2

2

32 <= . Si su centro es C, P4 estará en el

interior de uno de los tres triángulos que resultan de unir C con cada

vértice y la distancia de P4 a uno de los vértices será menor o igual que el

circunradio, es decir menor que 3

2y por tanto menor que 1.

hemos encontrado un par de puntos a distancia menor o igual que 1.

Por último si tres puntos están alineados se reduce al caso b) y si los cuatro puntos están alineados

llamando x1, x2, x3 a las distancias entre puntos consecutivos, tenemos:

2xxx 321 ≤++

y por el principio del palomar, uno de ellos, digamos x1, cumple: 13

2x1 <≤ .

Problema 6.

Demuestra que no existe ninguna función f : N → N que cumpla: f(f(n)) = n + 1.

Solución de Alberto Suárez Real de Oviedo.

Supongamos que exista ( )( ) 1nnff|NN:f +=→ .

Se tiene que f(0) = a ∈ N. Por el enunciado:

( )( ) ;10ff = ( )( ) ( ) 1af0ff ==

del mismo modo, f(1) = a + 1, f(a + 1) = 2, f(2) = a + 2,........

Supongamos que f(n - 1) = a + n - 1, entonces f( a + n -1) = a + n luego hemos probado por

inducción que

( )( ) ( ) na2nafnf +=+=

entonces,

N2

1a1nna2 ∉=⇒+=+

hemos llegado a una contradicción y la condición supuesta es falsa con lo que queda demostrado la

inexistencia de la función f.

2

C

Page 57: Olimpiada Matematica Española 1993-2010

OLIMPIADA MATEMÁTICA ESPAÑOLA

XXXVII Olimpiada Matemática EspañolaFase nacional 2001 (Murcia)

Primera sesión

1.- Probar que la gráfica del polinomio P(x) es simétrica respecto del punto A(a, b) si y sólo si existe unpolinomio Q(x) tal que:

P(x) = b + (x -a)Q((x-a)2)

2.- Sea P un punto en el interior del triángulo ABC, de modo que el triángulo ABP verifica:

AP =BP

Sobre cada uno de los otros dos lados de ABC se construyen exteriormente triángulos BQC y CRA, ambossemejantes al triángulo ABP cumpliendo:

BQ = QC y CR = RA

Probar que los puntos P, Q, C y R o están alineados o son los vértices de un paralelogramo.

3.- Se tienen cinco segmentos de longitudes a1, a2, a3, a4 y a5 tales que con tres cualesquiera de ellos esposible construir un triángulo.Demostrar que al menos uno de esos triángulos tiene todos sus ángulos agudos.

Segunda sesión

4.- Los números enteros desde 1 hasta 9 se distribuyen en las casillas de una tabla 3x3.Después se suman seis números de tres cifras: los tres que se leen en filas de izquierda a derecha y los tres que seleen en columnas de arriba abajo.¿Hay alguna distribución para la cual el valor de esa suma sea 2001?

5.- ABCD es un cuadrilátero inscrito en una circunferencia de radio 1 de modo que AB es un diámetro y elcuadrilátero admite circunferencia inscrita.Probar que: .

Page 58: Olimpiada Matematica Española 1993-2010

Probar que: .

6.- Determinar la función f : N ® N (siendo N = {1,2,3,...} el conjunto de los números naturales) que cumple,para cualesquiera s, n Î N, las siguientes condiciones:

f (1) = f (2s) = 1 y si n < 2s, entonces f (2s + n) = f (n) + 1.Calcular el valor máximo de f (n) cuando n£ 2001.Hallar el menor número natural n tal que f (n) = 2001.

Page 59: Olimpiada Matematica Española 1993-2010

1

Olimpiada Española 2001

Primera sesión

1.- Prueba que la gráfica del polinomio P es simétrica respecto del punto ),( baA sí y sólo sí

existe un polinomio Q tal que: ),)(()()( 2axQaxbxP −−+= para todo .Rx ∈

Solución:

Supongamos primero que exista el polinomio P que cumple las condiciones requeridas. Sea

hax =− ó .hax += Entonces :

+=+

−=−

)()(

)()(

2

2

hhQbhaP

hhQbhaP y ,

2

)()(b

haPhaP=

++− para todo .Rh ∈ Lo que significa

que la gráfica de P es simétrica respecto del punto ).,( baA

Sea ).()()(, hRhaPxPhax =+=+= La condición bhaPhaP

=++−

2

)()( es equivalente a

,2)()( hhRhR =+− porque ).()( hRhaP −=− Para ,...)( 10

n

n hahaahR +++= la condición

anterior se escribe de la forma: bhahaaahahaan

n

nn

n 2)1(...... 2

21010 =−+−+−++++ es

decir ,...2

20 bhahaam

m =+++ para cada .Rh ∈ nnm = par, nnm 1−= impar.

Se deduce que .,0.... 042 baaaa m =====

Por tanto ahora se tiene que ...)( 3

31 +++= hahabhR y así existe un polinomio Q tal que

),()( 2hhQbhR += para algún polinomio .Q Por último ).)(()()()( 2

axQaxbhRxP −−+==

2.- Sea P un punto, en el interior del triángulo ABC, de modo que

el triángulo ABP es isósceles. Sobre cada uno de los otros dos

lados de ABC se construyen exteriormente triángulos BCQ y CAR,

ambos semejantes al triángulo ABP.

Probar que los puntos P, Q, C y R o están alineados o son los

vértices de un paralelogramo.

Solución:

Los triángulos ABC y PBQ son semejantes pues tienen un ángulo

igual ∠ABC = ∠PBQ y los lados que lo forman proporcionales:

BQ

BP

a

c=

De modo análogo, ABC es semejante a APR , por tanto PBQ y APR son semejantes (y al ser PB =

PA son iguales).

En particular: ∠ARP = ∠ACB y ∠BQP = ∠ACB

Llamando α = ∠BAP = ∠ABP, resulta:

∠QPR = 360º − (180 – 2α) − (A + B) = 180º + 2α - (180º − ∠ACB ) = 2α + ∠ACB

∠QCR = ∠ACB + 2α

∠PRC = 180º − 2α −∠ARP = 180º - 2α − ∠ACB

∠PQC =180º − 2α −∠BQP = 180º - 2α − ∠ACB

Las cuatro igualdades establecen que los dos pares de ángulos opuestos del cuadrilátero PQCR son

iguales y es un paralelogramo.

La alineación es un caso particular y se producirá cuando ∠ACB + 2α = 180º, es decir cuando

2

º180 ACB−∠=α .

c

ab

A

B

C

P

Q

R

Page 60: Olimpiada Matematica Española 1993-2010

2

3.- Están dados 5 segmentos de longitudes a1, a2, a3, a4 y a5 tales que con tres cualesquiera de ellos

es posible construir un triángulo.

Demuestra que al menos uno de esos triángulos tiene todos sus ángulos agudos.

Solución:

Supongamos que 543210 aaaaa ≤≤≤≤< . Si ningún triángulo es acutángulo, tendríamos:

(3) a

(2) a

(1)

2

5

2

4

2

3

2

4

2

3

2

2

2

3

2

2

2

1

aa

aa

aaa

≤+

≤+

≤+

Pero (desigualdad triangular):

a5<a1 + a2, luego 21

2

2

2

1

2

5 2 aaaaa ++< (4)

Sumando las desigualdades (1),(2),(3) y (4) tenemos:

21

2

2

2

1

2

5

2

4

2

3

2

5

2

4

2

3

2

2

2

1 222 aaaaaaaaaaaa +++++<++++

es decir,

21

2

3

2

2 2 aaaa <+

Como 32 aa ≤ , resulta 21

2

3

2

2

2

2 22 aaaaa <+≤ , y por tanto a2 < a1, en contradicción con la

ordenación inicial.

Page 61: Olimpiada Matematica Española 1993-2010

3

Segunda sesión

4.- Los números enteros desde 1 hasta 9 se distribuyen en las casillas de una tabla 3x3.

Después se suman seis números de tres cifras: los tres que se leen en filas y los tres que se leen en

columnas.

¿Hay alguna distribución para la cual el valor de esa suma sea 2001?

Solución

Consideremos la distribución:

a b c

d e f

g h i

Resulta:

S = abc + def + ghi + adg + beh + cfi =

100 (a + c + f + b + a + d + g) + 10(d + e + f + b + e + h) + (g + h + i + c + f + i) =

200 a + 110b + 101c + 110d + 20e + 11f + 101g + 11h + 2i

Módulo 9 tenemos:

S = 2(a + b + c+....h + i) = 2.45 = 0

Como 2001 no es múltiplo de 9, no habrá ninguna distribución para la que la suma indicada tome el

valor 2001.

5.- ABCD es un cuadrilátero inscrito en una circunferencia de radio 1 de modo que AB es un

diámetro y el cuadrilátero admite circunferencia inscrita.

Probar que: 452 −≤CD

Solución:

Sea O el centro de la semicircunferencia.

Pongamos a = BC; b = AD; p = CD;

2α = ∠BOD; 2β = ∠AOD; 2γ = ∠COD.

La condición necesaria y suficiente para que

ABCD admita una circunferencia inscrita es:

p + 2 = a + b (1)

Como 2α + 2β + 2γ = 180º, entonces

β = 90 − ( α + β)

y además:

γα−γα=γ+α=β=γ=α= sensensenbsenpsena 2coscos2)cos(22;2;2

Vamos a expresar la condición (1) en función del ángulo α y el dato p que determina por completo

el cuadrilátero.

2

4

41cos

22 pp −=−=γ ,

de donde:

αα psenpb −−= cos4 2

sustituyendo en (1), queda:

a b

p

O B A

C

D

α β

Page 62: Olimpiada Matematica Española 1993-2010

4

α−α−+α=+ psenpsenp cos422 2

o lo que es lo mismo:

( ) 22cos4 2 +=−+− psenpp αα (2)

Por tanto, existirá circunferencia inscrita para los valores de p que hagan compatible la ecuación (2)

en la incógnita α.

Puede expresarse el seno en función del coseno y estudiar el discriminante de la ecuación de

segundo grado que se obtiene, pero es más rápido interpretar la ecuación (2) como el producto

escalar de los vectores ( )αα senu ,cosr

de módulo 1 y ( )ppv −− 2,4 2r. La condición (2) queda:

2cos += pv δr

(3)

siendo δ el ángulo formado por los vectores ur

y vr

.

Para que (3) sea compatible debe cumplirse ( )22 242 ppvp −+−=≤+r

, elevando al cuadrado

y operando queda:

0482 ≤−+ pp

Las raíces de la ecuación son 452 −±=p .

Como p es positivo la condición final es:

4520 −≤≤ p

6.- Determinar la función f : N → N (siendo N = {1,2,3,...} el conjunto de los números naturales)

que cumple, para cualesquiera s, n ∈ N, las dos siguientes condiciones:

a) f (1) = 1, f (2s) = 1.

b) Si n < 2s, entonces f (2

s + n) = f (n) + 1.

Calcular el valor máximo de f (n) cuando n ≤ 2001.

Hallar el menor número natural n tal que f (n) = 2001.

Solución

Para cada número natural n definimos f (n) como la suma de las cifras de la expresión de n escrito

en base 2. Está claro que esta función f cumple las condiciones a) y b). Además, es la única función

que las cumple, porque el valor de f (n) viene determinado por las condiciones a) y b). Probamos

esa afirmación por inducción sobre n. Si n = 1 o n = 2s, f (n) = 1. Supongamos n > 1, n ≠ 2

s y que es

conocido f (m) para todo m < n; se puede escribir n = 2s + m con m < 2

s tomando 2

s la mayor

potencia de 2 que es menor que n; entonces f (n) = f (m) + 1.

Ahora, es fácil resolver las dos cuestiones que nos plantean:

En el primer caso, se trata de ver cuántos unos puede tener como máximo un número menor o igual

que 2001 escrito en base 2. Ese número, escrito en base 2, es, obviamente, 1111111111, que

corresponde a n = 1023 = 210

- 1. Es f (n) = 10.

En el segundo caso, razonando de manera análoga, se observa que la respuesta es n = 2 2001

-1.

Page 63: Olimpiada Matematica Española 1993-2010

XXXVIII Olimpiada Matemática Española

Fase nacional 2002 (La Rioja)

Primera sesión (5 de abril)

1.- Hallar todos los polinomios P(t) de una variable, que cumplen:

P(x2 - y

2) = P(x + y)·P(x – y)

para todos los números reales x e y.

2.-En un triángulo ABC, A’ es el pie de la altura relativa al vértice A y H el ortocentro.

a)Dado un número real positivo k tal que , encontrar la relación

entre los ángulos B y C en función de k.

b) Si B y C son fijos, hallar el lugar geométrico del vértice A para cada

valor de k.

3.-La función g se define sobre los números naturales y satisface las condiciones:

·g(2) = 1

·g(2n) = g(n)

·g(2n + 1) = g(2n) + 1

Sea n un número natural tal que 1 £ n £ 2002. Calcula el valor máximo M de g(n). Calcula también cuántos

valores de n satisfacen g(n) = M.

Segunda sesión (6 de abril)

4.-Sea n un número natural y m el que resulta al escribir en orden inverso las cifras de n. Determinar, si

Page 64: Olimpiada Matematica Española 1993-2010

4.-Sea n un número natural y m el que resulta al escribir en orden inverso las cifras de n. Determinar, si

existen, los números de tres cifras que cumplen 2m + S = n, siendo S la suma de las cifras de n.

5.-Se consideran 2002 segmentos en el plano tales que la suma de sus longitudes es la unidad. Probar que

existe una recta r tal que la suma de las longitudes de las proyecciones de los 2002 segmentos dados sobre r es

menor que .

6.-En un polígono regular H de 6n + 1 lados (n entero positivo), R vértices se pintan de rojo y el resto de azul.

Demostrar que el número de triángulos isósceles que tienen sus tres vértices del mismo color no depende del modo

de distribuir los colores en los vértices de H.

Page 65: Olimpiada Matematica Española 1993-2010

Soluciones Olimpiada Española 2002

Problema 1

Hallar todos los polinomios P(t) de una variable, que cumplen:

2 2( ) ( ) · ( )P x y P x y P x y− = + −

para todos los números reales x e y.

La ecuación funcional dada )()()( 22yxPyxPyxP −+=− (*) es equivalente a la ecuación

funcional )()()( vPuPuvP = (**) con el cambio de variables yxu += y ,yxv −= para todos

., Rvu ∈

Poniendo 0== vu en (**) se obtiene ,))0(()0( 2PP = de donde 1)0( =P ó .0)0( =P Sea

,1)0( =P haciendo 0=v en (*) se deduce que )0()()0( PuPP = para todo ,Ru ∈ es decir

.1)( ≡uP Sea ahora .0)0( =P Entonces ),()( uuQuP = siendo )(uQ un polinomio de grado una

unidad inferior al grado de ).(uP Fácilmente se comprueba que )(uQ satisface la ecuación

funcional (**). Por tanto nuuP =)( con .Nn ∈

Recíprocamente se comprueba sin dificultad que 1)( ≡xP y nxxP =)( con Nn ∈ satisfacen la

ecuación funcional inicial (*).

También puede hacerse sin el cambio de variable haciendo x = y = 0 se llega a 2(0) ( (0))P P= .

Además está la solución trivial P(x) ≡ 0.

Problema 2

En un triángulo ABC, A’ es el pie de la altura relativa al vértice A y H el ortocentro.

a) Dado un número real positivo k tal que '

'

AAk

HA= , encontrar la relación entre los ángulos B y

C en función de k.

b) Si B y C son fijos, hallar el lugar geométrico del vértice A para cada valor de k.

a) Tenemos:

' cosBA c B= ; '

tg ' ctg'

HAHBA C

BA= = , ' senAA c B= .

De donde:

' sentg ·tg

' cos ctg

AA c Bk B C k

HA c B C= = ⇔ = (1)

b) Poniendo a = BC, tomando unos ejes con origen en el punto medio de BC y eje OX sobre el lado

BC, resulta , 0 ; , 02 2

a aB C

y llamando A(x, y), la condición (1) se escribe:

22 2·

4

2 2

y y ak y k x

a ax x

= ⇔ = −

− +

que, una vez operada resulta: 2 2

2 21

4 4

x y

a ka+ = (2)

A

B CA'

B'C'

H

Page 66: Olimpiada Matematica Española 1993-2010

ecuación de una elipse en la que distinguimos dos casos:

Si k < 1, elipse con eje mayor sobre OX, semidistancia focal = 12

ak− y semieje mayor =

2

a

Si k >1, elipse con eje mayor sobre OY, semidistancia focal = 12

ak − y semieje mayor =

2

an .

Problema 3 La función g se define sobre los números naturales y satisface las condiciones:

• g(2) = 1

• g(2n) = g(n)

• g(2n + 1) = g(2n) + 1

Sea n un número natural tal que 1 ≤ n ≤ 2002. Calcula el valor máximo M de g(n). Calcula también

cuántos valores de n satisfacen g(n) = M.

Para cualquier natural n, consideramos su representación binaria,

( )1

1 1 0 1 0 22 2 2k k

k k kn a a a a a a a−

−= + + + + =K K ,

donde aj = 0 o 1.

Probaremos por inducción que ( )0

k

j

j

g n a=

=∑ por inducción sobre k:

Para k = 0 es cierto: ( )( ) ( )2

1 1 1g g= = . Supuesto cierto para k, hay dos casos para k + 1:

( )( ) ( )( )

( )( ) ( )( )

1 0 12 0 2

0

1 0 12 0 20

0 2· ,

1 1 2· 1

k

k k j

j

k

k k j

j

g a a a g a a a a

g a a a g a a a a

=

=

= =

= + = +

K K

K K

donde se han aplicado las propiedades de g y la hipótesis inductiva.

Entonces g(n) es el número de unos de n escrito en base 2.

Como 211

= 2048 > 2002 > 1024 = 210

, resulta M = 10.

Hay cinco soluciones de g(n) = 10: 1023, 1535, 1791, 1919 y 1983.

B C

F

F'

A

H

A'B C

FF'

A

H

A'

Page 67: Olimpiada Matematica Española 1993-2010

Problema 4

Sea n un número natural y m el que resulta al escribir en orden inverso las cifras de n. Determinar,

si existen, los números de tres cifras que cumplen 2m + S = n, siendo S la suma de las cifras de n.

n = abc = c + 10b + 100a;

m = cba = 100c + 10b + a

2 m + S = n nos da:

200c + 20b + 2a + (a + b + c) = 100a + 10b + c,

es decir

200c + 11b – 97a = 0.

Por lo tanto, 200c – 97a es múltiplo de 11.

Módulo 11: 2(c + a) es 0, y como mcd (2,11) = 1, resulta que a + c es congruente con 0 módulo 11.

Módulo 9: 2(c + a + b) congruente con 0, y c + a + b congruente con 0.

Por la primera congruencia, c + a = 0, o bien c + a = 11.

Si c + a = 0, entonces a = c = 0 y no hay solución por ser números de tres cifras.

Si c + a = 11, entonces b = 7.

Por lo tanto, 200c – 97a es múltiplo de 7.

Trabajando módulo 7: 4c + a es congruente con 0 módulo 7, es decir;

4c + a = 0, 7, 14, 21, 28, 35, 42.

Como a + c = 11, tenemos que 3c debe tomar uno de los valores -11, -4, 3, 10, 17, 24, o 31 y ser

múltiplo de 3. Luego c = 1 o c = 8.

Si c = 1, entonces a = 10, imposible.

Si c = 8, a = 3. Pero n = 378 no es solución y no existen números con las condiciones pedidas.

Problema 5 Se consideran 2002 segmentos en el plano tales que la suma de sus longitudes es la unidad. Probar

que existe una recta r tal que la suma de las longitudes de las proyecciones de los 2002 segmentos

dados sobre r es menor que 3

2.

Cada segmento determina dos vectores de igual módulo y sentido opuesto.

Consideramos los 2 x 2002 = 4004 vectores así obtenidos y los ordenamos por sus direcciones

entre 0 y 2π respecto de un sistema de referencia ortonormal arbitrario. Construimos ahora un

polígono convexo de 4004 lados “uniendo” los vectores uno a continuación del otro, a partir de uno

cualquiera dado. Claramente el perímetro de este polígono es 2 Además es un polígono centrado y

simétrico, respecto de un punto O (la prueba de esta observación es sencilla y es necesario hacerla).

Tomamos entonces uno de los lados más próximos a O; sea d el segmento perpendicular a ese lado

y a su opuesto que pasa por el centro O. La proyección del polígono sobre la recta que contiene a

este segmento es d y por tanto la suma de las proyecciones sobre la recta anterior es también d. Por

otra parte la circunferencia de centro O y radio 2

d está totalmente contenida en el interior del

polígono y entonces su circunferencia es menor que el perímetro del polígono.

Es decir: 2<πd y .3

22<<

πd

Falta considerar el caso trivial de que todos los segmentos tengan la misma dirección en cuyo caso

ni hay polígono pero tomando la recta perpendicular a la dirección común sale d = 0.

Page 68: Olimpiada Matematica Española 1993-2010

Problema 6 En un polígono regular H de 6n + 1 lados (n entero positivo), r vértices se pintan de rojo y el resto

de azul. Demostrar que el número de triángulos isósceles que tienen sus tres vértices del mismo

color no depende del modo de distribuir los colores en los vértices de H.

Debido a que el número de lados del polígono H deja de resto uno al dividirse entre seis, cada

diagonal y cada lado del mismo pertenece sólo (exactamente) a tres triángulos isósceles distintos (la

demostración es sencilla y se debe hacer).

Denotamos por AA, AR y RR los números de segmentos que son lados y diagonales cuyos

extremos respectivamente están coloreados ambos de azul, de azul y de rojo o ambos de rojo.

Análogamente denotamos por AAA, AAR, ARR y RRR el número de triángulos isósceles cuyos

vértices son los tres azules, dos azules y uno rojo, uno azul y el otro rojo o los tres rojos y ninguno

azul, respectivamente.

Entonces ,33 AARAAAAA +×=× porque cada diagonal o lado de H pertenece a tres triángulos

isósceles y los triángulos isósceles con tres vértices azules tienen tres lados con sus dos extremos

azules. Los triángulos isósceles con dos vértices azules tienen sólo un lado con sus extremos de

color azul y los triángulos isósceles con menos de dos vértices azules no tiene ningún lado con los

extremos del mismo color azul.

Análogamente establecemos: ARRAARRA ×+×=× 223 y .33 RRRARRRR ×+=×

(se deben probar estas dos nuevas relaciones). Las tres relaciones obtenidas conducen a que:

1

2AAA RRR RR AA RA+ = + − × = ,

2

1)1(

2

1)1(

2

1ARAARR ××−−××+−××

donde A es el número de vértices azules, .16 RnA −+= Esto completa la prueba.

Se observa que el resultado es también cierto si el polígono H tiene 56 +n lados.

Page 69: Olimpiada Matematica Española 1993-2010

XXXIX Olimpiada Matemática Española

Fase nacional 2003 (Canarias)

Primera sesión (3 de marzo)

1.- Probar que para cualquier primo p distinto de 2 y 5 existe un múltiplo de p cuyas cifras son todas nueves.

Por ejemplo si p = 13, 999999 = 13·76923

2.-¿Existe algún conjunto finito de números reales M que contenga al menos dos elementos distintos y que

cumpla la propiedad de que para dos números a, b cualesquiera de M, el número 2a - b2 sea también un elemento

de M?

3.- Las alturas del triángulo ABC se cortan en el punto H. Se sabe que AB = CH. Determinar el valor del

ángulo BCA

Segunda sesión (4 de marzo)

4.-Sea x un número real tal que x3 + 2x

2 +10x = 20. Demostrar que tanto x como x

2 son irracionales.

5.- ¿Cuáles son las posibles áreas de un hexágono con todos los ángulos iguales y cuyos lados miden 1, 2, 3, 4,

5 y 6 en algún orden?

6.- Ensartamos 2n bolas blancas y 2n bolas negras formando una cadena abierta. Demuestra que, se haga en el

orden que se haga, siempre es posible cortar un segmento de cadena exactamente con n bolas blancas y n bolas

negras.

Page 70: Olimpiada Matematica Española 1993-2010

Olimpiada Española 2003

Problema 1 Probar que para cualquier primo p distinto de 2 y 5 existe un múltiplo de p cuyas cifras son todas nueves. Por ejemplo si p = 13, 999999 = 13·76923 Solución de Luis Hernández Corbato de Madrid.

Sea ai el número compuesto por i nueves 99 9i

ia =678K . Supongamos que p∃ tal que

| ip a i∀ ∈/ � para probar por contradicción el enunciado.

Considérense en dicho caso los números { }1 2, , pa a aK , en este conjunto sabemos que no hay

ningún ( )0ia p≡ (por hipótesis) . Por tanto al haber p números y sólo p – 1 restos posibles módulo

p, se sabe que existen m, n tales que ( )0m na a p− ≡ .

Suponemos sin pérdida de generalidad que m > n y:

| 99 9 99 9 99 900 0 ·10m n m n n

n

m n m np a a a

−− = − = =

678 678 678678K K K K

Como 2 y 5 | 10 2 ·5 |n n n

m np p p p a−

≠ ≠ ⇒ = ⇒/ y como am – n pertenece al conjunto escogido por

ser m – n < n y m – n ≥ 1 se ha llegado a una contradicción. Por ende:

tal que |i ip a p a∀ ∃

y el enunciado queda probado. Problema 2 ¿Existe algún conjunto finito de números reales M que contenga al menos dos elementos distintos y que cumpla la propiedad de que para dos números a, b cualesquiera de M, el número 2a - b2 sea también un elemento de M? Solución de Víctor González Alonso de Burgos. Como M es finito, necesariamente estará acotado. Pongamos [ ],M x y⊂ , con x = Mín M e y = Máx M. Supongamos x ≤ 0:

Tenemos x ≤ 0 ⇒ 2x ≤ x ⇒ 2x – k2 < x (k cualquier número de M). Esto contradice que x sea el mínimo de M. Por tanto x > 0 y 0 < x < y. En cualquier casi debe ser: (1) x ≤ 2x – y2 ≤ y y además (2) x ≤ 2y – y2 ≤ y . De (1) se desprende que : x ≤ 2x – y2 ⇒ 0 ≤ x – y2 ⇒ y2 ≤ x <y ; que sólo se cumple si y ∈ (0, 1). De (2) obtenemos que: 2y – y2 ≤ y ⇒ y – y2 ≤ 0 ⇒ y ≤ y2; y esto sólo es cierto si [1, )y ∈ + ∞ .

Como (1) y (2) deben cumplirse a la vez, no existe ningún y ∈ R que pueda ser máximo de M por lo que no estaría acotado y no sería finito. Problema 3 Las alturas del triángulo ABC se cortan en el punto H. Se sabe que AB = CH. Determinar el valor del ángulo ∠BCA. Solución de Ibón Arregui Bilbao del País Vasco. Ángulo C < 90º.

A

C BA'

C'

H

α

90 − αα

Page 71: Olimpiada Matematica Española 1993-2010

Llamaremos A’ al punto en que la altura de A corta al lado BC del triángulo ABC, y C’ al punto donde la altura de C corta al lado AB del triángulo ABC. El ángulo ∠CHA’ es igual al ángulo ∠AHC’. En el triángulo CA’H, el ángulo ∠CA’H es recto, por tanto el ángulo ∠HCA’ es 90º - α . En el triángulo AHC’ el ángulo ∠HC’A es recto, por tanto el ángulo ∠HAC’ es 90º - α . El ángulo ∠HAC’ es igual al ángulo ∠A’AB del triángulo A’AB que es rectángulo por tanto el ángulo ∠A’BA es α. De aquí concluimos que los triángulos CHA’ y A’AB son semejantes, y como CH = AB , son triángulos iguales de donde obtenemos que AA’ = CA’, por tanto el valor de tg C = 1, y C = 45º.

Ángulo C > 90º. Procediendo de modo análogo el ángulo ∠A’CH es igual al ángulo ∠C’CB . En el triángulo C’CB el ángulo ∠CA’H es recto, por tanto el ángulo ∠A’HC es 90º - α y en el triángulo CC’B el ángulo ∠CC’B es recto y por tanto ∠C’BC es 90º - α. El triángulo AA’B es rectángulo en A’ y por ello ∠BAA’ es α. Entonces los triángulos AA’B y A’CH son semejantes y tienen la hipotenusa igual, luego son iguales y deducimos AA’ = A’C , entonces la tangente de C vale –1 y C = 135º. Finalmente, si fuese C = 90º, C coincide con H y CH = 0. Como AB ≠ 0, este valor de C no es válido.

Problema 4 Sea x un número real tal que x3 + 2x

2 +10x = 20. Demostrar que tanto x como x2 son irracionales. Solución: Primero veamos que x no puede ser entero. Esto puede hacerse teniendo en cuenta que si lo fuese, sería un divisor de 20, y basta probar los 8 divisores para comprobar que ninguno verifica la ecuación. Otro modo de verlo es comprobar que f(x) = x3 + 2x

2 +10x – 20 es estrictamente creciente (su derivada es positiva para todo x) y además f(1) = 13 y f(2) = 36. Luego no hay raíces enteras. Veamos que x no puede ser racional por reducción al absurdo. Supongamos que x = p/q con q ≥ 1 y p/q irreducible. Entonces

( )3 3 2 2 2 220 10 2 20 10 2p q q p qp q q qp p= − − = − −

Si q fuera estrictamente mayor que 1, la igualdad anterior estaría en contradicción con la hipótesis de que p/q es irreducible. Por tanto q = 1, x sería entero lo que es imposible. Luego x es irracional. Para la irracionalidad de x2 basta ver que

( )2

2 22

20 210 20 2

10

xx x x x

x

−+ = − ⇒ =

+,

y si x2 fuese racional, también los sería x en contra de lo probado. Problema 5 ¿Cuáles son las posibles áreas de un hexágono con todos los ángulos iguales y cuyos lados miden 1, 2, 3, 4, 5 y 6 en algún orden? Solución:

A

CB

C'

A'

B'

H

α

α

α

90 − α

90 − α

Page 72: Olimpiada Matematica Española 1993-2010

La idea es prolongar los lados para formar un triángulo equilátero.

a + b + c + d + e + f = 21 l = a + b + c = c + d + e = e + f + a

3l =21 + a + c + e, por tanto l = 7 + (a + c + e) / 3

El valor más pequeño de a + c + e es 6 y el más grande 15 así que

9 ≤ l ≤ 12 Si a + c + e = 6, entonces son:

(a, c, e) = (1, 2, 3) y (b, c, d) = (4, 5, 6) Si a + c + e = 9 el único caso posible es:

(a ,c, e) = (1, 3, 5) y (b, c, d) = (2, 4, 6) Si a + c + e = 12 el único caso posible es (a ,c, e) = (2, 4, 6) Si a + c + e = 15 el único posible es (4, 5, 6).

Como el área del triángulo de lado l es 2 3

4l y la del hexágono es ( )2 2 2 23

4l a c eé ù- + +ê úë û, las

áreas posibles son:

Si a + c + e = 6, entonces l = 9 y el área 4

367

Si a + c + e = 9, entonces l = 10 y el área 4

365

Si a + c + e = 4, entonces l =11 y el área 4

365

Si a + c + e = 5, entonces l = 12 y el área 4

367

Problema 6 Ensartamos 2n bolas blancas y 2n bolas formando una cadena abierta. Demuestra que, se haga en el orden que se haga, siempre es posible cortar un segmento de cadena exactamente con n bolas blancas y n bolas negras. Solución de Mohammed Blanca Ruiz de Valencia. Tenemos la cadena con el total de 4n bolas, 2n blancas y 2n negras. Cogemos un grupo de un extremos con 2n bolas, este grupo tendrá x bolas negras e y bolas blancas, de forma que la diferencia es x – y = 2k para { },1 , ,0, 1,k n n n n∈ − − −K K .

Vamos moviéndonos de una en una posición hacia el extremo contrario, en cada movimiento la diferencia varía en 2 o no varía, es decir k aumente en 1, disminuye en 1 o no cambia. La diferencia varía en 2 si la bola que se deja y que se coge son de distinto color y no se mantiene si son del mismo color. La posición final, es decir en el otro extremo, tendrá las bolas al revés, x bolas blancas e y bolas negras con lo que la diferencia (blancas - negras) será ahora y – x = –2k, para el mismo k. Es decir que k pasa de una posición a su opuesta con el mismo valor absoluto. Como k sólo puede variar de 1 en 1 tiene que pasar por el cero ya que no se lo puede saltar. En el momento en que k = 0, x = y = n, c.q.d. Siempre se podrá cortar un segmento de longitud 2n con n bolas blancas y n bolas negras.

e e

a

a c

c

f

b

d

Page 73: Olimpiada Matematica Española 1993-2010

XL Olimpiada Matemática Española

Fase nacional 2004 (Ciudad Real)

Primera sesión (26 de marzo)

1.- Tenemos un conjunto de 221 números reales cuya suma es 110721. Los disponemos formando una tabla

rectangular de modo que todas las filas y la primera y última columnas son progresiones aritméticas de más de un

elemento. Probar que la suma de los elementos de las cuatro esquinas vale 2004

2.- ABCD es un cuadrilátero cualquiera, P y Q los puntos medios de las

diagonales BD y AC respectivamente. Las paralelas por P y Q a la otra diagonal

se cortan en O. Si unimos O con las cuatro puntos medios de los lados X, Y, Z y

T se forman cuatro cuadriláteros, OXBY, OYCZ, OZDT y OTAX.

Probar que los cuatro cuadriláteros tienen la misma área.

·

·

3.- Se representa por Z el conjunto de todos los enteros. Hallar todas las funciones f: Z → Z tales que para

cualesquiera x, y enteros se verifica:

f(x + f(y)) = f(x) – y

Segunda sesión (27 de marzo)

4.- ¿Existe alguna potencia de 2 que al escribirla en el sistema decimal tenga todos sus dígitos distintos de cero

y sea posible reordenar los mismos para formar con ellos otra potencia de 2?. Justificar la respuesta.

5.- Demostrar que la condición necesaria y suficiente para que, en el triángulo ABC, la mediana desde B sea

dividida en tras partes iguales por la circunferencia inscrita en el triángulo, es

Page 74: Olimpiada Matematica Española 1993-2010

6.- Colocamos, formando una circunferencia, 2004 fichas bicolores: blancas por una cara y negras por la otra.

Un movimiento consiste en elegir una ficha con la cara negra hacia arriba, y dar la vuelta a tres fichas: la elegida,

la de su derecha y la de su izquierda. Supongamos que inicialmente hay una sola ficha con la cara negra hacia

arriba. ¿Será posible, repitiendo el movimiento descrito, conseguir que todas las fichas tengan la cara blanca hacia

arriba? ¿Y si tuviéramos 2003 fichas, entre las cuales exactamente una tiene al comienzo la cara negra hacia arriba?

Page 75: Olimpiada Matematica Española 1993-2010

Olimpiada Española 2004

Problema 1

Tenemos un conjunto de 221 números reales e cuya suma es 110721. Los disponemos

formando un rectángulo de modo que todas las filas y la primera y última columna son

progresiones aritméticas de más de un elemento. Probar que la suma de los elementos de las

cuatro esquinas vale 2004

Solución

Denotaremos por j

ia al elemento de la fila i-ésima y columna j-ésima del rectángulo

Pongamos n para el número de filas, m para el de columnas y S para la suma de los n·m

elementos.

Con notación matricial queda:

1 2

1 1 1

1 2

2 2 2

1 2

m

m

m

n n n

a a a

a a aM

a a a

=

L

L

L L L L

L

Sumando por filas y llamando Sk a la suma de la fila k, resulta: 1

1 11

1

2 22

1

·2

·2

.......................

·2

m

m

m

n nn

a aS m

a aS m

a aS m

+=

+=

+=

y sumando miembro a miembro queda:

( ) ( ) ( )1 1 1 1 1

1 2 1 2 1 2 1 1

·

2 4

m m m m m

n n n n n

m n mS S S S a a a a a a a a a a = + + + = + + + + + + + = + + + K K K

1 1

1 1

4 4·1107212004

· 221

m m

n n

Sa a a a

n m+ + + = = =

Page 76: Olimpiada Matematica Española 1993-2010

Problema 2

ABCD es un cuadrilátero cualquiera, P y Q los puntos

medios de las diagonales BD y AC respectivamente. Las

paralelas por P y Q a la otro diagonal se cortan en O.

Si unimos O con las cuatro puntos medios de los lados X, Y,

Z y T se forman cuatro cuadriláteros, OXBY, OYCZ, OZDT y

OTAX.

Probar que los cuatro cuadriláteros tienen la misma área.

Solución 1 (“oficial”).

Bastará probar que el área de cada cuadrilátero es la cuarta parte del área total.

La quebrada APC divide al cuadrilátero en dos partes de igual área pues AP es la mediana de

ABD y PC lo es de CBD.

La quebrada TPZ divide al cuadrilátero APCB (sombreado) en dos partes de igual área pues

PT es mediana de APD y PZ es mediana de CPD.

Tenemos ya probado que el área del cuadrilátero TPZD es

la cuarta parte del área del cuadrilátero inicial.

Finalmente TZ es paralela a OP por serlo ambas a AC;

luego los triángulos TPZ y TOZ tienen la misma área y lo

mismo les ocurre a los cuadriláteros TPZD y TOZD.

Del mismo modo se probaría para los otros tres

cuadriláteros.

Solución 2 (de la concursante Elisa García Lorenzo)

La fórmula de la superficie del cuadrilátero

es:

· ·

2

AC BD senS

α=

Además 2

ACZT XY= = al ser ZT la paralela

media del triángulo ACD y XY la paralela

media del triángulo ABC.

Igualmente: 2

BDXT ZY= =

Para probar el enunciado bastará probar que:

B

A

D

C

Q

P

O

TX

Y Z

B

A

D

C

Q

P

O

TX

Y Z

β

β

α

α

A

B

C

D

T

Z

Y

X

Q

P

T '

Z '

O

Page 77: Olimpiada Matematica Española 1993-2010

· ·

42 2

· · 2 · ·

2

AC BD XT AOsen sen

AC BD sen XT AO sen

ACsen AOsen

AQsen AOsen

AQ AO

sen sen

α β

α β

α β

α β

β α

=

=

=

=

=

que es el teorema del seno en el triángulo AQO.

Queda probado el enunciado por extensión de la demostración a los 4 cuadriláteros pequeños

que resultan ser una cuarta parte de grande.

Solución 3 (de Marco Castrillón López).

Al ser OP paralela a AC, los triángulos OXY,

PXY tienen la misma base e igual altura y por

tanto la misma área.

De ahí que los cuadriláteros OXBY, PXBY

también tienen la misma área, pero el área de

PXBY (en amarillo en la figura) es la cuarta

parte del cuadrilátero inicial al ser semejantes

con razón 2 del grande al pequeño.

A

B

CD

T

Y

X

Z

P

Q

O

Page 78: Olimpiada Matematica Española 1993-2010

Problema 3

Se representa por Z el conjunto de todos los enteros. Hallar todas las funciones : ,f Z Z→

tales que para cualesquiera ,x y enteros se verifica:

( ( )) ( ) .f x f y f x y+ = −

Solución:

Primeramente observemos que ( ( )) ( ) .f x nf y f x ny+ = −

Para 0n = es obvio, y por inducción suponiendo que para cada entero 1n ≥

( ( 1) ( )) ( ) ( 1) ,f x n f y f x n y+ − = − −

entonces:

( ( )) ( ( 1) ( ) ( )) ( ( 1) ( ))

( ) ( 1) ( ) .

f x n f y f x n f y f y f x n f y y

f x n y y f x ny

+ = + − + = + − − =

= − − − = −

Análogamente se prueba para cada entero 1.n ≤ −

Por tanto

(1 (1) (1)) 0.f f f+ ⋅ =

Poniendo 21 (1) (1) 1 (1) 0,k f f f= + ⋅ = + > se tiene ( ) ( ( )) ( ) ,f x f x f k f x k= + = − que es una

contradicción.

Deducimos que no existen funciones que satisfagan la condición requerida.

Page 79: Olimpiada Matematica Española 1993-2010

Problema 4

¿Existe alguna potencia de 2 que al escribirla en el sistema decimal tenga todos sus dígitos

distintos de cero y sea posible reordenar los mismos para formar con ellos otra potencia de 2 ?.

Justificar la respuesta

Solución 1 (“oficial”):

Supongamos que exista tal potencia de 2, es decir, que haya dos potencias de 2 cuyas

expresiones decimales sólo difieran en el orden de colocación de los dígitos. Claramente

ninguna de las dos potencias es divisible por 3, y ambas dejan el mismo resto cuando se

dividen por 9. Esto último se debe a que el resto de un número al dividirse por 9 es

congruente, módulo 9, con la suma de sus dígitos.

Por otra parte la mayor de ambas potencias se obtiene de la menor multiplicando ésta por

2, 4 u 8 (de otra manera no tendrían ambas el mismo número de dígitos). Sin embargo al

multiplicar la menor potencia de las dos por 2,4 u 8, cambia el resto cuando se divide por 9.

Los restos de las sucesivas potencias de 2 al dividirse por 9 forman una sucesión periódica.

Efectivamente, los restos de: 2, 4,8,16,32,64,128, 256,512,1024, 2048, 4096,..., son:

2, 4,8,7,5,1, 2, 4,8,7,5,1,...

Esta sucesión tiene periodo 6, porque para todo n entero positivo

6 6 02 2 2 (2 2 ) 63 2 ,n n n n+ − = − = ⋅

y este número es divisible por 9, por lo que ambas potencias dejan el mismo resto.

No es posible por tanto, reordenar los dígitos de una potencia de 2 para obtener otra potencia

distinta de 2.

Solución 2 (del concursante Lander Ramos Garrido).

No existe ningún número que cumpla las condiciones del enunciado. En primer lugar, ambos

deben tener las mismas cifras lo que implica que el número de cifras sea el mismo, así que el

cociente entre ambos no debe ser mayor que 8, porque si fuera 16 se alteraría el número de

cifras.

Otra condición que han de cumplir es, obviamente, que la suma de sus cifras sea la misma.

Como duplicar un número implica que la nueva suma de sus cifras sea el doble de la antigua

menos 9x, donde x es el número de llevadas ya que a cada llevada restas 10 a un número y

sumas 1 al siguiente. Para que la suma fuera igual tiene que cumplirse

2 9 0y x− =

siendo y la suma de las cifras antiguas.

Entonces y debe ser múltiplo de 9 para que se cumpla la ecuación anterior ya que en caso

contrario habría “medias llevadas”, absurdo.

Si y es múltiplo de 9, según el criterio de divisibilidad, el número también debería ser

múltiplo de 9, pero como estamos tratando potencias de 2, no habrá ningún número que

cumpla esas características.

Para 4 y 8 el proceso es parecido, las fórmulas serían:

Para 4: (z son las llevadas en la segunda duplicación).

( ) ( )2 2 9 9 0 4 18 9 0 4 9 2x y x x y z x y z− − = ⇔ − − = ⇔ = +

Tampoco podría ser ya que 2y + z es un natural.

Para 8: (a son las llevadas en la tercera duplicación).

( ) ( )2 4 9 2 9 0 8 9 4 8x y z a x y z a− + − = ⇔ = + + .

Y tampoco podría ser.

Page 80: Olimpiada Matematica Española 1993-2010

Problema 5

Demostrar que la condición necesaria y suficiente para que, en el triángulo ABC, la mediana

desde B sea dividida en tras partes iguales por la circunferencia inscrita en el triángulo, es

5 10 13

a b c= =

Solución.

a) la condición es necesaria. Sea ABC un triángulo tal que la mediana BK (K punto medio de

AC) corte a la circunferencia inscrita en dos puntos, M y N, tales que

BM MN NK x= = =

Sea T el punto de tangencia del círculo inscrito con el lado BC.

Las siguientes relaciones se verifican en cualquier triángulo:

2 2 2 2

2

2 2 4

a c b BT

a c b BK

+ − =

+ − =

(La primera se deduce sin más de BT + CT = a, BT – CT = c – b; la segunda –fórmula de

Apolonio o de la mediana- se puede también obtener completando el triángulo ABC hasta

obtener un paralelogramo ABCD).

Entonces resulta

2 2 2 22 2 36a c b x+ − = (1)

La potencia del vértice B respecto del círculo inscrito se puede escribir de dos maneras:

2 ·BT BM BN= ,

con lo cual

( )2 28a c b x+ − = (2)

Como, evidentemente, en el triángulo del problema, los puntos B y K están igualmente

alejados del centro del círculo inscrito, resulta BC = KC, de donde

2b a=

Sustituyendo esta última igualdad en (1) y (2), obtenemos

( )22 2 2 218 , 8c a x c a x− = − =

y ya que 0, 0,c a x− ≠ ≠ resulta

9 13, de donde

4 5

c a c

c a a

+= =

Por lo tanto,

c

ab

BA

C

I

TK

MN

Page 81: Olimpiada Matematica Española 1993-2010

5 10 13

a b c= = .

b) la condición es suficiente.

No hay pérdida de la generalidad en suponer que a = 5, b = 10, c = 13.

Sustituyendo los valores de los lados en las fórmulas utilizadas en la parte a), resulta

26 2, 16 ·BK BT BM BN= = =

y en el inradio

6 14S

rp

= =

(calculando S por la fórmula de Herón).

El triángulo BCK es isósceles, así que la bisectriz del ángulo C es también altura. Sea

H CI BK= ∩ ; consideremos el triángulo rectángulo BIT; entonces

22 2 2 2 47

414

BI r×

= + =

por otra parte, en BIH, 2 4

7HI = , y finalmente en IHM, 2 2 2 2.HM r HI= − =

Como H es el punto medio de MN, resulta 2 2MN = , luego la mediana BK queda, en efecto,

dividida en tres partes iguales por el círculo inscrito.

c

ab

BA

C

I

TK

MN

H

Page 82: Olimpiada Matematica Española 1993-2010

Problema 6

Colocamos, formando una circunferencia, 2004 fichas bicolores: blancas por una cara y

negras por la otra. Un movimiento consiste en elegir una ficha negra, y dar la vuelta a tres

fichas: la elegida, la de su derecha y la de su izquierda. Supongamos que inicialmente hay una

sola ficha con la cara negra hacia arriba. ¿Será posible, repitiendo el movimiento descrito,

conseguir que todas las fichas tengan la cara blanca hacia arriba? ¿Y si tuviéramos 2003

fichas, entre las cuales exactamente una tiene al comienzo la cara negra hacia arriba?

Solución

Numeremos las fichas desde 1 hasta 2004: la 1 es negra y las restantes son blancas.

Cada ficha inicialmente blanca debe ser “tocada” un número par de veces, para que al final

del proceso siga teniendo la cara blanca hacia arriba. Cada movimiento posible cambia el

número de fichas negras en un número impar:

BNB pasa a NBN : el número de fichas negras aumenta en 1

NNB pasa a BBN : el número de fichas negras disminuye en 1

BNN pasa a NBB: el número de fichas negras disminuye en 1

NNN pasa a BBB: el número de fichas negras disminuye en 3

Como inicialmente hay exactamente una ficha negra, el número total de movimientos para

tener las 2004 fichas con la cara blanca hacia arriba debe se impar.

Designamos por xi el número de movimientos realizados eligiendo la ficha i (que debe ser

negra).

La ficha que ocupa el lugar i cambia de color en los movimientos en que la elegimos a ella

(xi), a la de su izquierda (xi-1) o a la de su derecha (xi+1). Por lo tanto, (xi-1 + xi + xi + 1) es el

número de veces que hemos dado la vuelta a la ficha que ocupa el lugar i (2004+1 se

identifica con 1, y 2003 +2 se identifica con 1)

El número total de movimientos será:

( ) ( )1 2 3 4 2002 2003 2004N x x x x x x x= + + + + + + +K

Como 2004 es múltiplo de 3, N es la suma del número de veces que hemos dado la vuelta a

las fichas en los lugares 2, 5 ... 3k+2, ... 2003, todas ellas blancas al principio: así que N,

suma de números pares, debería ser par: contradicción, pues N es impar. Por lo tanto, no será

posible conseguir que las 2004 fichas tengan la cara blanca hacia arriba.

Con 2003 fichas si es posible: iniciando el movimiento sobre la ficha 1, (única negra al

principio), y repitiéndolo sobre las fichas que ocupan los lugares 2 .....2001,2002 llegaríamos

a la configuración

NNN NNN .... NNN BB

Eligiendo ahora las fichas que ocupan los lugares 2, 5 ... 3k+2.... 2000 tendríamos:

BBB BBB .......... BBB BB

en la que todas las fichas tendrían la cara blanca hacia arriba.

Page 83: Olimpiada Matematica Española 1993-2010

XLI Olimpiada Matemática Española

Fase nacional 2005 (Santiago de Compostela)

Primera sesión (21 de marzo)

1.- Sean a y b enteros. Demostrar que la ecuación

admite a lo sumo una solución entera.

2.- ¿Es posible colorear los puntos del plano cartesiano Oxy de coordenadas enteras con tres colores, de tal modo que cada color

aparezca infinitas veces en infinitas rectas paralelas al eje Ox y tres puntos cualesquiera, cada uno de distinto color, no estén

alineados? Justificar la contestación.

3.- Diremos que un triángulo es multiplicativo si el producto de las longitudes de dos de sus lados es

igual a la longitud del tercer lado.

Sea ABC...XYZ un polígono regular de n lados con todos sus lados de longitud 1. Las n – 3 diagonales

que salen del vértice A dividen al triángulo ZAB en n – 2 triángulos más pequeños. Probar que cada uno

de esos triángulos es multiplicativo.

Segunda sesión (22 de marzo)

4.- Probar que para todo entero positivo n, la expresión decimal de

es periódica mixta.

5.- Sean números reales cualesquiera. Probar que:

mín

6.- En un triángulo de lados a, b, c el lado a es la media aritmética de b y c. Probar:

a) 0º ≤ A ≤ 60º.

b) La altura relativa al lado a es tres veces el inradio r.

c) La distancia del circuncentro al lado a es R – r , siendo R el circunradio.

Page 84: Olimpiada Matematica Española 1993-2010

-1-

Olimpiada Española 2005

Problema 1 Sean a y b enteros. Demostrar que la ecuación

( )( )( )3 1 0x a x b x− − − + =

admite a lo sumo una solución entera.

Solución.

Sea el entero p una raíz, entonces: ( )( )( )3 1x a x b x− − − + se anula para x = p, es decir

( )( )( )3 1p a p b p− − − = −

Distingamos varios casos

1.- ( )3 1 4p p− = ⇒ =

entonces para los otros factores tenemos dos posibilidades:

( )

( )

1 1 5

1 1 3

p a a p

p b b p

− = − ⇒ = + =

− = ⇒ = − = sustituyendo queda la ecuación

( ) ( )2 3 23 5 1 11 39 44 0x x x x x− − + = − + − =

y una vez separada la raíz 4 resulta la ecuación 2 7 11 0x x− + = que no tiene raíces enteras.

( )

( )

1 1 3

1 1 5

p a a p

p b b p

− = ⇒ = − =

− = − ⇒ = + = idéntico al anterior.

2.- ( )3 1 2p p− = − ⇒ =

entonces para los otros factores tenemos dos posibilidades:

( )

( )

1 1 1

1 1 1

p a a p

p b b p

− = ⇒ = − =

− = ⇒ = − = sustituyendo queda la ecuación

( ) ( )2 3 21 3 1 5 7 2 0x x x x x− − + = − + − =

y después de separar la raíz 2 resulta 2 3 1 0x x− + = que no tiene raíces enteras.

Finalmente,

( )

( )

1 1 3

1 1 3

p a a p

p b b p

− = − ⇒ = + =

− = − ⇒ = + = sustituyendo queda la ecuación

( )3 3 23 1 9 27 26 0x x x x− + = − + − =

y después de separar la raíz 2 resulta 2 7 13 0x x− + = que no tiene raíces.

Problema 2

¿Es posible colorear los puntos del plano cartesiano Oxy de coordenadas enteras con tres colores,

de tal modo que cada color aparezca infinitas veces en infinitas rectas paralelas al eje Ox y tres

puntos cualesquiera, cada uno de distinto color, no estén alineados? Justificar la contestación.

Solución oficial.

Probemos que tal coloración es posible. Pintemos el punto ( , )x y de rojo si x y+ es par, de blanco

si x es impar e y es par y de azul si x es par e y es impar.

Claramente se satisface la condición de que cada color aparezca infinitas veces en infinitas rectas

paralelas al eje .OX

Page 85: Olimpiada Matematica Española 1993-2010

-2-

Supongamos ahora que 1 1( , )x y sea rojo, 2 2( , )x y sea blanco y 3 3( , )x y sea azul. Entonces 2 1x x− e

2 1y y− tienen paridad opuesta y 3 2x x− e 3 2y y− son ambos impares. Por tanto:

2 1 3 2 3 2 2 1( )( ) ( )( ),y y x x y y x x− − ≠ − − con lo cual : 3 2 2 1

3 2 2 1

( ) ( ),

( ) ( )

y y y y

x x x x

− −≠

− − lo que significa que tres

puntos cualesquiera, cada uno de distinto color, no están alineados.

Solución de Miguel Teixidó Román

Consideremos una coloración de 2� d acuerdo con las siguientes reglas:

-Azul si sus dos coordenadas son pares.

-Verde si ambas coordenadas son impares.

-Naranja cuando tienen una coordenada de cada paridad.

Demostraremos que tal coloración cumple las condiciones requeridas.

Observamos que tres puntos A, B, C de 2� están alineados si y sólo si existe k tal que AB k AC=

uuur uuur.

Caso 1. Supongamos que A azul, B verde y C naranja con la primera coordenada par están

alineados, la condición AB k AC=uuur uuur

sobre la primera componente queda: ( )x x x xb a k c a− = − y

reduciéndola módulo 2 resulta: ( )1 0 0 0 1 0·k k− = − ⇔ = contradicción que prueba que no existen

tales puntos.

Caso 2. Igual que el anterior con la primera coordenada de C impar y por tanto la segunda par.

Razonando del mismo modo sobre la segunda componente tenemos:

( )y y y yb a k c a− = − y módulo 2 resulta ( )1 0 0 0 1 0·k k− = − ⇔ = y la correspondiente

contradicción.

La segunda condición también se cumple:

El azul se repite infinitamente en las rectas de la forma y k= para k par.

El verde se repite infinitamente en las rectas de la forma y k= para k impar.

El naranja se repite infinitamente en las rectas de la forma y k= para cualquier k .

Problema 3 Diremos que un triángulo es multiplicativo si el producto de las

longitudes de dos de sus lados es igual a la longitud del tercer lado.

Sea ABC...XYZ un polígono regular de n lados con todos sus lados de

longitud 1. Las n – 3 diagonales que salen del vértice A dividen al

B

A

Z

RQP

X

Y

Page 86: Olimpiada Matematica Española 1993-2010

-3-

triángulo ZAB en n – 2 triángulos más pequeños. Probar que cada uno de esos triángulos es

multiplicativo.

Solución oficial. El ángulo formado pos dos diagonales consecutivas con un extremo en A es el mismo por inscrito

en el mismo arco α.

Sean PQ y QR dos segmentos adyacentes sobre el segmento ZB, determinados por tres diagonales

consecutivas. Entonces

1 1· · ·

2 2

1 1· · ·

2 2

APQ

AQR

S h PQ AP AQ sen

S h QR AQ AR sen

α

α

= =

= =

donde h es la altura común a todos los triángulos. Por tanto

· ·AP AQ AQ AR

PQ QR= ,

es decir, la razón r del producto de dos lados a la base es la misma para cualquier par de triángulos

adyacentes, y por tanto es la misma para todos ellos. Pero el primero y el último son claramente

multiplicativo al tener dos lados iguales y el tercero igual a 1. Se deduce que r = 1 y todos los

triángulos son multiplicativos.

Solución de Anas El Barkani que mereció una mención especial del jurado. Vamos a probar el enunciado demostrando que si un triángulo

es multiplicativo, también lo es el colindante.

Obsérvese primero que el triángulo ABQ es isósceles puesto

que los ángulos A y B abarcan el mismo arco. Es obvio que el

triángulo ABQ es multiplicativo pues 1AB = .

· ·1QB AQ AB AQ AQ= = = (1)

Ahora bien, si aplicamos el teorema de la bisectriz al triángulo

APB tenemos que:

·PQ QB

AQ PQ AQ APAP AB

= = ⇒ =

lo que prueba que APQ también es multiplicativo.

c.q.d.

Problema 4 Probar que para todo entero positivo n, la expresión decimal de

1 1 1

1 2n n n+ +

+ +

es periódica mixta.

Solución: Tenemos

( )( )

21 1 1 3 6 2

1 2 1 2

n n

n n n n n n

+ ++ + =

+ + + +

Sabemos que para que una fracción origine un decimal periódico mixto, una vez reducida debe

tener en el denominador algún factor primo del conjunto { }2,5 y alguno que no sea ni 2 ni 5.

Z

A B

C

PQ

Page 87: Olimpiada Matematica Española 1993-2010

-4-

Veamos primero que la fracción anterior tiene en el denominador al menos una factor 2 más que en

el numerador, en efecto

Si n es par tenemos 2n k= que una vez sustituido resulta:

( )( ) ( )( ) ( )( )

2 2 21 1 1 3 6 2 12 12 2 6 6 1

1 2 1 2 2 2 1 2 2 2 2 1 1

n n k k k k

n n n n n n k k k k k k

+ + + + + ++ + = = =

+ + + + + + + +

el numerador es impar y el denominados par.

Si n es impar tenemos 2 1n k= + que una vez sustituido resulta:

( )( ) ( )( )( )

2 21 1 1 3 6 2 12 24 11

1 2 1 2 2 1 2 2 2 3

n n k k

n n n n n n k k k

+ + + ++ + = =

+ + + + + + +

el numerador es impar y el denominados par.

En ambos casos el denominador tiene al menos un factor 2 que no está en el numerador.

Además la expresión ( )( )

21 1 1 3 6 2

1 2 1 2

n n

n n n n n n

+ ++ + =

+ + + + muestra que el numerador no contiene el

factor primo 3 (da resto 2 al dividirlo entre tres) mientras el denominador al ser producto de tres

números consecutivos es múltiplo de tres.

Problema 5

Sean , , ,r s u v números reales cualesquiera. Probar que:

mín{ }2 2 2 2 1, , , .

4r s s u u v v r− − − − ≤

Solución.

Supongamos que los cuatro números 2 2 2, ,r s s u u v− − − y 2v r− son mayores estrictamente que

1.

4 Entonces 2 2 2 2 1 1 1 1

,4 4 4 4

r s s u u v v r− + − + − + − > + + + pero esta expresión es equivalente a

2 2 2 21 1 1 1

02 2 2 2

r s u v

> − + − + − + −

que es una contradicción.

Problema 6 En un triángulo de lados a, b, c el lado a es la media aritmética de b y c. Probar:

a) 0º ≤ A ≤ 60º.

b) La altura relativa al lado a es tres veces el inradio r.

c) La distancia del circuncentro al lado a es R – r.

Solución. a) Por la desigualdad triangular:

3 312

31 3

32 3

b c bb c b c

bc

b c b cc b c b

c

+ ≤ + ⇔ ≤ ⇔ ≤

⇒ ≤ ≤+ ≤ + ⇔ ≤ ⇔ ≥

Por el teorema de coseno:

( )2 2 2

2 2 3 3 22 cos cos

4 8

b c b c bcb c bc A A

bc

+ + −= + − ⇒ = r

da

R

a

cb

ha

BC

A

I

T

Page 88: Olimpiada Matematica Española 1993-2010

-5-

dividiendo numerador y denominador por c2 y llamando por comodidad de escritura

bx

c= , queda:

( )23 2 3 3 1 3

cos8 8 4 8

x xA f x x

x x

− += = = − + con

13

3x≤ ≤ .

Fácilmente se comprueba que ( )1

3 13

f f

= =

y que la derivada se anula en x = 1 donde hay un

mínimo que vale ( )1

12

f = .

También puede localizarse el mínimo sin recurrir a la derivada teniendo en cuenta la desigualdad de

las medias:

( )2

3 2 3 3 1 3 1 3 1 1 3cos ·2

8 8 4 8 4 8 4 8

x xA f x x x

x x x

− + = = = − + = − + + ≥ − +

con igualdad para x = 1.

Resumiendo queda 1

cos 1 60º 0º2

A A≤ ≤ ⇔ ≥ ≥ .

b) Designando A, B y C a los vértices opuestos a los lados a, b y c respectivamente, I al incentro y

ha a la altura correspondiente al lado a como se indica en la figura, S al área y p al semiperímetro,

tenemos:

3

3 12 23

1 2 2

2

a a

a

a b c arS pr r

arah r h

S ah

+ + = = =

⇒ = ⇔ ==

c) Pongamos da a la distancia entre el circuncentro y el lado a.

De una parte 2

2 2

4a

ad R= − (1)

y de otra 2

2

A r rtg

p a a= =

−.

Como 2a

RsenA

= y

12

22

Atg

senAA

tg

+

= , resulta:

2

2 222

41

2 24 4 4

r

a aaR a r Rr rr r

a

+

= = + ⇒ = −

que sustituida en (1) queda:

( )2

22 2 2 224

a a

ad R R Rr r R r d R r= − = − + = − ⇔ = −

Solución de Elisa Lorenzo García.

Los apartados a) y b) son esencialmente iguales a los de la solución oficial.

Para el apartado c) se da la solución que sigue sin utilizar trigonometría.

Q es la intersección de la bisectriz de A con la mediatriz de a que está en el punto medio del arco

BC.

Llamando x PB= , resulta

Page 89: Olimpiada Matematica Española 1993-2010

-6-

23

4

b ca x y

c bb y z x

c z x

+ = + =

−= + ⇒ =

= +

Aplicando el teorema de la bisectriz:

c b

BQ CQ=

y

2

2 2 2

b c c bcBQ CQ a BQ

b c

+ ++ = = ⇒ =

+

además '4

b cBA

+= por ser la mitad de a.

Calculemos

2 23 3' ' 2 0

4 2 2 4 4

b c c bc c b b b c c c bcQA PQ BA BQ BQ BP c c

b c b c

+ + − − + + +− = − − + = − + = − = − =

+ +

de donde 'QA PQ= y como � � ' 90ºIPQ QA S= = y � �'IQP A QS= , resulta que los triángulos PIQ y

A’SQ son iguales y 'IP A S r= = de donde queda finalmente:

' 'OA OS A S R r= − = −

c

a

b

A

B CA'

OI

P

S

Q

Page 90: Olimpiada Matematica Española 1993-2010

XLII Olimpiada Matemática Española

Fase nacional 2006 (Sevilla)

Primera sesión (24 de marzo)

1.- Sea un polinomio con coeficientes enteros. Demostrar que si existe un entero k tal que ninguno de

los enteros es divisible por k, entonces no tiene raíces enteras.

2.- Las dimensiones de un ortoedro de madera son enteras. Pintamos toda su superficie (las seis caras), lo

cortamos mediante planos paralelos a las caras en cubos de una unidad de arista y observamos que exactamente la

mitad de los cubos no tienen ninguna cara pintada. Probar que el número de ortoedros con tal propiedad es finito.

(Puede resultar útil tener en cuenta que ).

.

3.- ABC es un triángulo isósceles con AB = AC. Sea P un punto cualquiera de la circunferencia tangente a los

lados AB en B y a AC en C.

Llamamos a, b y c a las distancias desde P a los lados BC, AC y AB respectivamente. Probar que:

Segunda sesión (25 de marzo)

4.- Hallar todas las funciones que satisfacen la ecuación

para todo par de números reales e positivos, siendo un número real positivo tal que

Page 91: Olimpiada Matematica Española 1993-2010

5.- Probar que el producto de cuatro naturales consecutivos no puede ser ni cuadrado ni cubo perfecto

6.- Las diagonales y de un cuadrilátero convexo se cortan en Denotamos por y

a las áreas de los triángulos y del cuadrilátero respectivamente. Prueba que

¿Cuándo se alcanza la igualdad?

Page 92: Olimpiada Matematica Española 1993-2010

Olimpiada Española 2006

Problema 1

Sea ( )P x un polinomio con coeficientes enteros, demostrar que si existe un entero k tal que

ninguno de los enteros ( ) ( ) ( )1 , 2 , ,P P P kK es divisible por k, entonces ( )P x no tiene raíces

enteras.

Solución.

Por reducción al absurdo. Si n fuese una raíz, por una parte tenemos

( ) ( ) ( )P x x n Q x= −

y por otra siempre existen enteros q y r tales que n kq r= + , con 1 r k≤ ≤ (basta hacer la división

entera y en el caso de ser resto cero se rebaja el cociente en una unidad), entonces

( ) ( ) ( ) ( )P r r n Q r kqQ r= − = −

en contra de lo supuesto en el enunciado.

Problema 2 Las dimensiones de un paralelepípedo de madera son enteras. Pintamos toda su superficie (las seis

caras) y lo cortamos en cubos de una unidad de arista y observamos que exactamente la mitad de los

pequeños cubos no tienen ninguna cara pintada. Probar que el número de paralelepípedos con tal

propiedad es finito.

(Puede resultar útil tener en cuenta que 31

1,79... 1,82

<� ).

Solución . Consiste en acotar el número de soluciones de la ecuación diofántica:

( )( )( )2 2 2 2abc a b c= − − −

y para ello suponiendo que a b c≤ ≤ , se pone en la forma: 1 2 2 2

· ·2

a b c

a b c

− − −= , y como

3

32 1 2 1 2 1

2 2 2

a a a

a a a

− − − ≤ < ⇒ < ≤

fácilmente se obtiene que 4 < a < 10.

Fijado a se acotan de modo análogo los posibles valores de b:

De una parte

( ) ( )

22 2 2 2

·2 2 2 2

a b c b a b

a b c b a b

− − − − = ⇒ ≤ <

− −

y de ahí se sigue

( ) ( )2 5

12 2 2 2 6

a b a

a b a

−< < ≤ <

− −

con lo que para cada valor de a, b sólo puede tomar un conjunto finito de valores.

Finalmente fijados a y b a lo sumo hay un valor para c que cumpla la ecuación inicial lo que prueba

que el número de soluciones es finito.

Problema 3 ABC es un triángulo isósceles con AB = AC. Sea P un punto cualquiera de la circunferencia

tangente a los lados AB en B y a AC en C.

Pongamos a, b y c a las distancias desde P a los lados BC, AC y AB respectivamente. Probar que:

Page 93: Olimpiada Matematica Española 1993-2010

2 ·a b c=

Solución:

Pongamos m = PB; n = PC, Q, R y S las proyecciones de P sobre

cada lado y sea P’ el punto diametralmente opuesto a P .

Por la semejanza de los triángulos PBP’ y PBS se tiene:

222

m rm cr

c m= ⇔ = (1)

De modo análogo por la semejanza de PCP’ y PBC se cumple:

222

n rn br

b n= ⇔ = (2)

Por el teorema de los senos en PBC:

( )2

nsen PBC

r

=

y en el triángulo rectángulo PQB:

( )a

sen PBCm

=

de donde 2 2

2

22 4

mn m na a

r r= ⇔ = y por (1) y (2) queda finalmente:

2

2

2 ·2·

4

cr bra b c

r= =

Problema 4.

Hallar todas las funciones : (0, )f R∞ → que satisfacen la ecuación

( ) ( ) 2 ( )f x f y f f f xyx y

λ λ + =

para todo par de números reales x e y positivos, siendo λ un número real positivo tal que

( ) 1.f λ =

Solución:

Haciendo 1x y= = en la ecuación funcional dada se tiene que 2 2(1) ( ) 2 (1).f f fλ+ = De este

modo ( )2

(1) 1 0f − = y así (1) 1.f = Sustituyendo ahora 1y = en la ecuación funcional resulta

( ) (1) ( ) 2 ( ),f x f f f f xx

λλ

+ =

que es equivalente a ( )f x f

x

λ =

para todo 0.x > Tomemos a

continuación yx

λ= y observemos que

( ) ( ) 2 ( ).f x f f f x fx x

λ λλ

+ =

Entonces ( ) 1,f x fx

λ =

por lo que 2 ( ) 1f x = para todo 0.x >

m na

bc

CB

A

P

Q

RS

P'

Page 94: Olimpiada Matematica Española 1993-2010

Sustituyendo x y t= = de nuevo en la ecuación funcional inicial se tiene que

( )2 22 ( )f t f f t

t

λ + =

y debido a ser el miembro de la izquierda positivo f es positivo y

( ) 1f x = para todo 0.x > Se comprueba fácilmente que esta función constante e idénticamente

igual a 1 es solución de la ecuación funcional del enunciado y además es la única.

Problema 5 Probar que el producto de cuatro naturales consecutivos no puede ser ni cuadrado ni cubo perfecto.

Solución. Si el producto ( ) ( )( )1 1 2N n n n n= − + + fuese un cuadrado, basta ponerlo en la forma

( ) ( )( ) ( )( ) ( )2

2 2 21 1 2 2 1 1N n n n n n n n n n n= − + + = + − + = + − −

de donde se sigue una contradicción (no hay dos cuadrados consecutivos).

Si N fuese cubo perfecto, podemos suponer que 2n > (si n = 2 , N = 24).

Distinguimos ahora dos casos:

a) n impar, entonces n es primo con los otros tres factores y si N es cubo perfecto, también lo es

( )( )( ) 3 21 1 2 2 2M n n n n n n= − + + = + − −

pero si ( )33 3 22 2 2 1n n n n n n> ⇒ < + − − < + y ya tenemos la contradicción pues entre dos cubos

consecutivos no puede haber otro cubo.

b) si n es par, 1n + es impar y por tanto 1n + es primo con el producto

( ) ( ) 3 21 2 2M n n n n n n= − + = + −

que también debe ser cubo perfecto.

Finalmente, como ( )33 3 22 2 1x x x x x x> ⇒ < + − < + se sigue la contradicción.

Problema 6

Las diagonales AC y BD de un cuadrilátero convexo ABCD se cortan en .E Denotamos por

1 2,S S y S a las áreas de los triángulos ,ABE CDE y del cuadrilátero ABCD respectivamente.

Prueba que 1 2 .S S S+ ≤ ¿Cuándo se alcanza la igualdad?

Solución:

Denotando, como en el enunciado, la áreas de los triángulos BCE y DAE por 3S y 4S

respectivamente, tenemos que probar que 1 2 1 2 3 4 .S S S S S S+ ≤ + + +

Elevando al cuadrado la anterior desigualdad, se obtiene la desigualdad equivalente

1 2 3 42 S S S S≤ + (1).

Sean ahora K y L los pies de las perpendiculares en la diagonal AC trazadas desde D y

B respectivamente. Estos puntos K y L pueden estar dentro o fuera del segmento .AC Llamamos

, , , .b BL d DK m AE n CE= = = = Entonces

1 2 3 1

1 1 1 1, , , .

2 2 2 2S mb S nd S nb S md= = = =

Page 95: Olimpiada Matematica Española 1993-2010

Sustituyendo esta expresión en la desigualdad (1) se llega a 1

( ),2

mb nd nb md⋅ ≤ + que es

precisamente la desigualdad entre la media aritmética y la media geométrica de los dos productos

nb y .md

Esta última desigualdad se alcanza si y sólo si b m

nb mdd n

= ⇔ = (2).

Las rectas BL y DK son paralelas. Así ,b BL BE

d DK DE= = por la semejanza entre los triángulos BLE

y .DKE La relación (2) se convierte en BE AE

DE CE= (3). Y recíprocamente por la semejanza de

triángulos (3) se verifica si y sólo si AB y CD son paralelos, es decir el cuadrilátero dado es un

trapecio con los lados paralelos AB y CD . Esta es la condición para que se alcance (1).

Page 96: Olimpiada Matematica Española 1993-2010

XLIII Olimpiada Matemática Española

Fase nacional 2007 (Torrelodones)

Primera sesión (23 de marzo)

1.- Sean cinco números positivos en progresión aritmética de diferencia d. Probar que

2.- Determinar todos los posibles valores enteros no negativos que puede tomar la expresión

siendo y enteros no negativos tales que

3.- Sea O el circuncentro de un triángulo ABC. La bisectriz que parte de A corta al lado opuesto en P.

Probar que se cumple:

Segunda sesión (24 de marzo)

4.- ¿Cuáles son los números enteros positivos que se pueden obtener de exactamente 2007 maneras distintas,

como la suma de al menos dos números enteros positivos consecutivos? ¿Cuál es el menor de todos ellos?

Ejemplo: el número 9 se escribe exactamente de dos maneras distintas:

9 = 4 + 5

9 = 2 + 3 + 4

5.- Sea un número real positivo y un entero mayor que 1. Demostrar que

6.- Dada una semicircunferencia de diámetro AB = 2R, se considera una cuerda CD de longitud fija c. Sea E la

Page 97: Olimpiada Matematica Española 1993-2010

6.- Dada una semicircunferencia de diámetro AB = 2R, se considera una cuerda CD de longitud fija c. Sea E la

intersección de AC con BD y F la intersección de AD con BC.

Probar que el segmento EF tiene longitud constante y dirección constante al variar la cuerda CD sobre la

semicircunferencia.

Page 98: Olimpiada Matematica Española 1993-2010

1

Olimpiada española 2007

Problema 1.

Sean 0 1 2 3 4, , , ,a a a a a cinco números positivos en progresión aritmética de razón d. Probar que

( )3 3 3 3 3

2 0 1 3 4

14 4 .

10a a a a a≤ + + +

Solución 1 (del autor de la propuesta).

La desigualdad dada puede escribirse como

3 3 3 3 3

2 0 1 3 410 4 4 .a a a a a≤ + + +

y sumando 3

26a a ambos miembros se convierte en

( )3 3 3 3 3 3

2 0 1 2 3 4

14 6 4 .

16a a a a a a≤ + + + +

Por otro lado como 0 1 2 3 4, , , ,a a a a a están en progresión aritmética, entonces

( ) ( )0 1 2 3 4 0 4 1 3 2

4

2 2 2 2 2

4 4 4 4 4 4 4 4

0 1 2 3 4 0 1 2

4 4 4 4 4 4 4 42 2 2 .

0 1 2 0 1 2 3 4

a a a a a a a a a a

a a a a a

+ + + + = + + + + =

+ + = + + + + =

Aplicando la desigualdad de Jensen a la función ( ) 3f t t= , convexa en ( )0, +∞ , con

4 1,0 4

2k k

p kk

= ≤ ≤

resulta

( )4 4

0 0

k k k k

k k

f p a p f a= =

∑ ∑

o equivalentemente,

( )3 3 3 3 3 3 3 3 3 3 3

2 0 1 2 3 4 0 1 2 3 4

4 4 4 4 41 14 6 4

0 1 2 3 42 16ka a a a a a a a a a a

≤ + + + + = + + + +

.

Obsérvese que la igualdad tiene lugar cuando los cinco números son iguales y hemos terminado.

Solución 2.

Llamando a al término central y d a la diferencia, la progresión es 2 , , , , 2a d a d a a d a d− − + + y

tenemos:

( )

( )

( )

( )

33 3 2 2 3

0

33 3 2 2 3

4

33 3 2 2 3

1

33 3 2 2 3

3

2 6 12 8

2 6 12 8

4 4 4 12 12 4

4 4 4 12 12 4

a a d a a d ad d

a a d a a d ad d

a a d a a d ad d

a a d a a d ad d

= − = − + −

= + = + + +

= − = − + −

= + = + + +

sumando:

Page 99: Olimpiada Matematica Española 1993-2010

2

3 3 3 3 3 2

0 1 3 44 4 10 48a a a a a ad+ + + = +

dividiendo por 10 queda

( )3 3 3 3 3 2

0 1 3 4

14 4 4,8 0

10a a a a a ad+ + + − = ≥

con independencia del valor de d.

Solución 3.

Como se trata de cinco términos en progresión aritmética, se tiene

0 4 2 1 32a a a a a+ = = +

o también

( )( )0 4 2 22 2 .a a a d a d= − +

Entonces

( ) ( )33 3 3

0 4 0 4 0 4 0 4 2 0 4 23 8 6 ,a a a a a a a a a a a a+ = + − + = −

y

( ) ( ) ( ) ( )33 3 3

1 3 1 3 1 3 1 3 2 1 0 24 3 4 8 6a a a a a a a a a a a a a + = + − + = −

.

Entonces, lo que hay que probar es

( )3 3 3

2 2 0 1 2 2 1 3 2

18 6 32 24

10a a a a a a a a a≤ − + −

cuyo segundo miembro es

( )3

2 2 0 4 1 3

64 4 ;

10a a a a a a− +

trasponiendo términos, la desigualdad a probar se escribe como

( )2 2 2 2 3 3 2 3 2

2 2 2 2 2 2

3 24 244 4 4 3 3 3 0

5 5 5a a d a d a a d a d− + − ≤ ⇔ − ≤ ⇔ − ≤

la última desigualdad es cierta y hemos terminado.

Problema 2.

Determinar todos los posibles valores enteros no negativos que puede tomar la expresión

,1

22

++

mn

nmnm siendo m y n enteros no negativos tales que .1≠mn

Solución.

Sea Nkkmn

nmnm∈=

++,

1

22

(naturales con el ).0

En el caso ,nm = el número 1

33

2 −+=

mk es un entero positivo si 0=m ó ;2=m de donde

0=k ó 4=k respectivamente.

Page 100: Olimpiada Matematica Española 1993-2010

3

El caso 0=n lleva a que 2mk −= y por tanto .0== km

Consideremos ahora las soluciones ),( nm tales que .0>> nm Como la relación dada es

equivalente a ,0)1( 22 =++−− knmnkm observamos que si ),( nm es una solución y

,0)1( >−−> mnkn entonces ))1(,( mnkn −− es también una solución. La desigualdad

0)1( >−− mnk es cierta en todos los casos porque se convierte sucesivamente en una desigualdad

obvia:

,n

nmk

+> ,

1

22

n

nm

mn

nmnm +>

++ .3

nmn −−>

Del mismo modo la desigualdad mnkn −−> )1( es sucesivamente equivalente a:

,2

n

nmk

+< ,

2

1

22

n

nm

mn

nmnm +<

++

1

32 −

+>n

nnm para .1>n

Si ,13 2 −< nn esto es si ,4≥n la desigualdad anterior es cierta y por tanto para cada solución

),( nm con 4≥> nm encontramos una solución ),( pn con .0>> pn De este modo, cada

solución es tal que .3≤n

Para ,1=n obtenemos ,1

32

−−+=

mmk 4=m ó .7,2 == km

Para ,2=n obtenemos ,12

21524

−−+=

mmk 4=m ó 4,11 == km ó .7=k

Para ,3=n obtenemos ,13

911039

−++=

mmk que no conduce a ninguna solución.

Entonces los posibles valores k enteros no negativos de la expresión del enunciado son 4,0 y .7

Problema 3.

Sea O el circuncentro de un triángulo ABC. La bisectriz que parte de A corta al lado opuesto en P.

Probar que se cumple:

2 2 2AP OA OP bc+ − =

Solución:

Prolongamos AP hasta que corte en M al circuncírculo.

Los triángulos ABM y APC son semejantes al tener dos

ángulos iguales.(∠ACB = ∠AMB por inscritos en el mismo

arco y ∠BAN = ∠CAN por bisectriz).

Entonces:

·c AP

bc AM APAM b

= ⇔ =

como AM = AP + PM, queda:

( ) 2 ·bc AP AP PM AP AP PM= + = +

c

b

A

B C

O

P

M

Page 101: Olimpiada Matematica Española 1993-2010

4

AP·PM es la potencia de P respecto de la circunferencia circunscrita y su valor es OA2 – OP

2 sólo

queda sustituir y resulta:

2 2 2bc AP OA OP= + −

Problema 4.

¿Cuáles son los números enteros positivos que se pueden obtener de exactamente 2007 maneras

distintas, como la suma de al menos dos números enteros positivos consecutivos? ¿Cuál es el menor

de todos ellos?

Ejemplo: el número 9 se escribe exactamente de dos maneras distintas:

9 = 4 + 5

9 = 2 + 3 + 4

Solución

( ) ( )( )( )

( )( )1 2

1 2 1 22

n a nN a a a n N n a n

+ += + + + + + = ⇔ = + +K

Si n es par ( )1n + es impar y ( )2a n+ es par.

Si n es impar ( )1n + es par y ( )2a n+ es impar.

Siempre que 2N en su descomposición en factores primos tenga un factor impar distinto de uno,

existe una descomposición de N en suma de números consecutivos.

En efecto, sea ( )2 1N q Q= + con q, Q naturales y 1≥q , 1>Q , entonces:

a) )2)(1().12(2 nanQq ++=+ , y hacemos la siguiente descomposición:

112 +=+ nq , es decir qn 2= y 1>n

qaQ 222 += , de donde qQa −= Si 0a > ya hemos terminado, en otro caso

0Q q− ≤ y entonces,

b) 12 += nQ , de donde 12 −= Qn , es decir 1n ≥ y 12122 +=−+ qQa de donde Qqa −+= 1 , es

decir 1a ≥ y ya hemos terminado. Entonces necesitamos el menor número que en su descomposición tenga 2007 factores impares sin

contar el 1.

Si sólo tiene un factor primo es:

2.322007=N

Si tiene más de uno ..γβα rqp , el número de divisores que tiene es )......1)(1)(1( +++ γβα , y este

producto debe ser 2008 para que excepto l uno tenga 2007 divisores impares distintos.

)......1)(1)(1(251.22008 3 +++== γβα , entonces:

72505.3=N , o bien 7.5.3

3250=N , o bien 11.7.5.3250=N y este último es el menor.

Problema 5.

Sea 1a ≠ un número real positivo y n un entero positivo. Demostrar que 2

1

2.

2

n na an

a a

+ −<

+ −

Solución.

Page 102: Olimpiada Matematica Española 1993-2010

5

La desigualdad dada 2

1

2

2

n na a

na a

+ −<

+ − es equivalente a

2

2 2

2

21 1

2 2

,

n n

a a

n

a a

<

que a su vez equivale a

que 1

,n n

nα α

α α

−<

− siendo .aα = Entonces, usando la desigualdad aritmético-geométrica, se tiene

la desigualdad pedida:

21 1 2 4 2 2 1 2 4 ... (2 2) 1 1

1 2

1(1 ... ) .

1

n n nnn n n n n n n

n n nα α α

α α α α α α α α αα α α

−− − − − + + + − − −

− −= = + + + + > = =

− −

Problema 6.

Dada una semicircunferencia de diámetro AB = 2R, se considera una cuerda CD de longitud fija c.

Sea E la intersección de AC con BD y F la intersección de AD con BC.

Probar que el segmento EF tiene longitud constante y dirección constante al variar la cuerda CD

sobre la semicircunferencia.

Solución.

Como los triángulos EFC y EDF son rectángulos, el

cuadrilátero EDFC es inscriptible y EF es el diámetro.

Llamemos r al radio del circuncírculo de ECD, por el

teorema de los senos en ECD:

( )2 1sen

cEF r

E= =

Pongamos α = ∠BOD y β = ∠COD. Entonces

18090

2 2E

β β−= = −

expresión que prueba que el ángulo E es constante al serlo β y además el punto E se mueve en el

arco capaz de 2

90β

− sobre AB. Sustituyendo en (1) queda:

( )2 2

cos2

cEF r

β= = , por otra parte ( )3

R2

c

2sen =

β; eliminando β entre (2) y (3) y despejando

EF resulta: 2 2

2

4

cREF

R c=

− expresión que muestra que EF es constante al serlo c y R.

Además F es el ortocentro del triángulo ABE como intersección de las alturas AD y BC, por ello EF

que está sobre la tercera altura es siempre perpendicular a AB.

También admite solución analítica aunque mucho más larga.

βα

F

E

D

OA B

C

Page 103: Olimpiada Matematica Española 1993-2010

XLIV Olimpiada Matemática Española Fase nacional 2008 (Valencia)

PRIMERA SESIÓN (28 de marzo)

• 1.- Halla dos enteros positivos a y b conociendo su suma y su mínimo común múltiplo. Aplícalo en el caso de que la suma sea 3972 y el mínimo común múltiplo .985928

• 2.- Prueba que para cualesquiera números reales ba, tales que ,1,0 << ba se cumple la desigualdad siguiente:

.2)1()1()1)(1( 2222 <−−+−−++ bababaab

• 3.- Sea 3≥p un número primo. Se divide cada lado de un triángulo en p partes iguales y se une cada uno de los puntos de división con el vértice opuesto. Calcula el número máximo de regiones, disjuntas dos a dos, en que queda dividido el triángulo.

No está permitido el uso de calculadoras. Cada problema se valora hasta 7 puntos.

El tiempo de cada sesión es de tres horas y media.

Page 104: Olimpiada Matematica Española 1993-2010

XLIV Olimpiada Matemática Española Fase nacional 2008 (Valencia)

SEGUNDA SESIÓN (29 de marzo)

• 4.- Sean p y q dos números primos positivos diferentes. Prueba que existen enteros positivos a y ,b tales que la media aritmética de todos los divisores positivos del número baqpn = es un número entero.

• 5.- Dada una circunferencia y en ella dos puntos fijos A, B, otro variable P y una recta ;r se trazan las rectas PA y PB que cortan a r en C y D respectivamente. Determina dos puntos fijos de ,r M y N, tales que el producto ·CM DN sea constante al variar P.

• 6.- A cada punto del plano se le asigna un solo color entre siete colores distintos. ¿Existirá un trapecio inscriptible en una circunferencia cuyos vértices tengan todos el mismo color?

No está permitido el uso de calculadoras. Cada problema se valora hasta 7 puntos.

El tiempo de cada sesión es de tres horas y media.

Page 105: Olimpiada Matematica Española 1993-2010

1

XLIV Olimpiada Matemática Española Fase nacional 2008 (Valencia)

PRIMERA SESIÓN (28 de marzo) 1.- Halla dos enteros positivos a y b conociendo su suma y su mínimo común múltiplo. Aplícalo en el caso de que la suma sea 3972 y el mínimo común múltiplo .985928 SOLUCIÓN: Sea p un número primo que divide a la suma ba + y a su mínimo común múltiplo [ ].,ba Como [ ]bap , al menos divide a uno de los dos enteros a ó .b Si ,ap al

dividir p a la suma ,ba + también .bp (Obviamente el mismo razonamiento vale si

hubiéramos supuesto que ).bp Por tanto podemos dividir los dos números a y b por p y también su mínimo común múltiplo [ ],,ba para obtener dos enteros 1a y 1b

tales que pbb

paa == 11 , y [ ] [ ].,, 11 p

baba = Sea el máximo común divisor de a y ,b

( ).,bad = Repitiendo el proceso anterior llegaremos a obtener dos enteros A y B tales que dBbdAa == , y ( ) .1, =BA Entonces [ ] ., ABBA = Ahora es fácil

determinar A y B a partir del sistema de ecuaciones [ ] .,

⎪⎪⎩

⎪⎪⎨

=

+=+

dbaAB

dbaBA

Es decir A y

B son las raíces de la ecuación de segundo grado [ ] .0,)(2 =++− batbatd Observamos que el discriminante de esta ecuación es no negativo. En efecto: [ ] .0)(4)(,4)( 222 ≥−=−+=−+=∆ baabbabadba Si a y b son distintos, la ecuación anterior tiene por soluciones los dos enteros

positivos daA = y .

dbB =

En particular cuando 3972=+ ba y [ ] ,985928, =ba tenemos que .4)985928,3972( ==d Por tanto Aa 4= y Bb 4= siendo A y B las raíces de la ecuación .098592839724 2 =+− tt Es decir 491=A y 502=B y los números buscados son 1964=a (año de la primera OME) y 2008=b (año de la actual edición de la OME).

Page 106: Olimpiada Matematica Española 1993-2010

2

XLIV Olimpiada Matemática Española Fase nacional 2008 (Valencia)

PRIMERA SESIÓN (28 de marzo) 2.- Prueba que para cualesquiera números reales ba, tales que ,1,0 << ba se cumple la desigualdad siguiente:

.2)1()1()1)(1( 2222 <−−+−−++ bababaab

SOLUCIÓN:

Se verifica que 3 xx < para todo ).1,0(∈x Teniendo en cuenta que ,12

0 <+

<ba

utilizando la desigualdad anterior y aplicando la desigualdad entre las medias aritmética y geométrica, se tiene:

232

223

bababa

baabbaab +=

⎟⎠⎞

⎜⎝⎛ +

++≤⎟

⎠⎞

⎜⎝⎛ +

<⎟⎠⎞

⎜⎝⎛ +

y

.2

13

2111

21)1()1(

21)1()1( 3

bababa

babababa

+−=

+−+−+−

≤⎟⎠⎞

⎜⎝⎛ +−−−<⎟

⎠⎞

⎜⎝⎛ +−−−

Sumando las expresiones anteriores resulta

,12

1)1()1(2

<⎟⎠⎞

⎜⎝⎛ +−−−+⎟

⎠⎞

⎜⎝⎛ + bababaab

o equivalentemente

( ) ,1)1()1()1)(1(2

1 2222 <−−+−−++ bababaab

de donde se obtiene inmediatamente la desigualdad del enunciado.

Page 107: Olimpiada Matematica Española 1993-2010

3

XLIV Olimpiada Matemática Española Fase nacional 2008 (Valencia)

PRIMERA SESIÓN (28 de marzo) 3.- Sea 3≥p un número primo. Se divide cada lado de un triángulo en p partes iguales y se une cada uno de los puntos de división con el vértice opuesto. Calcula el número máximo de regiones, disjuntas dos a dos, en que queda dividido el triángulo. SOLUCIÓN: En primer lugar veremos que tres de estos segmentos (cevianas) no pueden ser concurrentes. Sea el triángulo ABC y ZYX ,, puntos de las divisiones interiores de los lados ABACBC ,, respectivamente. Si BYAX , y CZ fueran concurrentes aplicando el teorema de Ceva tendríamos

.1=⋅⋅YACY

XCBX

ZBAZ

Por otro lado, por la forma en que hemos construido los puntos de división, existen enteros positivos { },1,...,2,1,, −∈ pmlk tales que

.,,mp

mYACY

lpl

XCBX

kpk

ZBAZ

−=

−=

−=

Sustituyendo en la expresión anterior

),()()( mplpkpmlk −−−= o equivalentemente,

).()()(2 lpkpplkpmpmlk −−=−−+ De aquí resulta que p divide al producto ,klm que es imposible y nuestra afirmación inicial queda probada. Dibujando las cevianas desde el vértice A el triángulo ABC queda dividido en p triángulos. Las cevianas trazadas desde B dividen cada uno de los p triángulos

anteriores en p partes disjuntas, teniendo en total 2p regiones. Cada ceviana trazada desde el vértice C aumenta el número de regiones en un número exactamente igual a su número de intersecciones con las rectas que encuentra (incluido el lado AB ). Es decir,

.121)1(2 −=+− pp Por tanto, el número máximo de regiones disjuntas dos a dos, en que queda dividido el triángulo ABC es .133)12)(1( 22 +−=−−+ ppppp

Page 108: Olimpiada Matematica Española 1993-2010

1

XLIV Olimpiada Matemática Española Fase nacional 2008 (Valencia)

SEGUNDA SESIÓN (29 de marzo)

4.- Sean p y q dos números primos positivos diferentes. Prueba que existen enteros positivos a y ,b tales que la media aritmética de todos los divisores positivos del número baqpn = es un número entero. SOLUCIÓN: La suma de todos los divisores de n viene dada por la fórmula

),...1()...1( 22 ba qqqppp ++++++++ como se puede comprobar desarrollando los paréntesis. El número n tiene

)1)(1( ++ ba divisores positivos y la media aritmética de todos ellos es

.)1()1(

)...1()...1( 22

++++++++++

=ba

qqqpppmba

Si p y q son ambos impares, tomando pa = y ,qb = es fácil ver que m es un entero. Efectivamente: cada factor ppppp +++++ ...1 32 y qqqqq +++++ ...1 32 tiene un número par de sumandos y por ejemplo, el primero se puede escribir como sigue

)....1()1()1(...)1()1(...1

12

1232

++++

=++++++=+++++p

pp

pppppppppppp

Análogamente el segundo factor )....1()1(...1 1232 −++++=+++++ qq qqqqqqq Entonces ),...1()...1( 142142 −− ++++++++= qp qqqpppm que es un entero positivo. Si 2=p y q es impar, se eligen qb = y ....11 142 −++++=+ qqqqa Entonces

aq

qqqq

qqqqqqqqm

q

2...221)1()...1(

)...1()2...221( 2142

32...2 142

++++=+++++

+++++++++= −

+++ −

, que es

entero. Para 2=q y p impar, análogamente al caso anterior se eligen pa = y

142 ... −+++= ppppb y m es entero. Alternativamente y de una manera casi directa, se obtiene una solución completa observando que si p y q son primos impares se toman 1== ba y si 2=p y q primo

impar, entonces se consideran 2

1−=

qa y .1=b

Page 109: Olimpiada Matematica Española 1993-2010

2

XLIV Olimpiada Matemática Española Fase nacional 2008 (Valencia)

SEGUNDA SESIÓN (29 de marzo) 5.- Dada una circunferencia y en ella dos puntos fijos A, B, otro variable P y una recta ;r se trazan las rectas PA y PB que cortan a r en C y D respectivamente. Determina dos puntos fijos de ,r M y N, tales que el producto ·CM DN sea constante al variar P. SOLUCIÓN: Trazamos las paralelas a r por A y B que cortan a la circunferencia en A y ´B respectivamente de modo que ´´BBAA es un trapecio isósceles. Las intersecciones de ´AB y ´BA con r determinan los puntos M y N buscados. En efecto, los triángulos AMC y DNB (sombreados en la figura) son semejantes ya que tienen dos ángulos iguales:

,´ NDBBPBMAC ∠=∠=∠ donde la primera igualdad es cierta por ser ángulos inscritos en el mismo arco y la segunda por ser ´BB paralela a .r

,´ DNBBABAMC ∠=∠=∠ con argumentos análogos a los anteriores. Estableciendo la proporcionalidad de los lados resulta

,BNAMNDMCBNND

MCAM

⋅=⋅⇔=

cantidad que no depende de P.

r NM

B'

A'

C

DA

B

P

Page 110: Olimpiada Matematica Española 1993-2010

3

Se observa que si la recta r pasa por el punto ,A ,CAM == no se forma el triángulo

.AMC En este caso 0=CM y el producto ,0=⋅DNCM es constante. Análogamente este producto es cero si la recta r pasa por B o por los puntos A y B en cuyo caso

.0== DNCM

XLIV Olimpiada Matemática Española Fase nacional 2008 (Valencia)

SEGUNDA SESIÓN (29 de marzo) 6.- A cada punto del plano se le asigna un solo color entre siete colores distintos. ¿Existirá un trapecio inscriptible en una circunferencia cuyos vértices tengan todos el mismo color? SOLUCIÓN: La idea inicial es considerar una circunferencia C de radio r y sobre ella bloques de 8 puntos 821 ,...,, AAA igualmente espaciados; es decir que los arcos 7,...,11 =+ iAA ii tengan igual longitud 0>λ (que se elegirá convenientemente) para cada uno de los bloques. Se elige un sentido dado (por ejemplo, el antihorario). Se disponen entonces, en este sentido antihorario, 50177 =+× bloques de 817 =+ puntos cada uno en la semicircunferencia superior de ,C tales que dos bloques distintos no se intersequen o solapen, para lo cual se toma λ suficientemente pequeño, por

ejemplo .400

0 rπλ <<

Se observa que al menos hay dos puntos del mismo color en cada bloque. Se eligen dos de esos puntos y su color se le asocia al bloque. Y la distancia entre estos dos puntos que es uno de los siete números { },7,6,5,4,3,2,1, ∈= nndn λ se le asigna también al bloque. De este modo a cada uno de los 50 bloques se le hace corresponder el par (color, distancia), indicado anteriormente. Como el número total de posibles pares es

,49 por el principio el palomar, existirán dos bloques R y Q a los que se les asocia el mismo par (color, distancia). Por tanto los cuatro puntos determinados por estos dos bloques tienen el mismo color. Y como los dos puntos del bloque R distan igual que los dos puntos del bloque ,Q estando los cuatro puntos sobre la circunferencia

,C necesariamente, estos cuatro puntos son los vértices de un trapecio inscriptible. NOTA: Este mismo razonamiento se podría hacer considerando un arco de circunferencia de radio r de

longitud .20, rll π≤< En este caso se tomaría .400

0, l<< λλ

Y también se podría generalizar a un número de colores )2( ≥cc cualesquiera, considerando 1+c

puntos en vez de 17 + y 12 +c bloques disjuntos de 1+c puntos cada uno, en vez de 172 + bloques

de 17 + puntos cada uno. Y ahora λ debe cumplir que .)1)(1(

0 2 ++<<

cclλ

Page 111: Olimpiada Matematica Española 1993-2010

Fase Nacional de la XLV Olimpiada Matemática Española Sant Feliu de Guixols (Girona), 27 de marzo de 2009

PRIMERA SESIÓN

1.- Halla todas las sucesiones finitas de números naturales consecutivos con tales que

n,,...,, 21 naaa ,3≥n .2009...21 =+++ naaa

2.- Sean un triángulo acutángulo, ABC I el centro del círculo inscrito en el triángulo

,ABC r su radio y R el radio del círculo circunscrito al triángulo Se traza la altura con perteneciente al lado Demuestra que

.ABC,ahAD = D .BC

).2()2(2 rhhRDI aa −−=

3.- Se pintan de rojo algunas de las aristas de un poliedro regular. Se dice que una coloración de este tipo es buena, si para cada vértice del poliedro, existe una arista que concurre en dicho vértice y no está pintada de rojo. Por otra parte, se dice que una coloración donde se pintan de rojo algunas de las aristas de un poliedro regular es completamente buena, si, además de ser buena, ninguna cara del poliedro tiene todas sus aristas pintadas de rojo. ¿Para qué poliedros regulares es igual el número máximo de aristas que se pueden pintar en una coloración buena y en una completamente buena? Justifica la respuesta.

No está permitido el uso de calculadoras. Cada problema vale siete puntos.

El tiempo de cada sesión es de tres horas y media.

Page 112: Olimpiada Matematica Española 1993-2010

Fase Nacional de la XLV Olimpiada Matemática Española Sant Feliu de Guixols (Girona), 28 de marzo de 2009

SEGUNDA SESIÓN

4.- Determina justificadamente todos los pares de números enteros que verifican la ecuación

),( yx.200942 =− yx

5.- Sean números reales positivos tales que cba ,, .1=abc Prueba la desigualdad

siguiente

+⎟⎠⎞

⎜⎝⎛+

2

1 aba

+⎟⎠⎞

⎜⎝⎛

+

2

1 bcb

43

1

2

≥⎟⎠⎞

⎜⎝⎛+ cac

6.- En el interior de una circunferencia de centro y radio O ,r se toman dos puntos A y simétricos respecto de Se considera ,B .O P un punto variable sobre esta circunferencia y se

traza la cuerda perpendicular a Sea el punto simétrico de ´,PP .AP C B respecto de Halla el lugar geométrico del punto intersección de con al variar sobre la circunferencia.

´.PP,Q ´PP ,AC P

No está permitido el uso de calculadoras. Cada problema vale siete puntos.

El tiempo de cada sesión es de tres horas y media.

Page 113: Olimpiada Matematica Española 1993-2010

Fase Nacional de la XLV Olimpiada Matemática Española Sant Feliu de Guixols (Girona), 27 de marzo de 2009

PRIMERA SESIÓN

SOLUCIONES

PROBLEMA 1.- Halla todas las sucesiones finitas de números naturales consecutivos con tales que

n,,...,, 21 naaa ,3≥n .2009...21 =+++ naaa

Primera solución:

Supongamos que es la suma de números naturales consecutivos empezando por Entonces

N n.1+k

=++++++= )(...)2()1( nkkkN [ ] [ ] =+++−++++++++ knkkk ...21)(...)1(...21

.2

)12(2

)1(2

)1)(( ++=

+−

+++ nknkknknk

Teniendo en cuenta que 41492877200912009 ×=×=×= se tienen los

siguientes casos:

(1) Si y 7=n ( ) 2872

12=

++ nk resulta 283=k con lo que

.2902892882872862852842009 ++++++=

(2) Si 72=

n y ,28712 =++ nk resulta 136=k con lo que

.150...1381372009 +++=

(3) Si y 41=n ( ) 492

12=

++ nk resulta 28=k con lo que

.69...3130292009 ++++=

(4) Si y 49=n ( ) 412

12=

++ nk resulta 16=k con lo que

.65...1918172009 ++++=

(5) Los otros casos dan valores de k que no verifican el enunciado.

Page 114: Olimpiada Matematica Española 1993-2010

Segunda solución:

Claramente, es ,11 −+= naan de donde .2

)1(2009 1−

+=nnan Entonces

,2

)1(2

)1(20092−

>−

≥nnn luego ,40181<−n y al ser

resulta que ,6339694018409664 22 =>>= ,644018 < con lo que .65<n Supongamos en primer lugar que n es impar. Entonces, obviamente divide a

y puede tomar los valores pues cualquier otro divisor impar de es mayor o igual que

n4172009 2 ×= n 41,7 ó ,49

2009 .287417 =× Se obtiene entonces

,2

120091

−−=

nn

a con valores respectivos y es decir, se obtienen

las tres sucesiones

29,284 ,17

;290,...,285,284 ;69,...,30,29 .65,...,18,17

No hay otras sucesiones con un número impar de términos.

Supongamos finalmente que n es par. Entonces 2n divide a con lo que

ya que cualquier otro valor de ha de ser mayor o igual que lo que no es posible. Entonces

,2009

,14=n n,6582412 >=× ,1371 =a y la única sucesión

con número par de términos es .150,...,138,137

Page 115: Olimpiada Matematica Española 1993-2010

Fase Nacional de la XLV Olimpiada Matemática Española Sant Feliu de Guixols (Girona), 27 de marzo de 2009

PRIMERA SESIÓN

SOLUCIONES PROBLEMA 2 .- Sean un triángulo acutángulo, ABC I el centro del círculo inscrito en el triángulo ,ABC r su radio y R el radio del círculo circunscrito al triángulo Se traza la altura con perteneciente al lado Demuestra que

.ABC,ahAD = D .BC

).2()2(2 rhhRDI aa −−= Primera solución: Sean E y M las proyecciones ortogonales de I sobre y respectivamente.

BC ,AD

Se tiene: ;

2Asen

rAI = pSr = ⇒

2Asenp

SAI⋅

= (1) donde, evidentemente,

es el área del triángulo y S ABC p es su semiperímetro.

Por otra parte, ,2

cos22

AAsenbcsenAbcS ⋅⋅=⋅

= así que (1) se puede escribir

como ,2cos

p

AbcAI

⋅= y ya que ,)(

2cos2

bcappA −

= obtenemos

papbcAI )(2 −

= (2).

Teniendo en cuenta que ,2 ahRbc ⋅= ,rSp = ,2

ahSa = la expresión (2) se

escribe como )2(2212

2

122 rhRhrhR

rShS

hRAI aa

aa

a −=⎟⎟⎠

⎞⎜⎜⎝

⎛−⋅=

⎟⎟⎟⎟

⎜⎜⎜⎜

−⋅=

(3).Como el cuadrilátero IEDM es un rectángulo, .rIEMD == Aplicando el teorema de Pitágoras generalizado a ADI tenemos ⇔⋅−+= AMhAIhDI aa 2222 ),(2222 MDhhAIhDI aaa −−+=y teniendo en cuenta los resultados anteriormente obtenidos resulta, finalmente,

c.q.d. ),2()2()(2)2(222 rhhRrhhrhRhDI aaaaaa −−=−−−+=

Page 116: Olimpiada Matematica Española 1993-2010

Segunda solución: Sean las longitudes de los lados y cba ,, CABC , AB respectivamente, y sea T el punto donde la circunferencia inscrita es tangente al lado Por el teorema de Pitágoras, al ser

.BC,// DTADIT ⊥ se tiene que

222222 ADATrDTrDI −+=+= y la igualdad a demostrar es equivalente a .)(24 22

ahrRrRrAT +=++Ahora bien, llamando al área de es conocido que S ,ABC

.2

)()()()(41

2)(

4ahacbabacacbcbacbar

RabcS =−+−+−+++=

++==

De aquí se deduce que ,24cba

abcRr++

= ,)()()(2

cbacbabacacbr

++−+−+−+

=

bchR a =2 y .2

)()()(2a

cbabacacbhR a−+−+−+

=

Obsérvese que .4

2224222

2 cbacabcabrRr −−−++=+ Ahora bien, se

sabe que ,2

bacBT −+= .

2cbaCT −+

=

Por el teorema de Stewart, tenemos que

=⋅−⋅+⋅

= CTBTBC

ABCTACBTAT22

2

acbcbbcacb

2)()(

4233 2222 +−

−−−+

con lo que

,2

)()(2

42222

22

acbcbcaabacbrRrAT +−

=++−+

=++

e identificando términos se comprueba que esto coincide con el valor de ahrR )(2 +

y de este modo la igualdad requerida queda demostrada. Tercera solución: Nótese en primer lugar que

,2

22

2

222)(222

2

2

2

RIA

AsenR

rAsen

CsenBsenr

aracb

aarSrha ===

−+=

−=−

donde es el área de y se ha utilizado que S ABC .222

4 CsenBsenAsenRr =

Sea ahora el punto P en el que la paralela a por BC I corta a la altura Claramente,

.AD

).2()2()2()( 22222 rhhRrhhIArhIArDI aaaaa −−=−−=−−+=

Page 117: Olimpiada Matematica Española 1993-2010

Fase Nacional de la XLV Olimpiada Matemática Española Sant Feliu de Guixols (Girona), 27 de marzo de 2009

PRIMERA SESIÓN

SOLUCIONES

PROBLEMA 3.- Se pintan de rojo algunas de las aristas de un poliedro regular. Se dice que una coloración de este tipo es buena, si para cada vértice del poliedro, existe una arista que concurre en dicho vértice y no está pintada de rojo. Por otra parte, se dice que una coloración donde se pintan de rojo algunas de las aristas de un poliedro regular es completamente buena, si además de ser buena, ninguna cara del poliedro tiene todas sus aristas pintadas de rojo. ¿Para qué poliedros regulares es igual el número máximo de aristas que se pueden pintar en una coloración buena y en una completamente buena? Justifica la respuesta.

Solución: Claramente, las coloraciones completamente buenas son un subconjunto de las coloraciones buenas, con lo que si el máximo número de aristas que se pueden pintar de rojo para obtener una coloración buena se puede alcanzar con una coloración completamente buena, la pregunta del enunciado tiene respuesta afirmativa. NOTA: En cada caso véase el recuadro de las figuras al final de la solución. En el caso de un tetraedro, existen 6 aristas, tales que en cada vértice confluyen

de ellas. El número máximo de aristas pintadas de rojo en una coloración

buena sería por lo tanto

3

,4632

=× pues en caso contrario existiría algún vértice

donde más de 32 de las aristas estuvieran pintadas de rojo, es decir, todas las

aristas estarían pintadas de rojo. La figura muestra una coloración completamente buena de un tetraedro con aristas rojas (el tetraedro ha sido deformado para poder ser dibujado en el plano).

4

De igual forma, en el cubo existen 12 aristas, tales que en cada vértice confluyen de ellas. El número máximo de aristas pintadas de rojo en una

coloración buena sería, por lo tanto,

3

.81232

=× La figura muestra una

coloración completamente buena con 8 aristas rojas. Finalmente, en el dodecaedro existen 30 aristas, tales que en cada vértice confluyen de ellas. El número máximo de aristas pintadas de rojo en una 3

Page 118: Olimpiada Matematica Española 1993-2010

coloración buena sería, por lo tanto, .203032

=× La figura muestra una

coloración completamente buena de un dodecaedro con 12 aristas rojas. De lo anterior se deduce que para el tetraedro, el cubo y el dodecaedro, el número máximo de aristas rojas en una coloración buena se alcanza con una coloración completamente buena. En el octaedro existen 12 aristas, tales que en cada vértice confluyen 4 de ellas. El número máximo de aristas pintadas de rojo en una coloración buena es

por lo tanto .91243

=× La figura muestra una coloración buena de un octaedro

con aristas rojas. Ahora bien, como cada arista pertenece a dos caras, sumando para las 8 caras el número de aristas pintadas de rojo en dicha cara, obtenemos con lo que hay, al menos, dos caras con 3 aristas pintadas de rojo y una coloración buena con el número máximo de 9 aristas pintadas de rojo nunca puede ser completamente buena.

9

,282×18 +=

Finalmente, en el icosaedro existen 30 aristas, tales que en cada vértice confluyen de ellas. El número máximo de aristas pintadas de rojo en una

coloración buena es por lo tanto

5

.243054

=× La figura muestra una

coloración buena de un icosaedro con 24 aristas rojas. Ahora bien, eso quiere decir, que sumando el número de aristas rojas de cada cara para todas las caras, obtenemos es decir, existen, al menos, caras con todas sus aristas rojas, y una coloración buena con el número máximo de aristas pintadas de rojo nunca puede ser completamente buena.

,820248 +×= 824

Por lo tanto, los poliedros regulares que tienen la propiedad descrita en el enunciado son el tetraedro, el cubo y el dodecaedro (es decir, los poliedros tales que en cada vértice confluyen exactamente 3 aristas) y los que no la tienen son el octaedro e icosaedro (en cuyos vértices confluyen más de 3 aristas).

Page 119: Olimpiada Matematica Española 1993-2010

FIGURAS FIGURAS

Page 120: Olimpiada Matematica Española 1993-2010

Fase Nacional de la XLV Olimpiada Matemática Española Sant Feliu de Guixols (Girona), 28 de marzo de 2009

SEGUNDA SESIÓN

SOLUCIONES

PROBLEMA 4. - Determina justificadamente todos los pares de números enteros

que verifican la ecuación ),( yx .200942 =− yx

Solución: Dada una solución cualquiera, es claro que también son soluciones

y con lo que se puede asumir sin pérdida de generalidad que Supongamos entonces que es así. Es claro que

),( yx),(),,( yxyx −− ),,( yx −−

.0, ≥yx.417)()( 222 ⋅=+− yxyx

Si y no son primos entre sí, su máximo común divisor al cuadrado divide a luego es 7 y divide a

y a con lo que existen enteros no negativos y v tales que

2yx − 2yx +,4172009 2 ⋅=

xyxyx 2)()( 22 =−++ ,2)()( 222 yyxyx =−−+u ,7ux = y

Como ambos factores han de ser enteros, se tiene que

y con lo que

vy 7=.41)7)(7( 22 =−+ vuvu

417 2 =+ vu ,17 2 =− vu 21=u y .7

102 =v No existen pues

soluciones enteras en este caso. Si y son primos entre sí, un posible caso es que y

con lo que absurdo pues Resta entonces tan sólo el caso en que y que produce

con lo que la única solución con enteros no negativos es e y las únicas soluciones en enteros son

2yx − 2yx + 12 =− yx,20092 =+ yx ,10042 =y .32100496131 22 <<=

412 =− yx ,492 =+ yx,45=x ,42 =y

45=x ,2=y ).2,45(),( ±±=yx

Page 121: Olimpiada Matematica Española 1993-2010

Fase Nacional de la XLV Olimpiada Matemática Española Sant Feliu de Guixols (Girona), 28 de marzo de 2009

SEGUNDA SESIÓN

SOLUCIONES

PROBLEMA 5. - Sean números reales positivos tales que Prueba la desigualdad siguiente

cba ,, .1=abc

+⎟⎠⎞

⎜⎝⎛+

2

1 aba

+⎟⎠⎞

⎜⎝⎛+

2

1 bcb

43

1

2

≥⎟⎠⎞

⎜⎝⎛+ cac

Solución:

Como ,1=abc entonces .11

222

⎟⎠⎞

⎜⎝⎛+

=⎟⎠⎞

⎜⎝⎛

+=⎟

⎠⎞

⎜⎝⎛+ c

cacabc

caab

a Análogamente se

obtienen 22

11⎟⎠⎞

⎜⎝⎛+

=⎟⎠⎞

⎜⎝⎛+ a

abbc

b y .11

22

⎟⎠⎞

⎜⎝⎛+

=⎟⎠⎞

⎜⎝⎛+ b

bcca

c Por tanto la

desigualdad requerida se convierte en

+⎟⎠⎞

⎜⎝⎛+

2

1 aab

+⎟⎠⎞

⎜⎝⎛+

2

1 bbc ,

43

1

2

≥⎟⎠⎞

⎜⎝⎛+ cca equivalente a

.21

11131 222

≥⎥⎥⎦

⎢⎢⎣

⎡⎟⎠⎞

⎜⎝⎛+

+⎟⎠⎞

⎜⎝⎛+

+⎟⎠⎞

⎜⎝⎛+ c

cab

bca

ab

Usando ahora la desigualdad entre las medias aritmética y cuadrática, se obtiene

.1113

11113

1 222

⎥⎦

⎤⎢⎣

⎡⎟⎠⎞

⎜⎝⎛+

+⎟⎠⎞

⎜⎝⎛+

+⎟⎠⎞

⎜⎝⎛+

≥⎥⎥⎦

⎢⎢⎣

⎡⎟⎠⎞

⎜⎝⎛+

+⎟⎠⎞

⎜⎝⎛+

+⎟⎠⎞

⎜⎝⎛+ c

cab

bca

abc

cab

bca

ab

Así es suficiente demostrar que 23

111≥

++

++

+ cca

bbc

aab o equivalentemente

,23

)1()1()1(≥

++

++

+ cbabc

baabc

acabc que a su vez equivale a que

.23

)1(1

)1(1

)1(1

≥+

++

++ cbbaac

Page 122: Olimpiada Matematica Española 1993-2010

Poniendo zyb

yxa == , y

xzc = en la última desigualdad resulta

+⎟⎟⎠

⎞⎜⎜⎝

⎛+

−1

zx

yx

+⎟⎠⎞

⎜⎝⎛ +

−1

xy

zy .

23

1

≥⎟⎟⎠

⎞⎜⎜⎝

⎛+

yz

xz Sustituyendo ahora

yx1,1

== βα y

,1z

=γ se llega a la desigualdad de Nessbit .23

≥+

++

++ βα

γαγ

βγβ

α

La igualdad se alcanza si y sólo si .1=== cba

Page 123: Olimpiada Matematica Española 1993-2010

Fase Nacional de la XLV Olimpiada Matemática Española Sant Feliu de Guixols (Girona), 28 de marzo de 2009

SEGUNDA SESIÓN

SOLUCIONES

PROBLEMA 6. - En el interior de una circunferencia de centro O y radio ,r se toman dos puntos y simétricos respecto de Se considera un punto variable A ,B .O P sobre esta circunferencia y se traza la cuerda perpendicular a Sea el punto simétrico de

´,PP .AP CB respecto de Halla el lugar geométrico del punto intersección de ´.PP ,Q

´PP con al variar ,AC P sobre la circunferencia.

Primera solución: Establezcamos primero que es constante. ACMétodo 1. Se obtiene C a partir de A aplicando un giro de con centro en O

seguido de la simetría de eje PP º180´.

1 A

O AB

PC

Q

A'

P'

e1

e2

MDescomponiendo el giro en producto de dos simetrías de ejes perpendiculares e paralelo a P y

perpendicular a resulta que el triángulo es rectángulo en

2e ,APCAA A y además:

2´AA = ,MP2´CA = ;OM de donde es decir ;4444 2222 rOPMPOM ==+2AC =

2AC ,r= con independencia de la posición de P .

Método 2

O AB

P

P '

C

P ''

Prolongamos hasta que corte de nuevo a la circunferencia en P Se tiene CP

PA´´. ´´ P == ´´.APB

Además BP´ ´´;PPCA ´

es paralelo a luego el segmento es la imagen del segmento ´´PP mediante la

traslación de vector AP´´ P y como ´´´PPP∠ es recto y ´´´PP es un diámetro, resulta

.2´´´P rPAC == Finalmente, al ser PP la mediatriz de ´ ,BC

Page 124: Olimpiada Matematica Española 1993-2010

;QBQC = se deduce entonces que rACQAQCQAQB 2==+=+ y Q describe la elipse de focos y A B y constante La recta .2 r ´PP es la tangente en a la elipse. Q

Segunda solución:

Tomamos 1=r y unos ejes de coordenadas en los que la ecuación de la

circunferencia es y las coordenadas de con

,122 =+ yx ),0,(),0,( aBaA −.10 << a

En vez de empezar por sea con la condición Por las condiciones del problema, es el punto medio de llamando

a las coordenadas de se tiene Entonces la ecuación de la

recta CA es

,P ),(´ 00 yxP .120

20 =+ yx

´P ;BC ,( 11 yx

,C .22

01

01

⎩⎨⎧

=+=

yyaxx

),(0

0 axxy

y −= es decir .0000 =+− ayxyyx

)

Las pendientes de BP´ y de PP´ son respectivamente ax

y+0

0 y .0

0

yax +

− Por

tanto la ecuación de PP´ es .01)( 000 =+−++ axaxxyy Las coordenadas del punto intersección de y son: ,Q AC PP´

.1

)1(,

1 0

20

0

0⎟⎟⎠

⎞⎜⎜⎝

⎛+−

++

axay

axax

Q Denotando por yx, a las coordenadas de Q y

despejando los valores de e se obtiene 0x 0y .1

,1 00 −

−ax

y=

−−

= yax

xax

Imponiendo ahora la condición se llega a ,120

20 =+ yx

1)1()1(

)(2

2

2

2

=−

+−−

axy

axxa

y mediante operaciones se transforma en la ecuación ,11 2

22 =

−+

ayx que es la

ecuación de una elipse. Tercera solución: Demostraremos en primer lugar que, dados dos puntos del plano, el conjunto de los puntos (del mismo plano) tales que

BA,P 22 PBPA + es constante

y mayor que es una circunferencia de centro el punto medio de y que tiene a los puntos

,2AB ABA y B en su interior.

En efecto, supongamos )0,(),0,( dBdA −== y sea cualquier punto. Se tiene entonces

),( yxP =

.222 22222 dyxPBPA ++=+

Así que si se tiene que ,4 222 dkPBPA ≥=+ .2

2222 ddkyx ≥−=+

Page 125: Olimpiada Matematica Española 1993-2010

Sea entonces ahora R el punto donde corta a BC ´PP (que es perpendicular a ). Este punto BC R satisface

,22222 APARBRBPPR −=−= luego R está en la circunferencia y es distinto de , con lo que P ´.PR =Ahora bien, se tiene

,22

´)(´2

´)(´´222222 ABkBPAPPPkBPAPBPAPBPAP −

=−−−

=−−+

=⋅

donde .´´ 2222 BPAPBPAPk +=+=Además, la potencia de respecto de la circunferencia es A

,2

22

2222 ABkdkdr −=−=−

con lo que el segundo punto en el que S AP corta a la circunferencia es tal que ´.´ CPBPAS ==Como ,´´ APPPCP ⊥⊥ se tiene que es paralelo a y es un paralelogramo.

AS ´CP CASP´

Finalmente, ,442´´2´´ 222222222 rdkABkBPAPAPASASAPPSPP =−=−++=⋅⋅++=+

es decir, Como 'P S AC r= = 2 . ,2 rACBQAQ ==+ el lugar de es la elipse interiormente tangente a la circunferencia dada, con

QA y B como

focos.

Page 126: Olimpiada Matematica Española 1993-2010

OlimpiadaMatemáticaEspañola RSME

XLVI Olimpiada Matematica Espanola

Fase nacional 2010 (Valladolid)

Primera sesion (26 de marzo)

• Problema 1

Una sucesion pucelana es una sucesion creciente de dieciseis numeros impares positivosconsecutivos, cuya suma es un cubo perfecto. ¿Cuantas sucesiones pucelanas tienensolamente numeros de tres cifras?

• Problema 2

Sean N0 y Z el conjunto de todos los enteros no negativos y el conjunto de todos losenteros, respectivamente. Sea f : N0 → Z la funcion que a cada elemento n de N0 leasocia como imagen el entero f(n) definido por

f(n) = −f(⌊n

3

⌋)− 3

{n

3

}

donde �x� es la parte entera del numero real x y {x} = x − �x� su parte decimal.Determina el menor entero n tal que f(n) = 2010.

NOTA: La parte entera de un numero real x, denotada por �x� es el mayor entero queno supera a x. Ası �1,98� = 1, �−2,001� = −3, �7π − 8,03� = 13.

• Problema 3

Sea ABCD un cuadrilatero convexo. Sea P la interseccion de AC y BD. El angulo� APD = 60◦. Sean E, F , G y H los puntos medios de los lados AB, BC, CD y DArespectivamente. Halla el mayor numero real positivo k tal que

EG + 3HF ≥ kd + (1 − k)s

siendo s el semiperımetro del cuadrilatero ABCD y d la suma de las longitudes de susdiagonales. ¿Cuando se alcanza la igualdad?

No esta permitido el uso de calculadoras.Cada problema se puntua sobre siete puntos.

El tiempo de cada sesion es de tres horas y media.

Page 127: Olimpiada Matematica Española 1993-2010

OlimpiadaMatemáticaEspañola RSME

XLVI Olimpiada Matematica Espanola

Fase nacional 2010 (Valladolid)

Segunda sesion (27 de marzo)

• Problema 4

Sean a, b, c tres numeros reales positivos. Demuestra que

a + b + 3c

3a + 3b + 2c+

a + 3b + c

3a + 2b + 3c+

3a + b + c

2a + 3b + 3c≥ 15

8

• Problema 5

Sea P un punto cualquiera de la bisectriz del angulo A en el triangulo ABC, y sean A′,B′, C′ puntos respectivos de las rectas BC, CA, AB, tales que PA′ es perpendicular aBC, PB′ es perpendicular a CA y PC′ es perpendicular a AB. Demuestra que PA′ yB′C′ se cortan sobre la mediana AM , siendo M el punto medio de BC.

• Problema 6

Sea p un numero primo y A un subconjunto infinito de los numeros naturales. Sea fA(n) elnumero de soluciones distintas de la ecuacion x1+x2+· · ·+xp = n, con x1, x2, . . . , xp ∈ A.¿Existe algun numero natural N tal que fA(n) sea constante para todo n > N ?

No esta permitido el uso de calculadoras.Cada problema se puntua sobre siete puntos.

El tiempo de cada sesion es de tres horas y media.

Page 128: Olimpiada Matematica Española 1993-2010

OlimpiadaMatemáticaEspañola RSME

XLVI Olimpiada Matematica Espanola

Fase nacional 2010 (Valladolid, 26 y 27 de Marzo)Soluciones oficiales

• Problema 1

Una sucesion pucelana es una sucesion creciente de dieciseis numeros impares positivos con-secutivos, cuya suma es un cubo perfecto. ¿Cuantas sucesiones pucelanas tienen solamentenumeros de tres cifras?

Solucion:Sea la sucesion n, n + 2, . . . , n + 30. Entonces la suma es 1

2 16(2n + 30) = 8(2n + 30). Portanto, es necesario que 2n + 30 sea un cubo perfecto. Ahora hay que contar el numero detales n que son impares y verifican 101 ≤ n ≤ 969. Los cubos pares entre 232 y 1968 son512, 1000 y 1728, que corresponden a valores de n de 241, 485 y 849. Por lo tanto hayexactamente tres sucesiones pucelanas.

• Problema 2

Sean N0 y Z el conjunto de todos los enteros no negativos y el conjunto de todos los enteros,respectivamente. Sea f : N0 → Z la funcion que a cada elemento n de N0 le asocia comoimagen el entero f(n) definido por

f(n) = −f(⌊n

3

⌋)− 3

{n

3

}

donde �x� es la parte entera del numero real x y {x} = x−�x� su parte decimal. Determinael menor entero n tal que f(n) = 2010.

NOTA: La parte entera de un numero real x, denotada por �x� es el mayor entero que nosupera a x. Ası �1,98� = 1, �−2,001� = −3, �7π − 8,03� = 13.

Solucion:Se prueba facilmente por induccion que, si n = (akak−1 . . . a0)3, entonces

f(n) =k∑

j=0j impar

aj −k∑

j=0j par

aj

En efecto, f(0) = 0, f(1) = −1, f(2) = −2.

Supongamos que, para todo n menor que 3t, f(n) =∑k

j=0j impar

aj −∑k

j=0j par

aj . Entonces, si

t =(bk . . . b0

)3, 3t =

(bk . . . b00

)3, 3t+1 =

(bk . . . b01

)3, 3t+2 =

(bk . . . b02

)3. Por lo tanto,

Page 129: Olimpiada Matematica Española 1993-2010

como f(3t) = −f(t), f(3t+1) = −f(t)−1, f(3t+2) = −f(t)−2, la propiedad sigue siendocierta para todo entero n menor que 3t + 2.Luego, para todo n = (akak−1 . . . a0)3, f(n) =

∑kj=0

j imparaj −

∑kj=0

j paraj.

De esta forma, se obtiene el menor n =(2020 . . .20

)3, tal que f(n) = 2010. Este numero

contiene 1005 doses; su valor en base decimal es:

3 · 32010 − 14

• Problema 3

Sea ABCD un cuadrilatero convexo. Sea P la interseccion de AC y BD. El angulo� APD = 60◦. Sean E, F , G y H los puntos medios de los lados AB, BC, CD y DArespectivamente. Halla el mayor numero real positivo k tal que

EG + 3HF ≥ kd + (1 − k)s

siendo s el semiperımetro del cuadrilatero ABCD y d la suma de las longitudes de susdiagonales. ¿Cuando se alcanza la igualdad?

Solucion:Probaremos que k = 1 +

√3 y que la igualdad se da si, y solo si, ABCD es un rectangulo.

X

Z

YW

H

G

F

E

P

A

B C

D

Sean W , X , Y y Z cuatro puntos exteriores a ABCD de modo que los triangulos ABW yDCY sean equilateros, el triangulo BCX sea isosceles en X , el triangulo AZD sea isoscelesen Z y � BXC = � AZD = 120◦.Los cuadrilateros WAPB, XBPC, Y CPD y ZDPA son cıclicos. Luego, por el teoremade Ptolomeo, se obtiene que:

WP = PA + PB , XP√

3 = PB + PC , Y P = PC + PD , ZP√

3 = PD + PA

Por otro lado,

� WPY = � WPB + 60◦ + � CPY = � WAB + 60◦ + � CDY = 180◦

Page 130: Olimpiada Matematica Española 1993-2010

Luego W , P , Y estan alineados y, de forma analoga, Z, P , X estan alineados. Luego:

WY = WP + PY = PA + PB + PC + PD = AC + BD

XZ = XP + PZ =1√3(PB + PC + PD + PA) =

1√3(AC + BD)

Por la desigualdad, triangular:

WY ≤ WE + EG + GY , XZ ≤ XF + FH + HZ

Luego:

AC + BD ≤ AB

√3

2+ EG + DC

√3

2,

1√3(AC + BD) ≤ BC

2√

3+ FH +

AD

2√

3

Por lo tanto, sumando,

AC + BD ≤ AB√

32 + EG + DC

√3

2√3(AC + BD) ≤ BC

√3

2 + 3FH + AD√

32(

1 +√

3)(AC + BD) ≤ EG + 3FH + s

√3

o sea,EG + 3FH ≥

(1 +

√3)

d − s√

3

Luego, si k = 1 +√

3, entonces EG + 3FH ≥ kd + (1 − k)s.La igualdad se dara si, y solo si, por un lado, W , E, G, Y estan alineados y, por otro lado, X ,F , H, Z tambien estan alineados. Como WE es perpendicular a AB y GY es perpendiculara DC, AB y DC deben ser paralelas y, de forma analoga, BC y AD tambien deben serparalelas, luego ABCD debe ser un paralelogramo. Ademas, la recta EG es perpendiculara DC, lo que implica que ABCD es un rectangulo y se comprueba facilmente que si ABCDes un rectangulo, entonces se da la igualdad. Luego, la igualdad se da si, y solo si, ABCDes un rectangulo.Ahora, sea un numero real positivo l tal que EG+3HF ≥ ld+(1− l)s. Entonces, si ABCDes un rectangulo,

kd + (1 − k)s ≥ ld + (1 − l)s

o seak(d − s) ≥ l(d − s)

Pero la desigualdad triangular implica que d > s, lo que implica que k ≥ l. Luego el numeroreal buscado es k = 1 +

√3 y la igualdad se da si, y solo si, ABCD es un rectangulo.

• Problema 4

Sean a, b, c tres numeros reales positivos. Demuestra que

a + b + 3c

3a + 3b + 2c+

a + 3b + c

3a + 2b + 3c+

3a + b + c

2a + 3b + 3c≥ 15

8

Page 131: Olimpiada Matematica Española 1993-2010

Solucion:Haciendo a = x1, b = x2, c = x3 y llamando s = x1 + x2 + x3, resulta que el lado izquierdode la desigualdad se escribe como

S =s + 2x1

3s − x1+

s + 2x2

3s − x2+

s + 2x3

3s − x3

Por otro lado,

S + 6 =3∑

k=1

(s + 2xk

3s − xk+ 2

)=

3∑k=1

7s

3s − xk

con lo que

S + 6 =7s

3s − x1+

7s

3s − x2+

7s

3s − x3= 7s

3∑k=1

13s − xk

Dado que∑3

k=1 (3s − xk) = 8s, entonces s = 18

∑3k=1 (3s − xk), y

S + 6 =78

3∑k=1

(3s − xk)3∑

k=1

13s − xk

≥ 638

de donde resulta que

S =s + 2x1

3s − x1+

s + 2x2

3s − x2+

s + 2x3

3s − x3≥ 63

8− 6 =

158

La igualdad tiene lugar cuando x1 = x2 = x3, es decir, cuando a = b = c.

• Problema 5

Sea P un punto cualquiera de la bisectriz del angulo A en el triangulo ABC, y sean A′, B′,C′ puntos respectivos de las rectas BC, CA, AB, tales que PA′ es perpendicular a BC,PB′ es perpendicular a CA y PC′ es perpendicular a AB. Demuestra que PA′ y B′C′ secortan sobre la mediana AM , siendo M el punto medio de BC.

Solucion:Sea E el punto de interseccion de PA′ y B′C′. Si P se mueve sobre la bisectriz AI (I es elincentro), la figura PB′C′E es homotetica de sı misma con respecto al punto A. Luego Edescribe una recta que pasa por A. La bisectriz AI corta a la circunferencia circunscrita aABC en F , que se proyecta en el punto medio Am de BC; si P = F , la recta B′C′ es larecta de Simson de F , luego el lugar geometrico de E es la mediana AAm.

• Problema 6

Sea p un numero primo y A un subconjunto infinito de los numeros naturales. Sea fA(n) elnumero de soluciones distintas de la ecuacion x1 +x2 + · · ·+xp = n, con x1, x2, . . . , xp ∈ A.¿Existe algun numero natural N tal que fA(n) sea constante para todo n > N ?

Solucion:

Page 132: Olimpiada Matematica Española 1993-2010

Para demostrar el enunciado procederemos por contradiccion. Supongamos que existe unnumero N para el que se cumpla la propiedad anterior. Como el conjunto A es infinito,tomemos a ∈ A mayor que N . Vamos a estudiar el valor de fA(pa) y fA(pa + 1). Porhipotesis, se cumple que fA(pa) = fA(pa + 1).Sea S = (s1, s2, . . . , sp) solucion de la ecuacion

x1 + x2 + · · ·+ xp = n , s1, s2, . . . , sp ∈ A

Entonces, cualquier permutacion de los ındices da lugar a una nueva solucion de la ecuacion(posiblemente repetida si se permutan valores iguales). Diremos que una solucion S =(s1, s2, . . . , sp) es asociada a una solucion S′ =

(s′1, s

′2, . . . , s

′p

)si la primera se obtiene a

partir de la segunda mediante permutacion de ındices. Sea S = (s1, s2, . . . , sp) una soluciondel problema y sea Q = (q1, q2, . . . , qp) una solucion asociada a S con la propiedad queq1 = q2 = · · · = qr1 �= qr1+1 = qr1+2 = · · · qr1+r2 , y ası de manera sucesiva hasta llegar aqr1+r2+···+rk

= qp. En otras palabras, Q se obtiene a partir de S agrupando los valores si

que son iguales. En particular, r1 + r2 + · · · + rk = p. Con esta notacion, el numero de

soluciones asociadas a S (contando tambien S) es igual ap!

r1!r2! · · · rk!.

Observese que si todos los ri son estrictamente menores que p, entonces dicha expresion escongruente con 0 modulo p, puesto que el cociente de factoriales es un numero natural y enel denominador no hay ningun termino multiplo de p.Ya tenemos todas las herramientas que necesitabamos. Volviendo al problema original,observar que (a, a, . . . , a)︸ ︷︷ ︸

p

es solucion de x1 + x2 + · · · + xp = pa.

Por lo tanto fA(pa) ≡ 1 (mod p): la solucion (a, a, . . . , a)︸ ︷︷ ︸p

no se asocia a ninguna otra,

mientras que cualquier otra solucion de la ecuacion x1 +x2 + · · ·+xp = pa tiene un numeromultiplo de p de asociadas. Por otro lado no existen soluciones de x1 +x2+ · · ·+xp = pa+1

con todas las xi iguales (su valor tendrıa que ser a +1p), con lo que segun lo anterior

fA(pa + 1) ≡ 0 (mod p) y llegamos a una contradiccion.